Você está na página 1de 146

LivroOlimpiadaOcial

2006/5/27
page 1
i
i
i
i
i
i
i
i
Olimpadas de Matematica: uma introdu cao
Adan J. Corcho & Fernando E. Echaiz & Krerley Oliveira
27 de maio de 2006
LivroOlimpiadaOcial
2006/5/27
page 2
i
i
i
i
i
i
i
i
INTRODUC

AO
As Olimpadas de Matematica tem crescido rapidamente no Brasil, com
a participa c ao de milh oes de estudantes em competi c oes em varios nveis,
desde competi c oes municipais e estaduais ate as competi c oes nacionais e in-
ternacionais, como a Olimpada Brasileira de Matematica (www.obm.org.br)
e mais recentemente, a Olimpada Brasileira de Matematica das Escolas
P ublicas (www.obmep.org.br). Alem de serem um excelente passatempo
para os estudantes, as Olimpadas de Matematica incentivam os mais tal-
entosos para um estudo um pouco mais profundo da matematica no ensino
basico. O resultado dessa combina c ao sao alunos motivados e interessados
no estudo da Matematica, superando o tradicional tabu que existe em torno
da disciplina.
Apesar disso, os textos disponveis para o treinamento em Olimpadas de
Matematica sao escassos, em compara c ao com os numerosos e quase sempre
mal formulados livros-texto para o ensino regular. Pensando nisso, na neces-
sidade dos bons alunos e professores de ensino medio no tocante a material
adicional de matematica do ensino medio, e que escrevemos esse livro. Jun-
tando as experiencias didaticas vividas pelos autores em tres pases difer-
entes, Brasil, Cuba e Peru, esperamos tornar para o leitor a matematica
mais interessante, mostrando um pouco do imenso brilho e beleza que ela
esconde.
Agora algumas palavras de como o livro foi escrito. Os captulos estao
baseados em fatos basicos da matematica e trazem alguns teoremas fun-
damentais. Como regra geral, os estudantes podem simplesmente ler as
demonstra c oes, sem a preocupa c ao em entender os detalhes delas. Ja os
exemplos e aplica c oes dos teoremas devem ser lidos com cuidado e muita
aten c ao. Para os estudantes que desejem treinar para olimpadas de mate-
matica, sugerimos que formem grupos de estudo para trabalhar os temas
2
LivroOlimpiadaOcial
2006/5/27
page 3
i
i
i
i
i
i
i
i
3
individualmente, sob a orienta c ao de um professor.
O primeiro captulo e para introduzir o leitor no esprito do livro e
dar uma amostra do tipo de problemas e material que seguira nos demais
captulos. Sao propostos alguns problemas, muitos deles com solu c oes, e
discutimos alguns metodos importantes para uso no dia-a-dia dos estu-
dantes. Nesta discussao, inclumos o Metodo de Redu c ao ao Absurdo e
algumas regras basicas e cuidados que devemos ter ao resolver problemas
em Matematica.
O captulo seguinte trata do conceito de divisibilidade. Tentamos intro-
duzir o leitor nos principais aspectos basicos, incluindo-se a divisibilidade
com resto, maximo divisor comum e mnimo m ultiplo comum, n umeros pri-
mos e equa c oes diofantinas. A organiza c ao dos exemplos tenta seguir uma
linha em ordem crescente de diculdade e, para o melhor aproveitamento
do curso, o trabalho com os exerccios e parte fundamental.
No captulo de contagem, come camos com no c oes uteis sobre conjuntos e
princpios basicos para contar os elementos de um conjunto. Neste captulo,
estamos mais preocupados com as aplica c oes imediatas do que com os as-
pectos teoricos do assunto, sugerindo alguns problemas para o estudante
iniciante. Seguimos discutindo os tipos de agrupamento de elementos e
suas conseq uencias.
Um dos captulos mais uteis para o estudante que deseja participar de
Olimpadas de Matematica e o que trata do Princpio da Casa dos Pom-
bos. Este captulo e um belo exemplo de como algo aparentemente ingenuo
pode gerar conseq uencias interessantes. Alguns dos exemplos estao conec-
tados com os captulos anteriores e aparentemente aplicam o princpio de
modo inusitado, em problemas de geometria, teoria dos n umeros e em areas
diversas.
Finalmente, o estudante se depara com uma arma poderosa do matematico.
O captulo 5 esta dedicado a Metodo da Induc ao Finita. Novamente, procu-
ramos conectar este captulo com os captulos anteriores, re-obtendo com o
auxlio do Metodo da Indu c ao algumas coisas que ja foram deduzidas por
outros metodos. Varios exemplos e problemas sao resolvidos, alguns deles
de modo surpreendente e inesperado.
Para deixar o leitor com bastante trabalho para fazer e xar as ideias de-
senvolvidas, dedicamos um captulo a problemas de Olimpadas de Matematica.
De olimpadas regionais, nacionais e internacionais, eles apresentam um de-
sao suciente para motivar o leitor num treinamento posterior.
Somos gratos a muitas pessoas que colaboraram com a elabora c ao deste
LivroOlimpiadaOcial
2006/5/27
page 4
i
i
i
i
i
i
i
i
4
livro com sugestoes e corre c oes de muitos erros em versoes iniciais. Entre
eles, citamos: Carlos Gustavo Moreira, Ali Tahzibi, Iram Gleria, Eduardo
Teixeira, Valdenberg Ara ujo e Chico Potiguar. Finalmente, dedicamos este
livro as nossas esposas e lhos, os quais compreenderam os sabados sacri-
cados em fun c ao de escreve-lo.
LivroOlimpiadaOcial
2006/5/27
page 5
i
i
i
i
i
i
i
i
Sumario
1 Primeiros Passos 8
1.1 Tarefas Impossveis . . . . . . . . . . . . . . . . . . . . . . . . 8
1.1.1 A morte do barbeiro . . . . . . . . . . . . . . . . . . . 8
1.1.2 A prova surpresa . . . . . . . . . . . . . . . . . . . . . 9
1.1.3 Os Irm aos Gananciosos . . . . . . . . . . . . . . . . . 10
1.2 Organizando as Ideias . . . . . . . . . . . . . . . . . . . . . . 11
1.3 Verdadeiro ou Falso? . . . . . . . . . . . . . . . . . . . . . . . 13
1.4 O Metodo de Redu c ao ao Absurdo . . . . . . . . . . . . . . . 16
1.5 Dicas para Resolver Problemas . . . . . . . . . . . . . . . . . 19
2 Divisibilidade 29
2.1 Conceitos Fundamentais e Divis ao com Resto . . . . . . . . . 29
2.2 Maximo Divisor Comum e Mnimo M ultiplo Comum . . . . . 38
2.3 N umeros Primos e Compostos . . . . . . . . . . . . . . . . . . 48
2.3.1 O Crivo de Eratostenes . . . . . . . . . . . . . . . . . 50
2.3.2 O Teorema Fundamental da Aritmetica . . . . . . . . 53
2.3.3 Identicando N umeros Compostos . . . . . . . . . . . 58
2.4 Um pouco sobre Equa c oes Diofantinas . . . . . . . . . . . . . 63
2.5 Exerccios . . . . . . . . . . . . . . . . . . . . . . . . . . . . . 66
3 Contagem 69
3.1 Princpio Aditivo de Contagem . . . . . . . . . . . . . . . . . 69
3.2 Princpio Multiplicativo de Contagem . . . . . . . . . . . . . 76
3.3 Uso simultaneo dos Princpios Aditivo e Multiplicativo . . . . 82
3.4 Permuta c oes Simples . . . . . . . . . . . . . . . . . . . . . . . 85
3.5 Arranjos Simples . . . . . . . . . . . . . . . . . . . . . . . . . 87
5
LivroOlimpiadaOcial
2006/5/27
page 6
i
i
i
i
i
i
i
i
6 SUM

ARIO
3.6 Combina c oes Simples . . . . . . . . . . . . . . . . . . . . . . . 91
3.7 Contagem e Probabilidades . . . . . . . . . . . . . . . . . . . 94
3.8 Exerccios Propostos . . . . . . . . . . . . . . . . . . . . . . . 96
4 O Princpio da Casa dos Pombos 99
4.1 Primeiros Exemplos . . . . . . . . . . . . . . . . . . . . . . . 100
4.2 Uma Versao mais Geral . . . . . . . . . . . . . . . . . . . . . 102
4.3 Aplica c oes na Teoria dos N umeros . . . . . . . . . . . . . . . 103
4.4 Aplica c oes Geometricas . . . . . . . . . . . . . . . . . . . . . 105
4.5 Miscelanea . . . . . . . . . . . . . . . . . . . . . . . . . . . . 108
4.6 Exerccios . . . . . . . . . . . . . . . . . . . . . . . . . . . . . 110
5 Indu cao Matematica 111
5.1 Formula c ao Matematica . . . . . . . . . . . . . . . . . . . . . 111
5.1.1 Metodo de Indu c ao Matematica . . . . . . . . . . . . 112
5.2 Problemas Classicos . . . . . . . . . . . . . . . . . . . . . . . 113
5.2.1 Demonstrando Identidades . . . . . . . . . . . . . . . 113
5.2.2 Demonstrando Desigualdades . . . . . . . . . . . . . . 117
5.2.3 Indu c ao em problemas de divisibilidade . . . . . . . . 117
5.3 Indu c ao na Geometria . . . . . . . . . . . . . . . . . . . . . . 119
5.4 Miscelanea . . . . . . . . . . . . . . . . . . . . . . . . . . . . 123
5.5 Metodo Forte da Indu c ao . . . . . . . . . . . . . . . . . . . . 125
5.6 Exerccios . . . . . . . . . . . . . . . . . . . . . . . . . . . . . 128
6 Problemas de Olimpadas 131
LivroOlimpiadaOcial
2006/5/27
page 7
i
i
i
i
i
i
i
i
Sumario
7
LivroOlimpiadaOcial
2006/5/27
page 8
i
i
i
i
i
i
i
i
Captulo 1
Primeiros Passos
Neste captulo, faremos uma coletanea de problemas e ideias que servirao
para ilustrar varios metodos que iremos discutir nos captulos seguintes e dar
um sabor do tipo de argumento que aparece nos problemas em matematica.
Alguns dos exemplos que discutiremos serao uteis para orientar quanto ao
cuidado que devemos ter quando vamos discutir problemas em Matematica.
Come caremos com tres historias interessantes e muito intrigantes, os quais
discutiremos supercialmente.
1.1 Tarefas Impossveis
1.1.1 A morte do barbeiro
Num reino muito antigo e muito pequeno so havia um barbeiro, que cuidava
da aparencia Real. O Rei, para agradar sua popula c ao que estava insatisfeita
com seu governo, decidiu decretar o seguinte:
Decreto Real: O Barbeiro Real barbear a somente aquelas pessoas que n ao
se barbeiam. Caso esse decreto seja descumprido, ordeno a morte do
barbeiro.
O sabio da corte, mago e conhecedor de Matematica, ao ver o decreto real
que iria ser publicado, correu ao encontro do Rei, clamando para que Sua
Majestade desistisse de tal ideia. O Rei espantado, achou uma impertinencia
tal comportamento do mago, que prontamente lhe explicou suas razoes:
8
LivroOlimpiadaOcial
2006/5/27
page 9
i
i
i
i
i
i
i
i
[SEC. 1.1: TAREFAS IMPOSS

IVEIS 9
Majestade, caso Vossa Alteza decida publicar tal decreto, o pobre Bar-
beiro Real tera que ser morto, pois e humanamente impossvel cumpri-lo.
Apesar de Vossa Majestade ja saber disso, vou com sua permissao tomar a
liberdade de me explicar:
Imagine que tal decreto seja publicado e esteja em vigor em seu fabuloso
reino. Neste caso, restariam ao barbeiro duas possibilidades:
O barbeiro nao se barbeia.
Neste caso, Vossa Sabedoria obrigaria ao barbeiro a se barbear, o
que seria impossvel para essa pobre criatura, uma vez que ele nao se
barbeia. Resultado: ele seria morto.
O barbeiro se barbeia.
Neste caso, ele estaria descumprindo seu decreto de barbear somente
quem nao se barbeia. Assim ele deveria ser morto.
Note Vossa Sabedoria, que sabe mais do que eu, que ele seria morto de
qualquer modo caso tal decreto entrasse em vigor, aumentando ainda mais
a insatisfa c ao do povo. Assim, a humilde sugestao de seu servo e que em
sua gra ca e bondade, a publica c ao de tal decreto seja adiada.
Depois dessas palavras sabias e precisas do Mago da Corte, a vida do
pobre barbeiro foi salva e ele viveu ate os ultimos dias de sua vida cortando
feliz a barba de quem se barbeasse e de quem nao se barbeasse.
1.1.2 A prova surpresa
O Diretor de uma escola famosa, aborrecido com o mau desempenho dos
alunos do primeiro ano, pediu ao professor de matematica que comunicasse
aos estudantes que na semana seguinte seria realizada uma prova-surpresa,
na inten c ao de fazer com que os estudantes se dedicassem mais. Quando
conversou com o professor de matematica a esse respeito, o professor disse
que de acordo com a logica, ele nao deveria anunciar nada, pois se anunciasse
nao havia meios de que a prova fosse surpresa. Como o diretor nao aceitou
esse argumento sem uma explica c ao detalhada, o professor de matematica
teve que se explicar. Os dois tiveram a seguinte conversa:
- P: - Diretor, digamos que eu deseje realizar tal prova-surpresa. Neste
caso, nao poderei realiza-la na sexta-feira.
- D: Mas, por que?
LivroOlimpiadaOcial
2006/5/27
page 10
i
i
i
i
i
i
i
i
10 [CAP. 1: PRIMEIROS PASSOS
- P: Caso a seja realizada na sexta, na quinta-feira os alunos vao perceber
isso, pois notarao que ela nao foi realizada ate a quinta.
- D: Ah! Ta certo. Mas voce pode realizar a prova na quinta.
- P: Nao, eu tambem nao poderei realiza-la na quinta.
- D: Deixe-me entao tentar justicar isso. Veja so, como voce nao pode
realizar a prova na sexta, ela tera que acontecer entre segunda-feira e quinta-
feira. Logo, se ela for ser realizada na quinta, quando chegar a quarta-feira
os alunos perceber ao que nao houve prova ate o momento e deduzirao que
ela sera na quinta, ja que nao pode ser na sexta!
- P: Perfeito, diretor! Voce entendeu o raciocnio. Logo ela deve acon-
tecer segunda, ter ca ou quarta. Do mesmo modo, podemos argumentar e
excluir a quarta-feira.
- D: Entao ela sera na segunda!
- P: Exatamente! E neste caso deixara de ser surpresa.
- D: Entendi seu argumento, professor. Acho que vou ter que arrumar
outro castigo para esses alunos indisciplinados. Obrigado pela explica c ao!
1.1.3 Os Irmaos Gananciosos
No dia do aniversario dos gemeos Cosme e Damiao, seu pai, dono de uma
das maiores bombonieres da cidade, decidiu trazer para presentear os dois
aniversariantes uma sacola com quarenta doces. A cada minuto, eles se
alternam na escolha, podendo pegar um ou dois doces. Caso um deles
pegue dois doces, o pai pega a sacola de volta pois considera esta atitude
muito gananciosa. Todos os dois sabem disso e sabem que podem car com
vinte doces cada um, caso nao sejam gananciosos. O primeiro a escolher
seu doce e o Cosme. O que o Cosme deve fazer?
Se voce respondeu pegar um doce somente, vamos mostrar a voce que
a logica, pura e simples, vai nos levar na dire c ao contraria (apesar de toda
pessoa de bom senso come car escolhendo um doce).
Para ver isso, vamos imaginar que eles ja zeram varias escolhas e os
doces estao acabando. Como o Cosme come cou a retirar os doces, depois de
38 retiradas sucessivas havera exatamente dois doces na sacola. Neste caso,
o que o Cosme faria? Naturalmente, ele pegaria os dois doces. Porem, seu
irmao Damiao sabe disso e na escolha imediatamente anterior, quando hou-
vesse tres doces na sacola ele teria duas op c oes: tiraria um doce e deixaria
seu irmao com dois doces, ou tiraria dois doces e acabaria a distribui c ao.
LivroOlimpiadaOcial
2006/5/27
page 11
i
i
i
i
i
i
i
i
[SEC. 1.2: ORGANIZANDO AS ID

EIAS 11
Como ele conhece seu irmao e sabe que se deixasse os dois doces ele iria
tira-los, ele acabaria o jogo quando houvesse tres doces no saco.
Neste momento, argumentamos por analogia: quando houvesse quatro
doces no saco, Cosme saberia que se tirasse um doce, Damiao tiraria dois e
acabaria com a distribui c ao. Assim, ele tiraria dois doces e acabaria a dis-
tribui c ao quando houvesse quatro doces na sacola. Por sua vez, o Damiao...
Argumentando de tras-para-frente, conclumos que o Cosme deveria tirar
na sua primeira escolha dois doces e acabar com a distribui c ao. Entendeu?
1.2 Organizando as Ideias
Nesta se c ao vamos come car a montar os alicerces de nossa constru c ao em
Matematica. Para isso, precisamos ter bem claros em nossa mente alguns
conceitos que serao uteis na hora de distinguir entre o que devemos provar
e o que estamos assumindo como verdade em uma arma c ao matematica.

E sobre isso que falaremos agora. Come caremos observando as seguintes


arma c oes:
(a) Se como muitos doces, entao passo mal;
(b) Se estudo bastante, entao passo no exame de ingles.
Em ambas, estamos assumindo alguma coisa como verdadeira e a partir
disso, armamos que alguma outra coisa e valida. Na primeira arma c ao,
estamos assumindo que comemos muitos doces e armamos que, com isto,
acabamos passando mal. Em nosso estudo da Matematica, ter em mente o
que estamos assumindo como verdadeiro e o que desejamos provar e muito
importante para evitar confusoes e erros em nossos argumentos. Para facil-
itar essa divisao, vamos denir o que entendemos por proposic ao:
Deni cao 1.1. Uma proposi c ao e uma arma c ao composta de hip otese e
tese. A hipotese e tudo aquilo que assumimos como verdadeiro. A tese e
tudo aquilo que queremos mostrar.
Em nossa segunda arma c ao, estamos assumindo que estudamos bas-
tante. A hipotese neste caso e:
Hipotese: Estudo bastante.
Queremos mostrar que passaremos no exame de ingles. A tese e entao:
LivroOlimpiadaOcial
2006/5/27
page 12
i
i
i
i
i
i
i
i
12 [CAP. 1: PRIMEIROS PASSOS
Tese: Passo no exame de ingles.
Forma Direta: Vamos chamar o modo em que apresentamos uma proposi c ao
de forma direta. Por exemplo, quando enunciamos a proposi c ao Se x e uma
laranja, entao x e uma fruta, estamos enunciando sua forma direta. Vamos
descrever agora como podemos obter novas proposi c oes a partir da forma
direta de uma proposi c ao:
Forma Negativa: Para obter a forma negativa basta que neguemos a
hipotese e neguemos a tese:
Forma Direta Forma Negativa
Hipotese x e uma laranja x n ao e uma laranja
Tese x e uma fruta x nao e uma fruta
Logo a forma negativa ca sendo:
Se x n ao e uma laranja, ent ao x n ao e uma fruta.
Forma Recproca: Para construirmos a forma recproca, temos que trocar
a hipotese pela tese (e vice-versa) na forma direta. No nosso exemplo:
Forma Direta Forma Recproca
Hipotese x e uma laranja x e uma fruta
Tese x e uma fruta x e uma laranja
LivroOlimpiadaOcial
2006/5/27
page 13
i
i
i
i
i
i
i
i
[SEC. 1.3: VERDADEIRO OU FALSO? 13
Assim a Forma Recproca e:
Se x e uma fruta, ent ao x e uma laranja.
Forma Contra-recproca: Para obtermos a forma contra-recproca a par-
tir da forma direta, temos que trocar a hipotese pela tese e negar a hipotese
e a tese.
Forma Direta Forma Contra-recproca
Hipotese x e uma laranja x n ao e uma fruta
Tese x e uma fruta x n ao e uma laranja
Logo a forma contra-recproca ca sendo:
Se x n ao e uma fruta, ent ao x n ao e uma laranja.
1.3 Verdadeiro ou Falso?
Uma das coisas que distingue a Matematica das demais Ciencias Naturais e
o fato de que um tema de Matematica e discutido utilizando-se a logica pura
e, por conta disso, uma proposi c ao em Matematica uma vez demonstrada
atraves de argumentos logicos corretos, e aceita como verdade e nao sofre
grandes altera c oes ao longo dos tempos.
Como conseq uencia desse fato e que os resultados provados em Matematica
`as vezes podem tornar seus descobridores ilustres e se aplicam atraves dos
seculos. Por exemplo, o Teorema de Pitagoras e utilizado ate hoje do mesmo
modo que foi descoberto ha mais de 2.000 anos e seguira do mesmo jeito
nos proximos 2.000 anos.
Vamos ilustrar melhor essa diferen ca com um exemplo em Geograa:
Hoje, todos nos sabemos que a Terra tem aproximadamente o formato de
uma laranja, um pouco achatada nos polos. Porem, na epoca de Pitagoras,
um dos grandes temores dos navegadores era encontrar o m-do-mundo.
No pensamento de alguns destes aventureiros, a Terra tinha o formato de
um cubo, e uma vez chegando em um dos seus extremos, o navio despen-
caria no vazio. Esse e um dos muitos exemplos de como a concep c ao da
LivroOlimpiadaOcial
2006/5/27
page 14
i
i
i
i
i
i
i
i
14 [CAP. 1: PRIMEIROS PASSOS
natureza mudou ao longo do tempo, transformando uma proposi c ao ver-
dadeira num perodo da humanidade em algo completamente falso em outra
epoca. Porem, para nossa felicidade, isso nao acontece na Matematica.
Uma proposi c ao ou e verdadeira ou e falsa e permanecera deste modo pelos
milenios.
Mas como saber se uma proposi c ao e verdadeira ou falsa? A primeira
coisa que devemos fazer e tomar muito cuidado. As aparencias enganam
ou, como diziam nossos avos, nem tudo que reluz e ouro.O leitor, avisado
disso, pense agora na seguinte pergunta:
Qual e a chance de que pelo menos duas pessoas num onibus com 44
passageiros fa cam aniversario no mesmo dia do ano?
Como ja avisamos, o leitor deve ter cuidado ao responder a pergunta
acima, pois podemos nos enganar muito facilmente. Por exemplo, podemos
formular o seguinte argumento errado: o ano tem 365 dias e, como estou
escolhendo um grupo de 44 pessoas ao acaso, e claro que a chance de que
num grupo de 44 pessoas pelo menos duas delas fa cam aniversario no mesmo
dia do ano e pequena, pois 44 e um n umero muito pequeno com respeito
a 365.
`
A primeira vista isso pode ate parecer verdadeiro, mas com uma
analise mais cuidadosa veremos que e completamente falso. Na verdade,
a chance de que pelo menos duas pessoas do onibus fa cam aniversario no
mesmo dia do ano e de cerca de 80%!
Quem nao acreditar nisto pode fazer duas coisas: primeiro, ir a sua
sala de aula ou no seu onibus escolar, que deve ter pelo menos 40 pessoas,
e fazer o experimento ao vivo. Muito provavelmente voce deve conseguir
duas pessoas que fazem aniversario no mesmo dia do ano. Se voce verica
que existiam duas pessoas que faziam aniversario no mesmo dia do ano, nao
foi por acaso, pois a chance de isso acontecer era muito alta. Mas, cuidado!
Isso nao e uma prova matematica para este fato. Para provar que este fato e
verdadeiro voce deve vericar que se escolhermos ao acaso um grupo de 44
pessoas entao com 80% de chance pelo menos duas delas fazem aniversario
no mesmo dia do ano.
Porem, se voce faz o experimento e nao encontra duas pessoas que fazem
aniversario no mesmo dia do ano, nao se desespere. Lembre-se de que se
trata de algo que acontece com chance de 80% e que pode nao acontecer
quando fazemos um teste. Em qualquer um dos casos, para ter a certeza de
que a proposi c ao e verdadeira o leitor deve passar para o Capitulo 3, onde
demonstraremos matematicamente que o que armamos e verdade.
LivroOlimpiadaOcial
2006/5/27
page 15
i
i
i
i
i
i
i
i
[SEC. 1.3: VERDADEIRO OU FALSO? 15
Vamos analisar agora outro fato aparentemente obvio:
Num campeonato de futebol onde cada time joga a mesma quantidade
de jogos, cada vitoria vale tres pontos, o empate vale um ponto e a
derrota nenhum ponto. Em caso de empate, o criterio de desempate
entre as equipes era o seguinte:
- A melhor equipe e aquela que tem mais vitorias.
Os organizadores decidiram passar a adotar o criterio:
- A melhor equipe e aquela que tem mais derrotas.
Voce acha que este criterio e justo?
Com respeito a esta pergunta, a resposta do leitor deve ter sido que a
mudan ca de criterio e totalmente injusta (acertamos a sua resposta?), pois
um time que perdeu mais e pior que um que perdeu menos. Na verdade, nao
houve mudan ca nenhuma de criterio! Para ver isso rapidamente, lembre-se
de que se a equipe A perdeu mais que a equipe B e ainda assim empataram,
entao ela deve ter ganho mais, para que no m do campeonato a equipe A
ainda assim conseguisse empatar com a equipe B. Vamos mostrar isso
precisamente. Sejam a
1
, a
2
, a
3
o n umero de derrotas, empates e vitorias,
respectivamente, da equipe A. Do mesmo modo, sejam b
1
, b
2
, b
3
o n umero de
derrotas, empates e vitorias, respectivamente, da equipe B. Nossa hipotese
e de que a equipe A obteve mais vitorias que a equipe B, ou seja, que
a
3
> b
3
. Como cada equipe jogou o mesmo n umero de jogos, temos que
a
1
+ a
2
+ a
3
= b
1
+ b
2
+ b
3
. (1.1)
Por outro lado, note que o n umero de pontos obtidos pela equipe A e a
2
+
3a
3
. Do mesmo modo, o n umero de pontos obtidos pela equipe B e igual a
b
2
+ 3b
3
. Como as duas empataram, temos que:
a
2
+ 3a
3
= b
2
+ 3b
3
.
Ou ainda,
3(a
3
b
3
) = b
2
a
2
ou b
3
a
3
=
b
2
a
2
3
.
Como a
3
b
3
> 0 temos que b
2
a
2
> 0. Reescrevendo a equa c ao (1.1),
temos que:
a
1
b
1
= b
2
a
2
+ (b
3
a
3
) = b
2
a
2

b
2
a
2
3
=
2
3
(b
2
a
2
).
LivroOlimpiadaOcial
2006/5/27
page 16
i
i
i
i
i
i
i
i
16 [CAP. 1: PRIMEIROS PASSOS
Logo, temos que a
1
b
1
> 0, pois b
2
a
2
> 0. Isso signica que A teve
mais derrotas que B, como queramos demonstrar.
Assim, como estes dois exemplos mostram, ao depararmos com um prob-
lema em Matematica, devemos ter cuidado ao tirar conclus oes apressadas
para evitar que cometamos algum engano. Pode acontecer que uma situa c ao
que e claramente falsa para um observador menos atento, se mostre ver-
dadeira quando fazemos uma analise mais criteriosa.
1.4 O Metodo de Reducao ao Absurdo
Um detetive esta investigando um caso de roubo de um objeto que ocorreu
numa sala onde havia somente duas pessoas. Naturalmente, o criminoso
se encontra entre as duas pessoas que estavam na sala. Apos investigar
criteriosamente a primeira pessoa, ele chega `a conclus ao de que esta pessoa
e inocente. Logo, o crime foi cometido pela segunda pessoa.
Apesar de sua simplicidade, o metodo descrito no paragrafo anterior e
uma das ferramentas mais poderosas da Matematica. Nos conhecemos este
metodo pelo nome de Metodo de Reduc ao ao Absurdo ou Metodo do Terceiro
Excludo. Resumidamente, se uma arma c ao nao e verdadeira, ela so pode
ser falsa. Para escrever o metodo em termos de proposi c oes, devemos seguir
o seguinte roteiro:
Assumindo a Hipotese como verdade, tentamos mostrar que a nossa
Tese e verdadeira.
Se nao conseguimos isso diretamente, supomos que a Tese e falsa.
Apos argumentarmos corretamente, deduzimos que a Hipotese tem
que ser falsa. Isso e um absurdo, pois estamos assumindo que a
Hipotese e verdadeira.
Deste modo, nao podemos supor que a tese e falsa, onde ela deve ser
verdadeira.
Em termos de proposi c oes, podemos concluir que o Metodo de Redu c ao
ao Absurdo pode ser formulado do seguinte modo:
Para mostrar que uma proposi cao e verdadeira, basta mostrar
que sua forma contra-recproca e verdadeira.
LivroOlimpiadaOcial
2006/5/27
page 17
i
i
i
i
i
i
i
i
[SEC. 1.4: O M

ETODO DE REDUC

AO AO ABSURDO 17
Em resumo, podemos dizer que o metodo da redu c ao ao absurdo e:
Forma Direta Forma Contra-recproca
O smbolo signica que a Forma Direta de uma proposi c ao e verdadeira
se, e somente se, a sua Forma Contra-recproca tambem o for.
Vamos mostrar como esse metodo funciona na pratica. Dizemos que um
n umero natural n e par, se existir outro n umero natural k tal que n = 2k.
No caso contrario, este n umero e chamado de mpar e se escreve da forma
2k + 1 para algum n umero natural k.
Considere a seguinte proposi c ao:
Proposi cao 1.2. Se n e par, ent ao n
2
e par.
Hipotese: n e par.
Tese: n
2
e par.
Apesar de parecer obvia, esta proposi c ao merece uma prova matematica
rigorosa. Vamos dar essa prova:
Demonstrac ao. Se n e par, entao pela deni c ao de n umero par, deve existir
um n umero natural k tal que
n = 2k.
Logo, n
2
= (2k)(2k) = 2(2k
2
). Denotando q = 2k
2
, temos que n
2
= 2q e
par tambem.
Vamos agora considerar uma outra proposi c ao:
Proposi cao 1.3. Se n
2
e par, ent ao n e par.
Hipotese: n
2
e par.
Tese: n e par.
LivroOlimpiadaOcial
2006/5/27
page 18
i
i
i
i
i
i
i
i
18 [CAP. 1: PRIMEIROS PASSOS
n
2
n
4 2
16 4
36 6
64 8
100 10
144 12
Desaamos o leitor a tentar mostrar esta proposi c ao partindo da hipotese e
tentando concluir a tese. Note que podemos vericar que nossa proposi c ao
e verdadeira para varios valores de n
2
como na tabela abaixo, mas isso nao
e uma prova matematica da nossa proposi c ao.
Mesmo vericando para um bilhao de valores de n
2
, sempre nos restariam
n umeros para serem vericados. Como nossas tentativas de provar a forma
direta desta proposi c ao estao sendo frustradas, neste momento iremos apelar
para o Metodo de Redu c ao ao Absurdo, ou seja, para mostrar a Proposi c ao 1.3,
iremos mostrar a sua forma contra-recproca:
Negativa da Tese: n nao e par.
Negativa da Hipotese: n
2
nao e par.
Demonstrac ao. Como estamos assumindo que n nao e par, logo n tem que
ser mpar, ou seja, existe k, n umero natural, tal que n = 2k + 1. Logo,
n
2
= (2k + 1)(2k + 1)
= 4k
2
+ 2k + 2k + 1
= 4k
2
+ 4k + 1
= 2(2k
2
+ 2k) + 1,
chamando q = 2k
2
+ 2k, temos que:
n
2
= 2q + 1
Logo, n
2
e mpar, de onde conclumos a prova.
LivroOlimpiadaOcial
2006/5/27
page 19
i
i
i
i
i
i
i
i
[SEC. 1.5: DICAS PARA RESOLVER PROBLEMAS 19
Isto mostra um pouco do poder do Metodo de Redu c ao ao Absurdo.
Antes de usa-lo, nao tnhamos a menor ideia de como mostrar a proposi c ao 2.
Aplicando o metodo, construmos uma proposi c ao equivalente a proposi c ao
2, cuja prova conseguimos obter sem maiores diculdades.
1.5 Dicas para Resolver Problemas
Nesta se c ao, estudaremos algumas regras geraisque devemos ter em mente
na hora de resolver um problema de matematica. Aplicaremos estas re-
grasa alguns problemas interessantes para ilustrar sua import ancia. Vamos
`a primeira regra:
Regra 1: Ler bem o enunciado do problema e utilizar todas as in-
forma c oes disponveis.
Problema 1.4. Ao encontrar uma velha amiga, um matem atico tem a
seguinte conversa:
M: Como v ao os tres lhos da senhora?
S: V ao bem, obrigada!
M: Qual a idade deles mesmo?
S: Vou lhe dar dicas. O produto das idades deles e 36.
M: S o com essa dica e impossvel!
S: A soma das idades e igual ao n umero daquela casa (apontou para uma
casa em frente aos dois).
M: Ainda n ao sei!
S: O mais velho toca piano!
M: Agora eu sei!
Voce e capaz de descobrir as idades dos tres lhos da senhora?
Soluc ao.

E muito importante neste problema tirar o maximo de informa c ao
das dicas da senhora. Vamos `a primeira dica: o produto das idades e 36.
Digamos que as idades dos lhos sejam 0 x y z 36. Como
xyz = 36, temos as seguintes possibilidades:
LivroOlimpiadaOcial
2006/5/27
page 20
i
i
i
i
i
i
i
i
20 [CAP. 1: PRIMEIROS PASSOS
x y z
1 1 36
1 2 18
1 3 12
1 4 9
1 6 6
2 2 9
2 3 6
3 3 4
Vamos agora calcular a soma das idades:
x y z x + y + z
1 1 36 38
1 2 18 21
1 3 12 16
1 4 9 14
1 6 6 13
2 2 9 13
2 3 6 11
3 3 4 10
Observe que nao conhecemos a princpio o n umero da casa. Porem, sabe-
mos que apos a segunda dica, o matematico ainda nao conseguiu deduzir as
idades das crian cas.
Por que ele nao conseguiu? Imagine que o n umero da casa fosse 14. Ora,
de acordo com nossa tabela, so existe um terno de n umeros cujo produto e
36 e a soma e 14, que e o terno (1,4,9). Assim, se o n umero da casa fosse
14 o matematico teria dado a resposta apos a segunda dica. Como ele cou
em d uvida, olhando a tabela 2, chegamos `a conclus ao de que o n umero da
casa so pode ser igual a 13.

Ultima dica: O mais velho toca piano.


No incio essa dica parecia in util, mas agora ela e fundamental para resolver-
mos o problema. De fato, como o mais velho toca piano, isso signica que
LivroOlimpiadaOcial
2006/5/27
page 21
i
i
i
i
i
i
i
i
[SEC. 1.5: DICAS PARA RESOLVER PROBLEMAS 21
existe um mais velho, o que descarta o caso (1,6,6). Assim, as idades sao 2,
2, e 9.
Problema 1.5. Numa cesta encontram-se 9 moedas identicas, sendo que
oito delas tem o mesmo peso e uma moeda e mais leve que as demais.
Usando duas vezes uma balanca de dois pratos, encontrar a moeda mais
leve.
Soluc ao. Este e o tipo de problema que a primeira vista pode parecer difcil,
mas que quando usamos todas as informa c oes do seu enunciado se torna
facil. A ideia e dividir as moedas em tres grupos de tres moedas cada, que
chamaremos grupos A, B e C. Colocaremos na balan ca os grupos A e B e
deixaremos o grupo C fora. Podem acontecer duas coisas:
(a) Os pratos cam equilibrados;
(b) Os pratos cam desequilibrados.
No caso 1, temos que os grupos A e B tem o mesmo peso. Logo, a moeda
mais leve deve estar no grupo C. No caso 2, um dos grupos cou mais leve,
o que signica que a moeda mais leve esta neste grupo. Assim, utilizando
a balan ca apenas uma vez conseguiremos descobrir qual e o grupo em que
a moeda mais leve esta. Digamos que este grupo seja o grupo A. Para
achar a moeda mais leve, procedemos de modo semelhante ao que zemos
anteriormente: separamos as tres moedas do grupo A colocando uma em
cada prato e deixando a terceira de fora. Podem acontecer duas coisas:
(a) Os pratos cam desequilibrados e assim a moeda mais leve esta no
prato mais leve;
(b) Os pratos cam equilibrados, logo a moeda mais leve foi a que cou
fora.
No nal, usamos a balan ca exatamente duas vezes.
Regra 2: Fazer casos particulares ou casos mais simples de problemas
similares, para adquirir familiaridade com o problema.
Problema 1.6. Numa pequena ilha existem 5 pessoas de olhos azuis e 5
pessoas de olhos verdes. Existe um grande tabu nesta ilha que e o seguinte:
LivroOlimpiadaOcial
2006/5/27
page 22
i
i
i
i
i
i
i
i
22 [CAP. 1: PRIMEIROS PASSOS
se uma pessoa descobre que possui olhos azuis ela se suicida ` a meia-noite
do dia em que descobriu, pulando do alto da prefeitura. Por conta disso,
ninguem conversa sobre o assunto, olha para espelhos ou ve seu reexo na
agua. Todos se cruzam diariamente e conhecem os olhos de seus amigos.
Numa manh a, um estrangeiro chegou a ilha e reuniu as dez pessoas para o
seguinte pronunciamento:
Nesta ilha, existe uma pessoa de olhos azuis.
Pergunta-se:
1. O que aconteceu com os habitantes da ilha?
2. Que informac ao nova o estrangeiro trouxe?
Dicas:
1. Fazer o caso uma pessoa com olhos azuis e uma com olhos verdes;
2. Fazer o caso de duas pessoas de olhos azuis e duas de olhos verdes.
Soluc ao. Como em muitos problemas de matematica, abordar casos mais
simples do problema pode ajudar bastante na solu c ao. Assim, vamos imagi-
nar o seguinte caso mais simples: na Ilha existe somente uma pessoa de olhos
azuis e a outra de olhos verdes. Pensando neste caso, a pessoa que tinha
olhos azuis so via as que tinham olhos verdes. Quando o estrangeiro armou
que existia uma pessoa de olhos azuis, ela descobriu que tinha olhos azuis,
pois as outras pessoas tinham olhos verdes. Assim, `a meia-noite ela subiu
na prefeitura e pulou. Com isso, a pessoa que tinha olhos verdes descobriu
que tinha olhos verdes, pois se ela tivesse olhos azuis sua companheira nao
se suicidaria no dia anterior.
Vamos agora dar um passo crucial na solu c ao do nosso problema original,
considerando o caso onde existem duas pessoas de olhos azuis e duas pessoas
de olhos verdes na ilha. Vamos chamar as pessoas de olhos azuis de A e
B e as pessoas de olhos verdes de C e D. No dia em que o estrangeiro
fez o seu pronunciamento, nada aconteceu, pois as pessoas C e D viam as
pessoas A e B com olhos azuis e a pessoa A via a pessoa B com olhos azuis
e vice-versa. Ja no segundo dia, a pessoa A teve o seguinte pensamento:
LivroOlimpiadaOcial
2006/5/27
page 23
i
i
i
i
i
i
i
i
[SEC. 1.5: DICAS PARA RESOLVER PROBLEMAS 23
Se eu tivesse olhos verdes, a pessoa B teria descoberto que tinha
olhos azuis ontem, pois ela veria tres pessoas de olhos verdes.
Como ela n ao se suicidou ontem, eu tenho olhos azuis.
Pensando da mesma forma, a pessoa B descobriu que tambem tinha
olhos azuis. Por isso, `a meia noite do segundo dia, as pessoas A e B se
suicidaram.
O que aconteceu depois? As pessoas C e D ainda tinham a d uvida da
cor de seus olhos. Para chegar `a conclus ao de que seus olhos sao verdes, no
terceiro dia, a pessoa C pensou assim:
Bem, se eu tivesse olhos azuis, as pessoas A e B veriam cada
uma duas pessoas com olho azul. Logo, elas n ao teriam se sui-
cidado no segundo dia, pois n ao conseguiriam deduzir a cor de
seus olhos. Logo, tenho olhos verdes. Ufa!
Do mesmo modo, a pessoa D conseguiu descobrir a cor de seus olhos.
Analisando de modo semelhante, conseguiremos deduzir que no pro-
blema original as cinco pessoas de olhos azuis descobrirao que possuem
olhos azuis e juntas se suicidarao no quinto dia apos o pronunciamento do
estrangeiro.
Agora vamos descobrir a resposta da segunda pergunta do enunciado:
que informa c ao nova o estrangeiro trouxe? Aparentemente nada de novo
foi acrescentado pela frase do estrangeiro, pois cada pessoa estava vendo
alguma pessoa com olhos azuis. Mas isso nao e verdade.
Para ver isso e descobrir qual e a nova informa c ao que o estrangeiro
trouxe, vamos voltar ao caso de somente duas pessoas na Ilha, uma de olhos
azuis e outra de olhos verdes. Neste caso, a pessoa de olhos azuis somente
ve uma pessoa de olhos verdes. Com a informa c ao de que existe uma pessoa
de olhos azuis ela pode descobrir a cor de seus olhos. Note que a pessoa de
olhos verdes ja sabia que existia pelo menos uma pessoa de olhos azuis. Mas
ela nao sabia que a pessoa de olhos azuis tinha conhecimento de que na Ilha
existia alguem com olhos azuis. Essa e a nova informa c ao que o estrangeiro
trouxe.
Problema 1.7. Um viajante deseja se hospedar durante 31 dias num hotel.
Entretanto, percebe que est a sem dinheiro e que a unica coisa que possui e
uma corrente com 31 elos de ouro (veja gura). Para pagar sua conta, ele
LivroOlimpiadaOcial
2006/5/27
page 24
i
i
i
i
i
i
i
i
24 [CAP. 1: PRIMEIROS PASSOS
acertou com o gerente pagar um elo por dia, sem atrasar ou adiantar o
pagamento, durante os 31 dias. O gerente pode dar troco em elos. Depois
ele deseja recuperar a corrente e por isso ele quer pagar a conta cortando
a corrente no menor n umero de pedacos. Quantos cortes voce conseguiria
dar e pagar a conta?
Soluc ao. Uma primeira solu c ao e cortar a corrente 30 vezes, separando todos
os elos. Porem, essa nao e a melhor solu c ao, como veremos a seguir. Vamos
iniciar nossa analise observando que para pagar o primeiro dia precisamos
dar um corte na corrente. Assim, o gerente recebera um elo. O pulo do
gatodo problema vem agora: para pagar o 2

dia, vamos cortar a corrente


de modo a separar dois elos de uma vez. Assim, daremos dois elos ao gerente
e ele devolvera um elo de troco. Com este elo pagaremos o terceiro dia. Note
que pagamos tres dias fazendo dois cortes na corrente, como mostra a tabela:
Gerente Viajante
Elos 1, 2 28
Note que o n umero 2 denota o peda co que contem 2 elos. Para pagar o
4

dia, cortaremos a corrente de modo a obter um peda co com quatro elos.


Entregamos ao gerente este peda co e recebemos de troco um elo solto e um
peda co com dois elos. Com o elo solto, pagamos o 5

dia. Assim, no 5

dia
teremos o seguintes grupos de elos:
Gerente Viajante
Elos 1, 4 2, 24
Assim, pagamos o 6

dia com o peda co que contem dois elos e rece-


beremos o elo solto de troco. Finalmente pagaremos o 7

dia com o elo


solto. Note que foi possvel pagar 7 dias com apenas tres cortes na corrente.
A continua c ao do procedimento esta quase revelada. Para pagar o 8

dia,
cortaremos um peda co com oito elos. Daremos este peda co e receberemos
de troco 7 elos, sendo um elo solto, um peda co com 4 e um peda co com dois
elos. Repetindo o procedimento anterior, pagaremos os 7 dias seguintes, pa-
gando ate o 15

dia sem precisar de cortes adicionais. Ou seja, para pagar


LivroOlimpiadaOcial
2006/5/27
page 25
i
i
i
i
i
i
i
i
[SEC. 1.5: DICAS PARA RESOLVER PROBLEMAS 25
os 15 primeiros dias, precisamos de 4 cortes na corrente. Neste momento, a
corrente esta distribuda do seguinte modo:
Gerente Viajante
Elos 1, 2, 4, 8 16
Para pagar o 16

dia, entregaremos ao gerente o peda co com os 16 elos


restantes, recebendo 15 elos divididos em peda cos de 1, 2, 4 e 8 elos. Se
repetirmos o processo, pagaremos o hotel ate o 31

dia sem precisar de novos


cortes. Assim, o mnimo n umero de cortes e 4.
Regra 3: Mudar a representa c ao do problema, transformando-o em
um problema equivalente.
Problema 1.8. Sabendo que em cada jogada o movimento do cavalo con-
siste em se deslocar duas casas na horizontal e uma na vertical ou duas
na vertical e uma na horizontal, decidir se e possvel sair do tabuleiro 1 e
chegar ao tabuleiro 2 sem que em algum momento existam dois cavalos na
mesma casa.
(a) (b)
Figura 1.1::
Soluc ao: Para resolver este problema vamos usar a estrategia de mudar a
representac ao. O que signica isso? Vamos reescrever o problema com
LivroOlimpiadaOcial
2006/5/27
page 26
i
i
i
i
i
i
i
i
26 [CAP. 1: PRIMEIROS PASSOS
outros ingredientes, porem sem alterar em nada sua essencia. Primeira-
mente, enumere as casas do tabuleiro com os n umeros 1, 2, . . . , 9, como na
Figura 1.5.
1 2 3
4 5 6
7 8 9
Figura 1.2::
Vamos agora associar ao tabuleiro, um conjunto de nove pontos tambem
enumerados com os n umeros 1,2, . . . , 9. Se for possvel sair de uma casa i e
chegar `a casa j com apenas uma jogada do cavalo, colocaremos um segmento
ligando os pontos i e j. Por exemplo, e possvel, saindo da casa 1 chegar
`a casa 6 e a casa 8. Deste modo, o ponto com n umero 1 esta ligado com
o ponto com n umero 8. Se analisarmos todas as possveis liga c oes entre os
pontos obteremos a seguinte gura:


1
8 6
3
4
9
2
7

Figura 1.3::
Assim, se colocarmos os cavalos como no tabuleiro 1, teremos a situa c ao
descrita na Figura 1.4. Deste modo, ca evidente que nao podemos trocar a
LivroOlimpiadaOcial
2006/5/27
page 27
i
i
i
i
i
i
i
i
[SEC. 1.5: DICAS PARA RESOLVER PROBLEMAS 27
posi c ao dos cavalos branco e preto sem que em algum momento eles ocupem
a mesma casa.


1
8 6
3
4
9
2
7
CB

CB
CP
CP
Figura 1.4: CB=cavalo branco e CP=cavalo preto.
Regra 4: Use sua imagina c ao pesquisando caminhos alternativos.
Extrapole seus limites!
Para exemplicar como o enunciado de um problema pode limitar nossa
imagina c ao, vamos considerar o problema abaixo:
Exemplo 1.9. Mostre que podemos cobrir os 9 pontos no reticulado abaixo
tra cando 4 segmentos de reta sem tirar o lapis do papel.

Figura 1.5: Cobrir os 9 pontos com quatro segmentos


O problema acima e o tpico problema de bar. Vou explicar o porque
da terminologia estranha: quando voce encontra um conhecido que voce
nao ve ha muito tempo e fala que esta estudando Matematica, ele vem logo
LivroOlimpiadaOcial
2006/5/27
page 28
i
i
i
i
i
i
i
i
28 [CAP. 1: PRIMEIROS PASSOS
com um desses desaos, imaginando que voce deve ser uma maquina de
resolver problemas. Resolva isso, arma ele, esperando que voce nao consiga
imaginar a solu c ao. Para se vingar desses inoportunos, daremos a solu c ao
desse problema e complementaremos a receita dando uma generalizac ao no
exemplo 5.11 do Captulo 5.
A solu c ao desse problema passa por extrapolar o convencional. Podemos
tentar varias vezes cobrir os nove pontos com quatro segmentos sem tirar o
lapis do papel, mas so iremos de fato conseguir fazer isso se libertarmos as
nossas ideias e nos permitirmos sair um pouco do convencional. Se voce e
curioso e nao quer tentar por si so resolver esse problema, veja a solu c ao no
exemplo 1.9 do Captulo 5. Recomendamos que voce tente resolver sozinho
o desao.
LivroOlimpiadaOcial
2006/5/27
page 29
i
i
i
i
i
i
i
i
Captulo 2
Divisibilidade
A Teoria dos N umeros e o ramo da matematica encarregado de desvendar
os misterios dos n umeros e teve sua origem na antiga Grecia. Os belssimos
problemas ligados a esta area constituem, ate hoje, uma das principais fontes
inspiradoras dos amantes da Matematica.
Apesar de se terem passado muitos anos, ainda hoje persistem muitas
questoes naturais e simples sem resposta e outras, da mesma natureza, so
tem sido resolvidas recentemente. Isto faz desta area um grande atrativo
para os matematicos do mundo todo.
Este captulo sera dedicado ao estudo de algumas propriedades basicas
relativas aos n umeros inteiros.
2.1 Conceitos Fundamentais e Divisao com
Resto
Denotamos por Z o conjunto dos n umeros inteiros formado pelo conjunto dos
n umeros naturais N = {1, 2, 3, } munido do zero e dos inteiros negativos,
ou seja, Z = { , 3, 2, 1, 0, 1, 2, 3, }.
Come camos observando que a soma, diferen ca e produto de n umeros
inteiros tambem serao n umeros inteiros. Entretanto, o quociente de dois
inteiros pode ser um inteiro ou nao.
29
LivroOlimpiadaOcial
2006/5/27
page 30
i
i
i
i
i
i
i
i
30 [CAP. 2: DIVISIBILIDADE
Uma das propriedades fundamentais dos n umeros naturais que utilizare-
mos ao longo do texto e o conhecido Princpio da Boa Ordenac ao. Este
princpio pode ser enunciado do seguinte modo:
Princpio da Boa Ordena cao: Todo subconjunto n ao-vazio A N possui
um elemento menor que todos os outros elementos deste.
Por exemplo, se A e o conjunto dos n umeros pares, o menor elemento de
A e o n umero 2. Por outro lado, observemos que o conjunto dos n umeros
inteiros nao goza da boa ordena c ao.
Apesar do Princpio da Boa Ordena c ao parecer inocente e natural, muitos
resultados importantes a respeito dos n umeros naturais decorrem do mesmo,
como veremos ao longo de todo este captulo.
Deni cao 2.1. Sejam a e b inteiros. Dizemos que a divide b, ou a e divisor
de b, ou b e m ultiplo de a, se existe um inteiro q tal que b = aq.
Usaremos a nota c ao: a | b para representar todas as frases equivalentes
ditas anteriormente. Se a nao for divisor de b, entao escreveremos a b.
Exemplo 2.2. 7 | 21 pois 21 = 7 3. Por outro lado 3 8 pois considerando
o conjunto M = {3k, k N} = {3, 6, 9, 12, } dos m ultiplos positivos de
3 vemos que 8 nao pertence ao mesmo.
A seguinte proposi c ao e um bom exerccio para entender os conceitos
enunciados acima.
Proposi cao 2.3. Sejam a, b e c n umeros inteiros. Ent ao
(a) Se a | b e b | c ent ao a | c;
(b) Se a | b e a | c ent ao a | (b + c) e a | (b c);
(c) Se a e b s ao positivos e a | b ent ao 0 < a b;
(d) Se a | b e b | a ent ao a = b ou a = b.
Demonstrac ao. Se a | b e b | c entao existem inteiros q
1
e q
2
tais que
b = aq
1
(2.1)
e
c = bq
2
. (2.2)
LivroOlimpiadaOcial
2006/5/27
page 31
i
i
i
i
i
i
i
i
[SEC. 2.1: CONCEITOS FUNDAMENTAIS E DIVIS

AO COM RESTO 31
Substituindo (2.1) em (2.2) temos que
c = a q
1
q
2
..
qZ
, (2.3)
provando isto a arma c ao feita em (a).
Agora provaremos (b). Com efeito, se a | b e a | c valem as igualdades
b = aq
1
, q
1
Z (2.4)
e
c = aq
2
, q
2
Z. (2.5)
Operando com os ambos lados das igualdades (2.4) e (2.5) temos que
b + c = a(q
1
+ q
2
. .
rZ
) e b c = a(q
1
q
2
. .
sZ
),
obtendo assim o resultado desejado.
Continuamos agora com a prova de (c). De fato, se a | b, sendo ambos
positivos, entao b = aq com
q 1. (2.6)
Logo, multiplicando por a ambos lados de (2.6) temos (como a e positivo)
que
b = aq a > 0,
como esperavamos.
Finalmente, provaremos (d). Com este proposito observamos que se
a | b e b | a entao |a| divide |b| e |b| divide |a|. Portanto, pelo item (c)
temos que |a| |b| e |b| |a|, ou seja, |a| |b| |a|. Logo, |a| = |b| e
conseq uentemente a = b ou a = b.
Exemplo 2.4. Prove que o n umero N = 5
45362
7 nao e divisvel por 5.
Soluc ao. Vamos mostrar isso utilizando o metodo do absurdo. Se este
n umero fosse divisvel por 5, entao 5
45362
7 = 5q. Logo, 7 = 5
45362
5q,
ou seja, 7 seria divisvel por 5, o que e um absurdo.
A proxima proposi c ao apresenta alguns dos criterios de divisibilidade que
sao estudados nas li c oes de matematica do colegio. A prova que daremos
LivroOlimpiadaOcial
2006/5/27
page 32
i
i
i
i
i
i
i
i
32 [CAP. 2: DIVISIBILIDADE
para estes esta baseada na representa c ao de um inteiro positivo na base
decimal. Por exemplo, 345 escreve-se na base decimal da seguinte forma
345 = 300 + 40 + 5 = 3 10
2
+ 4 10 + 5,
assim como 2768 se escreve da forma
2768 = 2000 + 700 + 60 + 8 = 2 10
3
+ 7 10
2
+ 6 10 + 8.
De modo geral, se denotamos por N = b
n
b
n1
b
1
b
0
o n umero inteiro
positivo formado pelos algarismos b
n
, b
n1
, , b
1
e b
0
, nessa ordem, entao
N se escreve na base decimal da forma
N = b
n
00 0
. .
n vezes
+ b
n1
00 0
. .
n1 vezes
+ + b
1
0 + b
0
= b
n
10
n
+ b
n1
10
n1
+ b
1
10 + b
0
(2.7)
Proposi cao 2.5 (Criterios de divisibilidade). Seja N um inteiro posi-
tivo, ent ao:
(a) N e divisvel por 3 ou por 9, se a soma dos seus dgitos e divisvel por
3 ou por 9, respectivamente;
(b) N e divisvel por 5, se seu ultimo dgito for 0 ou 5.
Demonstrac ao. Faremos uso da seguinte propriedade que e muito facil de
vericar. Dado qualquer n umero m N vale que
10
m
= 99 9
. .
m vezes
+1.
Como ja foi observado, se N = b
n
b
n1
b
0
, entao
N = b
n
10
n
+ b
n1
10
n1
+ + b
1
10 + b
0
= b
n
(99 9
. .
n vezes
+1) + b
n1
( 99 9
. .
n1 vezes
+1) + + b
1
(9 + 1) + b
0
= b
n
99 9
. .
n vezes
+b
n1
99 9
. .
n1 vezes
+ + b
1
9 + b
n
+ b
n1
+ + b
0
= 9
_
b
n
11 1
. .
n vezes
+b
n1
11 1
. .
n1 vezes
+ + b
1
_
+b
n
+ b
n1
+ + b
0
. .
S
.
LivroOlimpiadaOcial
2006/5/27
page 33
i
i
i
i
i
i
i
i
[SEC. 2.1: CONCEITOS FUNDAMENTAIS E DIVIS

AO COM RESTO 33
Entao, aplicando o item (b) da Proposi c ao 2.3 temos que se 9 | S, entao
tambem dividira a N. Reciprocamente, suponhamos que 9 | N e observemos
que S = N 9k, com
k = a
1
11 1
. .
n1 vezes
+a
2
11 1
. .
n2 vezes
+ + a
n1
.
Entao, usando novamente o item (b) da Proposi c ao 2.3 temos que 9 | S.

E obvio que a prova para o caso da divisibilidade por 3 segue de maneira


identica, logo ca provado o item (a).
A prova do item (b) segue de maneira muito semelhante e deixamos a
mesma a cargo do leitor.
Exemplo 2.6. Prove sem fazer muitas contas que o n umero
N = 13424136 + 1234567890
e divisvel por 3.
Soluc ao. Note que nao precisamos fazer a soma dos n umeros anteriores.
Para mostrar isso, basta aplicar o item (ii) da Proposi c ao 2.3 e o item (i)
da Proposi c ao 2.5, observando que cada um dos n umeros acima e divisvel
por 3, pois a soma de seus dgitos e um m ultiplo de 3.
O proximo passo de nossa discussao e ver o que acontece quando um
n umero nao e divisvel por outro. Por exemplo, analisemos se 31 e divisvel
por 7; para isto listamos a diferen ca entre 31 e os m ultiplos positivos de 7,
ou seja,
r
1
= 31 7 1 = 24,
r
2
= 31 7 2 = 17,
r
3
= 31 7 3 = 10,
r
4
= 31 7 4 = 3,
r
5
= 31 7 5 = 4,
r
6
= 31 7 6 = 11,
.
.
.
LivroOlimpiadaOcial
2006/5/27
page 34
i
i
i
i
i
i
i
i
34 [CAP. 2: DIVISIBILIDADE
Claramente 31 nao e divisvel por 7, pois caso contrario teramos que alguma
das diferen cas acima seria igual a zero, o que e impossvel pois as diferen cas
r
q
:= 31 7q com 1 q 4 sao todas positivas e com q 5 sao todas
negativas. Entretanto, notamos que dentre as diferen cas positivas a unica
que e menor que 7 corresponde ao caso q = 4. O resultado seguinte nos
diz o que acontece no caso geral da divisao de um inteiro b por um inteiro
positivo a.
Teorema 2.7 (Algoritmo da Divisao). Dados dois inteiros a e b, com
a > 0, existem unicos inteiros q e r tais que
b = aq + r, 0 r < a.
Se a b, ent ao r satisfaz a desigualdade estrita 0 < r < a.
Demonstrac ao. Por simplicidade, suporemos que b e positivo. Se b < a,
basta tomar q = 0 e r = b. Se b = a, entao tomamos q = 1 e r = 0. Assim,
assumiremos tambem que b > a > 0. Consideremos o conjunto
R = {b aq Z; b aq 0} N {0} (2.8)
Notemos que o conjunto R e nao-vazio, pois b a R, ja que b a > 0.
Deste modo, pelo Princpio da Boa Ordena c ao temos que R admite um
menor elemento, que denotaremos por r. Claramente r = b aq, para
algum q 0; assim como r 0, pois foi escolhido com essa condi c ao. Alem
disso, r < a pois caso contrario
r = b aq a b a(q + 1) 0. (2.9)
Alias,
a > 0 b a(q + 1) < b aq. (2.10)
Das desigualdades (2.9) e (2.10) segue que
0 b a(q + 1) < b aq,
contradizendo o fato de que r = b aq e o menor elemento nao-negativo de
R.
Agora provaremos que de fato r e q escolhidos desta forma sao unicos.
Com efeito, suponhamos que existem outros inteiros r
1
e q
1
tais que
b = aq
1
+ r
1
, 0 r
1
< a.
LivroOlimpiadaOcial
2006/5/27
page 35
i
i
i
i
i
i
i
i
[SEC. 2.1: CONCEITOS FUNDAMENTAIS E DIVIS

AO COM RESTO 35
Entao resulta que aq + r = aq
1
+ r
1
. Logo,
(r r
1
) = (q
1
q)a; (2.11)
sendo assim, r r
1
e m ultiplo de a. Mas em virtude de a < r r
1
< a, o
unico valor que r r
1
pode tomar, sendo este m ultiplo de a, e r r
1
= 0.
Portanto, r = r
1
, de onde se deduz diretamente de (2.11) que q = q
1
.
Os n umeros q e r no enunciado do teorema acima sao chamados, respec-
tivamente, de quociente e resto da divisao de b por a.
Os exemplos a seguir apresentam a utilidade do Teorema 2.7.
Exemplo 2.8. Encontrar o ultimo dgito do n umero 2
2043
.
Soluc ao.

E claro que este n umero e muito grande e seria muito tedioso
multiplicar 2043 vezes o n umero 2 para descobrir o ultimo dgito de 2
2043
.
Entretanto, se calculamos as primeiras potencias vemos que existe regular-
idade nos resultados, como mostra a tabela a seguir.
2
1
= 2 2
2
= 4 2
3
= 8 2
4
= 16
2
5
= 32 2
6
= 64 2
7
= 128 2
8
= 256
2
9
= 512 2
10
= 1024 2
11
= 2048 2
12
= 4096
2
13
= 8192 2
14
= 16384 2
15
= 32768 2
16
= 65536
Agora observamos que:
na primeira coluna da tabela so aparecem potencias da forma 2
4k+1
,
com k = 0, 1, 2, 3, , e todas elas terminam em 2;
na segunda coluna da tabela so aparecem potencias da forma 2
4k+2
,
com k = 0, 1, 2, 3, , e todas elas terminam em 4;
na terceira coluna da tabela so aparecem potencias da forma 2
4k+3
,
com k = 0, 1, 2, 3, , e todas elas terminam em 8;
na quarta coluna da tabela so aparecem potencias da forma 2
4k
, com
k = 1, 2, 3, , e todas elas terminam em 6.
Portanto, so resta saber que resto deixa 2043 na divisao por 4. Como
2043 = 510 4 + 3, o n umero 2
2043
aparecera na terceira coluna da tabela,
logo o ultimo dgito deste e 8.
LivroOlimpiadaOcial
2006/5/27
page 36
i
i
i
i
i
i
i
i
36 [CAP. 2: DIVISIBILIDADE
O proximo exemplo, como veremos, motiva a procura de caminhos e-
cientes para encontrar o resto que deixa um n umero quando e dividido por
outro.
Exemplo 2.9. Um turista brasileiro chega a Cuba e troca parte de seu
dinheiro na casa de c ambio, recebendo 175 notas de 50 pesos e 213 notas
de 20 pesos. Ele decide trocar este dinheiro pela maior quantidade possvel
das famosas moedas de 3 pesos cubanos, devido `as mesmas terem gravadas
a imagem do guerrilheiro Che Guevara. Quanto sobrou do dinheiro depois
de fazer a troca pelas moedas?
Soluc ao. O problema e facil de resolver. Basta achar o resto que deixa o
n umero N = 17550+21320 quando e dividido por 3. Entretanto, queremos
destacar que nao e preciso fazer os produtos e a soma envolvidos no n umero
N. Em lugar de fazer isto substitumos cada n umero que aparece em N
pelo resto que este deixa na divisao por 3, formando assim um novo n umero
N
1
, ou seja,
N
1
= 1 2 + 0 2 = 2.
Agora procuramos o resto que N
1
deixa na divisao por 3, que obviamente
e 2. A surpresa e que este resto e o mesmo que deixa N na divisao por 3.
Logo, sobraram 2 pesos depois de fazer a troca.
A solu c ao do exemplo anterior e uma aplica c ao particular do seguinte
lema que e de muita utilidade na resolu c ao de problemas.
Lema 2.10 (Lema dos Restos). A soma e o produto de quaisquer dois
n umeros naturais deixa o mesmo resto que a soma e o produto dos seus
restos, na divis ao por um inteiro positivo a.
Demonstrac ao. Sejam N
1
, N
2
Z. Fazendo a divisao com resto de ambos
n umeros por a temos que
N
1
= aq
1
+ r
1
e N
2
= aq
2
+ r
2
,
com 0 r
1
, r
2
< a. Entao
N
1
N
2
= (aq
1
+ r
1
)(aq
2
+ r
2
)
= a
2
q
1
q
2
+ aq
1
r
2
+ aq
2
r
1
+ r
1
r
2
= a(aq
1
q
2
+ q
1
r
2
+ q
2
r
1
) + r
1
r
2
= aq + r
1
r
2
,
(2.12)
LivroOlimpiadaOcial
2006/5/27
page 37
i
i
i
i
i
i
i
i
[SEC. 2.1: CONCEITOS FUNDAMENTAIS E DIVIS

AO COM RESTO 37
onde q = aq
1
q
2
+ q
1
r
2
+ q
2
r
1
. Agora dividimos r
1
r
2
por a para obtermos
r
1
r
2
= ap + r, p Z, 0 r < a. (2.13)
Das igualdades (2.12) e (2.13) segue que
N
1
N
2
= aq + ap + r = a(p + q) + r, 0 r < a. (2.14)
Portanto, de (2.13) e (2.14) conclumos que os restos que deixam N
1
N
2
e
r
1
r
2
na divisao por a sao iguais, cando provado o resultado para o produto.
A prova para a soma e analoga.
A vantagem do lema e que em certos problemas que envolvem n umeros
muito grandes podemos substituir estes por n umeros muito menores e mais
confortaveis para trabalhar. Vejamos isto atraves dos seguintes exemplos.
Exemplo 2.11. Prove que o produto de dois n umeros naturais consecutivos
e sempre divisvel por 2.
Soluc ao. Se n N temos que provar que N = n(n + 1) e divisvel por 2.
Quando fazemos a divisao de n por 2 temos duas possibilidades para o resto:
r = 0 ou r = 1. Analisemos os dois casos por separado.
[r = 0] Neste caso o resto que deixa N na divisao por 2 e o mesmo
que o resto que deixa 0(0+1)=0, logo N e divisvel por 2.
[r = 1] Neste caso podemos substituir N por 1(1+1)=2 e o resto
que este ultimo deixa quando e dividido por 2 e 0, logo N tambem e
divisvel por 2.
Mostraremos agora como utilizar o exemplo anterior pra resolver um dos
problemas da 1
a
Olimpada Brasileira de Matematica.
Exemplo 2.12. Prove que se n e mpar, entao n
2
1 e m ultiplo de 8.
Soluc ao. Como n e mpar, podemos escrever n = 2k +1, para algum k Z.
Logo
n
2
1 = (2k + 1)
2
1 = 4k
2
+ 4k + 1 1 = 4k
2
+ 4k.
Assim,
n
2
1 = 4k(k + 1).
LivroOlimpiadaOcial
2006/5/27
page 38
i
i
i
i
i
i
i
i
38 [CAP. 2: DIVISIBILIDADE
Observe que de acordo com o exemplo 2.11, k(k + 1) e m ultiplo de 2. Por-
tanto, k(k + 1) = 2q para algum q Z, de aonde
n
2
1 = 4k(k + 1) = 4 2q = 8q,
como queramos demonstrar.
Exemplo 2.13. Prove que em qualquer tri angulo ret angulo com lados in-
teiros, pelo menos um deles e m ultiplo de 3.
Soluc ao. Comecemos analisando quais sao os restos possveis para a divisao
por 3 de um n umero que e quadrado. De acordo com o Lema dos Restos
temos a seguinte tabela para os restos de n e n
2
, na divisao por 3:
n n
2
0 0
1 1
2 1
Resumindo, se um n umero nao e m ultiplo de 3 entao o resto da divisao de
seu quadrado por 3 deve ser igual a 1.
Agora denotemos por a e b os catetos e por c a hipotenusa. Suponhamos
que nenhum deles e divisvel por 3. Entao a
2
e b
2
deixam resto 1 na divisao
por 3. Logo, a
2
+ b
2
deixa resto 1
2
+ 1
2
= 2 na divisao por 3; mas isto e
uma contradi c ao pois, pelo Teorema de Pitagoras, a
2
+ b
2
= c
2
e c
2
deixa
resto 1 quando e dividido por 3.
2.2 Maximo Divisor Comum e Mnimo M ultiplo
Comum
Nesta se c ao estudaremos dois conceitos fundamentais, que aparecem natu-
ralmente em varios problemas de divisibilidade, assim como a rela c ao exis-
tente entre eles.
O primeiro destes conceitos esta relacionado com os inteiros positivos
que dividem simultaneamente a dois inteiros pre-xados e e denominado
M aximo Divisor Comum.
Daqui por diante so consideraremos os divisores positivos dos n umeros.
LivroOlimpiadaOcial
2006/5/27
page 39
i
i
i
i
i
i
i
i
[SEC. 2.2: M

AXIMO DIVISOR COMUM E M

INIMO M

ULTIPLO COMUM 39
Deni cao 2.14 (Maximo Divisor Comum). Sejam a e b inteiros difer-
entes de zero. O maximo divisor comum, resumidamente m.d.c, entre a e b
e o n umero d que satisfaz as seguintes condi c oes:
(a) d e um divisor comum de a e b, isto e, d | a e d | b;
(b) d e o maior inteiro positivo com a propriedade (a).
Neste caso, denotamos o m.d.c entre a e b por d = m.d.c(a, b) ou por
d = (a, b). Se (a, b) = 1, entao dizemos que a e b sao primos entre si.
Exemplo 2.15. Observando que 12 = 62, 18 = 63 temos que m.d.c.(12, 18) =
6. Por outro lado, m.d.c.(4, 15) = 1, logo os n umeros 4 e 15 sao primos entre
si.
Vejamos agora algumas das propriedades mais importantes dos divisores
comuns de dois inteiros.
Proposi cao 2.16. Sejam a e b dois inteiros. Ent ao valem as seguintes
armac oes.
(a) Se a e m ultiplo de b, ent ao (a, b) = b;
(b) Se a = bq + c, c = 0, ent ao o conjunto dos divisores comuns dos
n umeros a e b coincide com o conjunto dos divisores comuns dos
n umeros b e c. Particularmente, (a, b) = (b, c).
Demonstrac ao. Come camos com a prova de (a). Com efeito, todo divisor
comum dos n umeros a e b e um divisor de b. Reciprocamente, usando que
a e m ultiplo de b, todo divisor de b e tambem um divisor de a, ou seja,
um divisor comum dos n umeros a e b. Portanto, o conjunto dos divisores
comuns dos n umeros a e b e igual ao conjunto dos divisores de b. Como o
maior divisor de b e ele mesmo, resulta que (a, b) = b.
Vejamos (b). Usando o item (b) da Proposi c ao 2.3 temos que todo
divisor comum de a e b tambem divide c e, conseq uentemente, e um divisor
de b e c. Pela mesma razao todo divisor comum de b e c tambem divide a
e, conseq uentemente, e um divisor de a e b. Portanto os divisores comuns
de a e b sao os mesmos que os divisores comuns de b e c. Particularmente,
tambem coincidem os maiores divisores comuns, ou seja, (a, b) = (b, c).
O teorema a seguir e uma das ferramentas basicas na resolu c ao de prob-
lemas que envolvem o m.d.c entre dois n umeros. O mesmo e devido ao
matematico frances

Etienne Bezout (1730-1783).
LivroOlimpiadaOcial
2006/5/27
page 40
i
i
i
i
i
i
i
i
40 [CAP. 2: DIVISIBILIDADE
Teorema 2.17 (Teorema de Bezout). Se d e o m.d.c de a e b, ent ao
existem n umeros inteiros x
0
e y
0
tais que d = (a, b) = ax
0
+ by
0
.
Demonstrac ao. Considere a combina c ao linear ax+by, onde x e y percorrem
todos os inteiros. Este conjunto de inteiros, denotado por
C = {ax + by; x, y Z},
inclui valores positivos e negativos. Alem disso, escolhendo x = y = 0,
vemos que C tambem contem o zero.
Pelo Princpio da Boa Ordena c ao, podemos escolher x
0
e y
0
tais que
= ax
0
+ by
0
seja o menor n umero inteiro positivo contido no conjunto C.
Agora mostraremos que | a e | b. Provaremos que | a e o outro
segue analogamente. Usaremos para este proposito o metodo de redu c ao ao
absurdo, ou seja, vamos supor que a e obteremos uma contradi c ao.
De a segue que existem inteiros q e r tais que a = q + r com
0 < r < (pelo Teorema 2.7). Portanto,
r = a q = a q(ax
0
+ by
0
) = a(1 qx
0
) + b(qy
0
)
e assim r esta no conjunto C, o que contradiz a hipotese de ser o menor
elemento positivo contido em C.
Uma vez que divide a e b so resta provar que = d. Com efeito, desde
que d = (a, b), podemos escrever a = da
1
, b = db
1
e
= ax
0
+ by
0
= d(a
1
x
0
+ b
1
y
0
).
Assim d | . Logo pela parte (c) da Proposi c ao 2.3, conclumos que d .
Agora d < e impossvel pois d = m.d.c(a, b), e portanto d = = ax
0
+
by
0
.
A seguinte proposi c ao resume algumas conseq uencias importantes da
demonstra c ao dada ao Teorema de Bezout.
Proposi cao 2.18. Sejam d, k N e a, b, c Z. Ent ao valem as seguintes
armac oes:
(a) Se d | a e d | b, ent ao d | (a, b);
(b) O m.d.c.(a, b) e o menor valor positivo de ax+by, onde x e y percorrem
todos os n umeros inteiros;
LivroOlimpiadaOcial
2006/5/27
page 41
i
i
i
i
i
i
i
i
[SEC. 2.2: M

AXIMO DIVISOR COMUM E M

INIMO M

ULTIPLO COMUM 41
(c) (ka, kb) = k(a, b);
(d) Se d | a e d | b, ent ao (
a
d
,
b
d
) =
1
d
(a, b). Conseq uentemente,
_
a
(a, b)
,
b
(a, b)
_
= 1;
(e) Se (a, c) = (b, c) = 1, ent ao (ab, c) = 1;
(f) Se c | ab e (b, c) = 1, ent ao c | a.
Demonstrac ao. A prova de (a) e conseq uencia imediata da igualdade (a, b) =
ax
0
+ by
0
anunciada no Teorema de Bezout; assim como (b) segue direta-
mente da demonstra c ao dada a este teorema.
Para provar (c), primeiro observamos que
(ka)x + (kb)y = k(ax + by) onde x, y Z.
Usando o item (a) e o fato de k ser positivo, da igualdade acima segue que
(ka, kb) = menor valor positivo de (ka)x + (kb)y, onde x, y Z
= k {menor valor positivo de ax + by, onde x, y Z}
= k(a, b).
A arma c ao feita em (d) segue diretamente de (c), observando que
(a, b) =
_
d
a
d
, d
b
d
_
= d
_
a
d
,
b
d
_
.
Continuamos com a prova de (e). De (a, c) = (b, c) = 1, temos que
existem inteiros x
j
e y
j
, j = 1, 2, tais que
ax
1
+ cy
1
= 1,
bx
2
+ cy
2
= 1.
Multiplicando lado-a-lado as igualdades obtemos
(x
1
x
2
)
. .
x
ab + (ax
1
y
2
+ y
1
bx
2
+ cy
1
y
2
)
. .
y
c = 1.
Entao, usando o item (a) e a igualdade acima resulta que (ab, c) = 1.
LivroOlimpiadaOcial
2006/5/27
page 42
i
i
i
i
i
i
i
i
42 [CAP. 2: DIVISIBILIDADE
Finalmente, provaremos (f). Das hipoteses temos que existem inteiros
x
0
e y
0
tais que
bx
0
+ cy
0
= 1.
Multiplicamos a igualdade acima por a em ambos lados para obtermos
abx
0
+ acy
0
= a.
Por outro lado, ab = cq para algum inteiro q. Usando esta condi c ao na
ultima igualdade temos que
cqx
0
+ acy
0
= c(qx
0
+ ay
0
) = a,
logo c | a.
Apesar de conhecermos propriedades teoricas do m.d.c entre dois in-
teiros, encontra-lo de fato pode ser uma tarefa complicada, sem auxlio das
ferramentas corretas. Lembrando o seu signicado, o leitor talvez pudesse
pensar que devemos calcular todos os divisores de a, todos os divisores de
b e descobrir qual e o maior elemento comum aos dois conjuntos.
Na pratica, para achar o m.d.c se faz uso de um importante metodo
denominado Algoritmo de Euclides.
Teorema 2.19 (Algoritmo de Euclides). Dados dois inteiros positivos,
a e b, aplicamos sucessivamente o algoritmo da divis ao para obter a seguinte
seq uencia de igualdades
_

_
b = aq
1
+ r
1
, 0 r
1
< a,
a = r
1
q
2
+ r
2
, 0 r
2
< r
1
,
r
1
= r
2
q
3
+ r
3
, 0 r
3
< r
2
,

r
n2
= r
n1
q
n
+ r
n
, 0 r
n
< r
n1
,
r
n1
= r
n
q
n+1
.
(2.15)
O m.d.c.(a, b) = r
n
, ou seja, e o ultimo resto n ao-nulo no processo de divis ao
anterior.
Demonstrac ao. Come camos observando que o processo de divisao (2.15)
e nito. Com efeito, a seq uencia de n umeros inteiros r
k
e estritamente
decrescente e esta contida no conjunto {r Z, 0 r < a}, portanto nao
LivroOlimpiadaOcial
2006/5/27
page 43
i
i
i
i
i
i
i
i
[SEC. 2.2: M

AXIMO DIVISOR COMUM E M

INIMO M

ULTIPLO COMUM 43
pode conter mais do que a inteiros positivos. Examinando as igualdades
(2.15) de cima para baixo e usando a Proposi c ao 2.16 temos que
(a, b) = (a, r
1
) = (r
1
, r
2
) = = (r
n1
, r
n
) = r
n
.
Observa cao 2.20. Notemos que o Teorema de Bezout tambem pode ser
obtido como conseq uencia do processo de divisao (2.15). Com efeito, pode-
mos escrever
r
n
= r
n2
r
n1
q
n
r
n1
= r
n3
r
n2
q
n1
_
r
n
= r
n2
(r
n3
r
n2
q
n1
)q
n
.
Logo, conseguimos escrever r
n
em termos de r
n2
e r
n3
. Utilizando a
express ao r
n2
= r
n4
r
n3
q
n2
podemos escrever r
n
como combina c ao de
r
n3
e r
n4
. Repetindo este processo varias vezes, concluimos que existem
x, y Z tais que
d = r
n
= xr
1
+ yr
2
.
Ora, como r
1
= baq
1
e r
2
= ar
1
q
2
= a(1+q
1
q
2
)bq
2
, entao, substituindo
estes valores na ultima igualdade obtemos o Teorema de Bezout.
Exemplo 2.21. Achar o maximo divisor comum dos n umeros 471 e 1176.
Soluc ao. Aplicando o algoritmo de Euclides obtemos a seguinte seq uencia
de divisoes com resto:
1176 = 471 2 + 234,
471 = 234 2 + 3,
234 = 78 3,
entao o m.d.c(471, 1176) = 3.
Exemplo 2.22. Provar que a fra c ao
2n+8
4n+15
e irredutvel para todo n umero
natural n.
Soluc ao. Usando o algoritmo de Euclides temos que
4n + 15 = (2n + 8) 1 + 2n + 7,
2n + 8 = (2n + 7) 1 + 1,
2n + 7 = (2n + 7) 1.
LivroOlimpiadaOcial
2006/5/27
page 44
i
i
i
i
i
i
i
i
44 [CAP. 2: DIVISIBILIDADE
Entao o m.d.c.(4n +15, 2n +8) = 1 e portanto 4n +15 e 2n +8 sao primos
entre si para qualquer valor de n.
Exemplo 2.23. Achar o m.d.c.(111 111
. .
100vezes
, 11 11
. .
60 vezes
)
Soluc ao. Primeiro escrevemos os n umeros na base decimal, isto e,
111 111
. .
100 vezes
= 10
99
+ 10
98
+ + 1,
11 11
. .
60 vezes
= 10
59
+ 10
58
+ + 1.
Aplicamos agora o algoritmo de Euclides para obter as seguintes igualdades
111 111
. .
100 vezes
= (10
59
+ 10
58
+ + 1)10
40
+ 10
39
+ 10
38
+ + 1,
10
59
+ 10
58
+ + 1 = (10
39
+ 10
38
+ + 1)10
20
+
+ 10
19
+ 10
18
+ + 1,
10
39
+ 10
38
+ + 1 = (10
19
+ 10
18
+ + 1)10
20
+
+ 10
19
+ 10
18
+ + 1.
Disto resulta que
m.d.c.(111 111
. .
100 vezes
, 11 11
. .
60 vezes
) = 10
19
+ 10
18
+ + 1 = 11 11
. .
20 vezes
.
Agora passamos ao segundo conceito importante desta se c ao. O mesmo
esta relacionado com os inteiros positivos que sao simultaneamente m ultiplos
de dois inteiros pre-xados e e denominado Mnimo M ultiplo Comum.
Deni cao 2.24 (Mnimo M ultiplo Comum). Sejam a e b inteiros difer-
entes de zero. O mnimo m ultiplo comum, resumidamente m.m.c, entre a e
b e o inteiro positivo m que satisfaz as seguintes condi c oes:
(a) m e um m ultiplo comum de a e b, isto e, a | m e b | m;
(b) m e o menor inteiro positivo com a propriedade (a).
LivroOlimpiadaOcial
2006/5/27
page 45
i
i
i
i
i
i
i
i
[SEC. 2.2: M

AXIMO DIVISOR COMUM E M

INIMO M

ULTIPLO COMUM 45
Neste caso, denotamos o m.m.c entre a e b por m = m.m.c(a, b) ou por
m = [a, b].
Resumimos a seguir algumas das propriedades fundamentais do m.m.c
de dois inteiros.
Proposi cao 2.25. Sejam a, b, c Z e k Z. Ent ao valem as seguintes
armac oes:
(a) Se c e m ultiplo comum de a e b, ent ao [a, b] | c;
(b) [ka, kb] = k[a, b];
(c) |ab| = [a, b] (a, b).
Demonstrac ao. Come camos com a prova de (a). A divisao com resto de c
por [a, b] nos da
c = [a, b]q + r, 0 r < [a, b]. (2.16)
Da igualdade acima, basta provar que r = 0 para obter o resultado desejado.
Suponhamos, pelo contrario, que 0 < r < [a, b]. Notemos que tanto a como
b dividem c e [a, b]. Logo, pelo item (b) da Proposi c ao 2.3 e a igualdade
(2.16), temos que a e b tambem dividem r, ou seja, r e m ultiplo comum de
a e b e nao pode ser menor que [a, b], contradizendo nossa suposi c ao.
Prosseguimos com a prova de (b). Observemos que k[a, b] e m ultiplo
comum de ka e kb, logo pelo item (i) vale que
[ka, kb] k[a, b]. (2.17)
Por outro lado, [ka, kb] = q
1
ka = q
2
kb, para alguns inteiros q
1
e q
2
; logo,
[ka,kb]
k
e um m ultiplo comum de a e b. Portanto,
[a, b]
[ka, kb]
k
k[a, b] [ka, kb]. (2.18)
Das igualdades (2.17) e (2.18) segue que
k[a, b] [ka, kb] k[a, b],
de onde vem diretamente o resultado.
Para provar (c) podemos supor sem perda de generalidade que a e b sao
positivos devido `as igualdades
[a, b] = [a, b] = [a, b] = [a, b].
LivroOlimpiadaOcial
2006/5/27
page 46
i
i
i
i
i
i
i
i
46 [CAP. 2: DIVISIBILIDADE
Dividiremos a prova em dois casos:
Caso 1: (a, b) = 1.
Sabemos que b | [a, b] e [a, b] = qa, para algum q N. Entao b | qa e
alem disso (a, b) = 1. Logo, pelo item (v) da Proposi c ao 2.18 temos que
b | q. Portanto, b q e conseq uentemente
ab aq = [a, b]. (2.19)
Entretanto, da deni c ao de [a, b] vale que
[a, b] ab. (2.20)
Das desigualdades (2.19) e (2.20) segue que ab [a, b] ab. Assim, ab =
[a, b] = [a, b] 1 = [a, b] (a, b).
Caso 2: (a, b) > 1.
Da parte (c) da Proposi c ao 2.18 sabemos que
_
a
(a,b)
,
b
(a,b)
_
= 1. Apli-
cando o caso anterior vale que
a
(a, b)

b
(a, b)
=
_
a
(a, b)
,
b
(a, b)
_

_
a
(a, b)
,
b
(a, b)
_
.
Multiplicamos esta ultima igualdade por (a, b)
2
e usamos o item (b) provado
anteriormente, assim como a parte (d) da Proposi c ao 2.18 para obter
ab = (a, b)
_
a
(a, b)
,
b
(a, b)
_
(a, b)
_
a
(a, b)
,
b
(a, b)
_
= [a, b] (a, b).
Exemplo 2.26. Dois amigos passeiam de bicicleta, na mesma dire c ao, em
torno a uma pista circular. Para dar uma volta completa um deles demora
15 minutos e o outro demora 18 minutos. Eles partem juntos e combinam
interromper o passeio quando os dois se encontrarem pela primeira vez no
ponto de partida. Quantas voltas deu cada um?
Soluc ao. Denotemos por N
1
e N
2
, respectivamente, o n umero de voltas que
da cada um dos amigos. Notemos que o tempo total da corrida e o menor
valor positivo de T que satisfaz as igualdades
T = 15N
1
= 18N
2
,
LivroOlimpiadaOcial
2006/5/27
page 47
i
i
i
i
i
i
i
i
[SEC. 2.2: M

AXIMO DIVISOR COMUM E M

INIMO M

ULTIPLO COMUM 47
ou seja
T = [15, 18] =
15 18
3
= 90.
Portanto, N
1
= 6 e N
2
= 5.
Finalizamos esta se c ao com um exemplo que nos fornece uma bela in-
terpreta c ao geometrica do mnimo m ultiplo comum. O mesmo foi proposto
na Olimpada Brasileira de Matematica.
Exemplo 2.27. Um ret angulo de lados inteiros AB = m e CD = n, e
dividido em quadrados de lado 1. Em cada um dos vertices ele possui um
pequeno orifco. Um raio de luz entra no ret angulo por um dos vertices, na
dire c ao da bissetriz do angulo reto, e e reetido sucessivamente nos lados
do ret angulo. Quantos quadrados sao atravessados pelo raio de luz?
Soluc ao. Se zermos alguns testes preliminares dando valores a m e n, ver-
emos que em cada caso a resposta coincidira com o m.m.c.(m,n). Provemos
que isto de fato vale para m e n quaisquer. Para realizar a prova nos aux-
iliaremos da seguinte gura
A B
C D
Figura 2.1::
Primeiramente, notemos que cada vez que o raio de luz atravessa um
quadrado ele avan ca uma unidade tanto na dire c ao horizontal como na
dire c ao vertical. Usando este fato fazemos as seguintes observa c oes
Se o raio entra pelo vertice A, tera que atravessar m quadrados ate
chegar ao lado BC, imediatamente mais m para chegar ao lado AD,
LivroOlimpiadaOcial
2006/5/27
page 48
i
i
i
i
i
i
i
i
48 [CAP. 2: DIVISIBILIDADE
depois mais m para chegar novamente ao lado BC, e assim sucessi-
vamente. Alem disto, depois do raio percorrer pm quadrados, com
p N, estara batendo no lado BC ou no lado AD.
Analogamente o raio batera no lado AB ou no lado DC se, e somente
se, atravessar qn quadrados, com q N.
Somente nos vertices B, C e D do ret angulo pode acontecer que o raio
incidente saia do ret angulo, terminando assim o processo de reexao.
Usando as observa c oes acima e facil ver que o raio chegara a um vertice
quando chegar simultaneamente a dois lados perpendiculares do ret angulo.
Portanto, deve ter atravessado um n umero x de quadrados tal que x =
pm = qn, ou seja, x devera ser um m ultiplo comum de m e n.

E claro que a
primeira vez que o raio chega a um vertice o n umero x e o menor m ultiplo
comum de m e n, isto e, x = [m, n].
Finalmente, observamos que nenhum dos quadrados e atravessado duas
vezes no percurso do raio de A ate bater no primeiro vertice, pois como
vemos na gura numa das dire c oes os quadrados atravessados serao todos
cinzas e na outra dire c ao, serao todos brancos.
2.3 N umeros Primos e Compostos
Ao longo da Historia da matematica, os n umeros primos foram protago-
nistas de celebres problemas que motivaram o desenvolvimento de teorias
e tecnicas pelas mentes mais ferteis, como Fermat, Euler e Gauss. Ate
hoje muitos destes problemas, simples de enunciar, que envolvem n umeros
primos sao desaos intelectuais para toda a humanidade.
Esta se c ao sera dedicada ao estudo de propriedades basicas dos n umeros
primos.
O n umero 1 tem um unico divisor, precisamente ele mesmo. Isto faz
dele um n umero bem peculiar dentro dos n umeros naturais. Todo n umero
natural N maior do que 1 tem pelo menos 2 divisores, claramente 1 e N.
Isto motiva a seguinte deni c ao.
Deni cao 2.28 (N umeros Primos e Compostos). Um inteiro positivo
N e dito primo se os unicos divisores que ele tem sao 1 e ele proprio; caso
contrario, e dito composto.
LivroOlimpiadaOcial
2006/5/27
page 49
i
i
i
i
i
i
i
i
[SEC. 2.3: N

UMEROS PRIMOS E COMPOSTOS 49


Exemplo 2.29. Os n umeros 1, 3, 5, 7, e 11 sao primos e os n umeros 10, 15, 35,
e 348 sao compostos.
Proposi cao 2.30. Seja N > 1 um n umero inteiro. Ent ao
(a) O menor divisor de N diferente de 1 e um n umero primo;
(b) Se N e composto, o seu menor divisor diferente de 1 n ao e maior que

N;
Demonstrac ao. Come camos provando (a). Seja p o menor divisor de N,
diferente de 1. Se p fosse composto teria algum divisor q tal que 1 < q < p;
mas
q | p e p | N,
o que nos diz que q | N, e isto contradiz a hipotese levantada sobre p.
Para provar (b) denotamos por p o menor divisor de N, diferente de 1.
Portanto, N = pq com q p. Multiplicando ambos lados da desigualdade
por p obtemos
N = pq p
2
,
e conseq uentemente vale

N p.
Agora vamos enunciar um dos resultados mais classicos da matematica,
que garante a existencia de innitos n umeros primos. Ate onde se conhece,
a demonstra c ao a seguir foi a primeira demonstra c ao escrita utilizando o
metodo de redu c ao ao absurdo, e e devida a Euclides (300 a.C.).
Teorema 2.31 (Teorema de Euclides). A quantidade de n umeros pri-
mos e innita.
Demonstrac ao. Faremos a prova por redu c ao ao absurdo. Suponha que
existe uma quantidade nita de n umeros primos e denotemos estes por
p
1
, p
2
, p
3
, . . . , p
k
.
Consideremos o n umero
N = p
1
p
2
p
3
p
k
+ 1
e chamemos de q o seu menor divisor primo. Obviamente q nao coincide
com nenhum dos n umeros p
i
, 1 i k, pois caso contrario, como ele divide
N, teria que dividir 1, o que e impossvel. Logo, temos uma contradi c ao ` a
hipotese de termos uma quantidade nita de primos.
LivroOlimpiadaOcial
2006/5/27
page 50
i
i
i
i
i
i
i
i
50 [CAP. 2: DIVISIBILIDADE
Os n umeros primos tambem podem ser caracterizados da seguinte maneira:
Proposi cao 2.32. Um inteiro positivo p e primo se, e somente se, satisfaz
a seguinte propriedade:
p | ab = p | a ou p | b (2.21)
onde a, b Z.
Demonstrac ao. Primeiramente, suponhamos que p e primo e que p b, logo
(p, b) = 1. Entao, pelo item (f) da Proposi c ao 2.18 temos que p | a.
Reciprocamente, suponhamos que, a propriedade 2.21 e valida e alem
disto vamos supor, pelo absurdo, que p nao e primo. Entao,
p = d
1
d
2
, com 1 < d
1
< p, 1 < d
2
< p. (2.22)
De (2.21) segue que p | d
1
ou p | d
2
; conseq uentemente
p d
1
, ou p d
2
, (2.23)
contradizendo isto o armado em (2.22).
2.3.1 O Crivo de Eratostenes
Apesar de sabermos que existem innitos n umeros primos, determinar se
um n umero e primo nem sempre e uma tarefa rapida. Uma das princi-
pais ferramentas utilizadas para esse m e um metodo simples, denominado
Crivo de Erat ostenes, que nos permite achar todos os n umeros primos que
nao sao maiores que um inteiro N dado. O metodo consiste nos seguintes
passos: Escrevemos os n umeros de forma ordenada a partir de 2, isto e,
2, 3, 4, 5, 6, 7, 8, 9, 10, 11, 12, 13, 14, 15, 16, 17, . . . , N (2.24)
Observamos que o primeiro primo que aparece em (2.24) e 2 e ime-
diatamente apagamos da lista (2.24) todos os m ultiplos de 2 maiores
que ele, por serem compostos; resta assim a seguinte lista
2, 3, 5, 7, 9, 11, 13, 15, 17 . . .
O primeiro n umero nao apagado que aparece na lista restante e 3, que
tambem e primo. Imediatamente apagamos da lista todos os m ultiplos
de 3 maiores que ele, por serem compostos; resta agora a lista
2, 3, 5, 7, 11, 13, 17, . . .
LivroOlimpiadaOcial
2006/5/27
page 51
i
i
i
i
i
i
i
i
[SEC. 2.3: N

UMEROS PRIMOS E COMPOSTOS 51


O primeiro n umero nao apagado que aparece na lista que restou do
passo anterior e 5, que tambem e primo. Imediatamente apagamos da
lista todos os m ultiplos de 5 maiores que ele, por serem compostos.
Se repetirmos este processo ate o maior n umero menor que

N, os
n umeros que sobram sao exatamente os n umeros primos.
Por exemplo, se N = 40, temos que

40 = 6, 324555 . Entao, aplicando


o metodo:
2 3 4 5 6 7 8 9 10
11 12 13 14 15 16 17 18 19 20
21 22 23 24 25 26 27 28 29 30
31 32 33 34 35 36 37 38 39 40
Passo 1: Ordenamos os n umeros
2 3 5 7 9
11 13 15 17 19
21 23 25 27 29
31 33 35 37 39
Passo 2: Tiramos os m ultiplos de 2
2 3 5 7
11 13 17 19
23 25 29
31 35 37
Passo 3: Tiramos os m ultiplos de 3
2 3 5 7
11 13 17 19
23 29
31 37
Passo 4: Tiramos os m ultiplos de 5
Como 7
2
= 49 > 40, paramos agora.
LivroOlimpiadaOcial
2006/5/27
page 52
i
i
i
i
i
i
i
i
52 [CAP. 2: DIVISIBILIDADE
Note que ao come car a apagar os m ultiplos de um n umero primo p
podemos come car a apagar a partir de p
2
, pois se supomos que existe um
n umero composto N
1
nao apagado menor que p
2
, temos que N
1
= p
1
q
1
,
sendo p
1
seu menor divisor primo. Entao, pelo item (b) da Proposi c ao 2.30,
p
1
<

N
1
< p, logo N
1
deveria ter sido apagado pois e m ultiplo de um
primo menor que p.
2 3 5 7 11 13 17 19 23 29
31 37 41 43 47 53 59 61 67 71
73 79 83 89 97 101 103 107 109 113
127 131 137 139 149 151 157 163 167 173
179 181 191 193 197 199 211 223 227 229
233 239 241 251 257 263 269 271 277 281
283 293 307 311 313 317 331 337 347 349
353 359 367 373 379 383 389 397 401 409
419 421 431 433 439 443 449 457 461 463
467 479 487 491 499 503 509 521 523 541
Tabela 2.1: Os primeiros 100 n umeros primos
Os n umeros primos alem de belos e desaadores do ponto de vista
matematico, sao extremamente importantes para as atividades usuais de
nosso dia-a-dia. Por exemplo, nenhuma transa c ao bancaria ou pela inter-
net estaria segura sem o uso de n umeros primos muito grandes. Assim,
surge naturalmente a pergunta de como podemos produz-los em grandes
quantidades. Essa pergunta sempre intrigou os matematicos e continua sem
solu c ao ate os dias atuais. Apesar deles serem abundantes, em quantidade
innita de acordo com o Teorema 2.31, nao existe nenhum metodo razoavel
de produ c ao de n umeros primos, mesmo tendo em m aos a alta tecnologia
de hoje em dia. Porem, ao longo do tempo algumas formulas se mostraram
uteis para a descoberta de n umeros primos. Entre estas formulas, destaca-se
o que chamamos de primos de Mersenne.
Marin Mersenne (1588-1648) foi um monge frances que nasceu na cidade
de Maine e foi um dos grandes inuenciadores da matematica francesa nos
seculos XVI e XVII. Apaixonado pelos n umeros, teve entre seus correspon-
dentes Descartes, Fermat, Pascal e Galileu. Entre suas varias descobertas,
LivroOlimpiadaOcial
2006/5/27
page 53
i
i
i
i
i
i
i
i
[SEC. 2.3: N

UMEROS PRIMOS E COMPOSTOS 53


ele estudou os n umeros da forma:
M
n
= 2
n
1
Observe que vale o seguinte fato a respeito desses n umeros:
Proposi cao 2.33. Se M
n
e primo, ent ao n e primo.
Demonstrac ao. Provar essa proposi c ao equivale a mostrar que a sua forma
contra-recproca vale. Ou seja, que se n e composto, digamos n = a.b, com
a b > 1, entao M
n
tambem e composto. De fato, podemos decompo-lo
do seguinte modo:
M
a.b
= 2
ab
1 =
_
2
a(b1)
2
a(b2)
+ + 2
a
+ 1
__
2
b
1
_
.
Porem, nao e verdade a recproca da arma c ao acima. Por exemplo,
Hudalricus Regius mostrou em 1536 que M
11
= 2
11
1 = 2047 nao e primo,
ja que 2047 = 23 89.
Em 1643, Mersenne armou que para
n = 2, 3, 5, 7, 13, 17, 19, 31, 67, 127 e 257,
os valores de M
n
sao todos primos e para todos os outros valores de n
menores que 257, M
n
e composto.
Hoje sabemos que Mersenne errou na sua arma c ao, esquecendo tres
valores de n onde M
n
e primo: 61, 89 e 107 e incluindo M
67
e M
257
como
n umeros primos. Esses fatos so foram mostrados em 1947.
O maior n umero primo conhecido ate 15 de Dezembro de 2005 e M
30402457
que possui 9.152.052 de dgitos! Para descobrir esse n umero, o grupo de
pesquisa liderado por Cooper, Boone, Woltman e Kurowski (GIMPS) pre-
cisou do auxlio de mais de 700 computadores! Para mais informa c oes veja
a pagina: www.mersenne.org.
2.3.2 O Teorema Fundamental da Aritmetica
Os n umeros primos sao as celulas dos n umeros naturais, no sentido de que
qualquer n umero natural e produto de n umeros primos. Por exemplo,
560 = 56 10 = 7 8 5 2 = 7 2 2 2 5 2,
LivroOlimpiadaOcial
2006/5/27
page 54
i
i
i
i
i
i
i
i
54 [CAP. 2: DIVISIBILIDADE
onde cada um dos fatores que aparecem no produto sao n umeros primos.
Perguntamo-nos, o que acontece se come camos com uma outra fatora c ao
inicial de 560, por exemplo, 560 = 28 20. Vejamos:
560 = 28 20 = 14 2 10 2 = 7 2 2 5 2 2.
Surpreendentemente chegamos `a mesma representa c ao anterior, salvo a or-
dem dos fatores.
2
2
2
2
7
5
Figura 2.2: O n umero 560 e composto de 4 celulas do tipo 2 , uma celula do tipo
7 e uma celula do tipo 5.
O fato observado acima vale para qualquer n umero natural maior que
1. Especicamente, temos o seguinte resultado conhecido como Teorema
Fundamental da Aritmetica.
Teorema 2.34 (Teorema Fundamental da Aritmetica). Todo n umero
natural N maior que 1 pode ser escrito como um produto
N = p
1
1
p
2
2
p
3
3
p
m
m
, (2.25)
onde m 1 e um n umero natural,
i
N e p
i
e primo para todo 1 i m
. Alem disso, a fatorac ao em (2.25) e unica se exigirmos que p
1
< p
2
<
< p
m
.
Demonstrac ao. Seja N um inteiro maior que 1. Denotando por p
1
seu menor
divisor primo tem-se que
N = p
1

1
, 1
1
< N.
Se
1
= 1, entao N
1
= p
1
e a fatora c ao desejada e obtida. Caso contrario,
denotando por p
2
o menor divisor primo de
1
tem-se que
N = p
1
p
2

2
, 1
2
<
1
.
LivroOlimpiadaOcial
2006/5/27
page 55
i
i
i
i
i
i
i
i
[SEC. 2.3: N

UMEROS PRIMOS E COMPOSTOS 55


Se
2
= 1, entao N = p
1
p
2
e novamente chegamos `a fatora c ao desejada.
Caso contrario, denotando por p
3
o menor divisor primo de
2
tem-se que
N = p
1
p
2
p
3

3
, 1
3
<
2
.
Continuando este processo sucessivamente obtemos entao uma seq uencia
estritamente decrescente de n umeros naturais
n
, ou seja,
N >
1
>
2
>
2
> >
n
>
n+1
> 1,
Entao, pelo princpio da boa ordem, so pode existir uma quantidade nita
de indices n tais que
n
> 1 e conseq uentemente
n+1
= 1, de onde segue
que
N = p
1
p
2
p
n
.
Notemos que na representa c ao acima os p
i
podem-se repetir, resultando
nalmente a representa c ao desejada em (2.25).
Provaremos agora a unicidade de tal fatora c ao. Com efeito, suponha
que existem duas fatora c oes:
p
1
1
p
2
2
p
3
3
p
m
m
= N = q
1
1
q
2
2
q
3
3
q
s
s
Pela Proposi c ao 2.32 temos que cada p
i
divide algum q
j
, logo p
i
= q
j
,
por serem primos. Portanto, cada p
i
aparece no lado direito da igualdade
acima, e, um argumento analogo nos da que cada q
j
tambem aparece no
lado esquerdo da igualdade. Entao, como os p
is
e os q
j
s
sao diferentes dois
a dois e organizados crescentemente, temos m = s e a igualdade se reduz a
p
1
1
p
2
2
p
3
3
p
m
m
= p
1
1
p
2
2
p
3
3
p
m
m
.
Suponhamos agora que
1
seja diferente de
1
; sem perda de generali-
dade vamos supor que
1
<
1
. Portanto,
p
2
2
p
3
3
p
m
m
= p
11
1
p
2
2
p
3
3
p
m
m
,
e como
1

1
> 0 entao, pela Proposi c ao 2.32 temos que p
1
divide algum
p
j
, com j > 1, o que e impossvel. Portanto,
1
=
1
. Similarmente
provamos que
i
=
i
, com i = 1, , n.
Observa cao 2.35. O Teorema Fundamental da Aritmetica foi enunciado
precisamente por Gauss (1777-1855). Seus antecessores, Fermat, Euler, La-
grange e Legendre, utilizavam este teorema sem a preocupa c ao de te-lo
enunciado ou demonstrado com precissao.
LivroOlimpiadaOcial
2006/5/27
page 56
i
i
i
i
i
i
i
i
56 [CAP. 2: DIVISIBILIDADE
Uma prova alternativa deste teorema sera apresentada no Captulo 5,
usando o metodo de indu c ao.
Exemplo 2.36. Prove que um n umero N e par se, e somente se, o n umero
2 aparece na fatora c ao de N em fatores primos.
Soluc ao. Obviamente, se 2 aparece na fatora c ao em primos de N, entao N
e par. Ora, se N e par temos que N = 2q. Por outro lado q e N se fatoram,
respeitivamente, como
q = q
1
1
q
2
2
q
m
m
e N = p
1
1
p
2
2
p
s
s
.
Logo,
2 q
1
1
q
2
2
q
m
m
= p
1
1
p
2
2
p
s
s
.
Pela unicidade da fatora c ao, para algum i, com 1 i s, o correspondente
p
i
deve ser igual a 2. Portanto, 2 aparece na fatora c ao de n.
Exemplo 2.37. Seja A = {1, 2, 3, 4, 5, 6, 7}.

E possvel decompor o con-
junto A em dois subconjuntos disjuntos tais que o produto dos elementos
de um seja igual ao produto dos elementos do outro?
Soluc ao. Mostraremos que e impossvel fazer esta decomposi c ao. Com
efeito, suponha que existem tais conjuntos, A
1
= {p
1
, p
2
, , p
r
} e A
2
=
{q
1
, q
2
, , q
s
}. Entao
p
1
p
2
p
r
. .

= q
1
q
2
q
s
. .

e alem disso, como os conjuntos A


1
A
2
sao disjuntos, temos que o n umero 5
aparece no produto ou no produto , mas nao em ambos simultaneamente.
Por outro lado, o Teorema 2.34 nos diz que a fatora c ao em primos de e
igual `a fatora c ao em primos de , logo o n umero 5 deveria aparecer tanto no
produto como no produto , contradizendo isto o fato anterior. Portanto
nao existe uma decomposi c ao com as condi c oes exigidas.
Exemplo 2.38. Encontre todos os n umeros inteiros e positivos n com a
propriedade de que o conjunto
A = {n, n + 1, n + 2, n + 3, n + 4, n + 5}
pode ser particionado em dois subconjuntos tais que o produto dos elementos
de um dos subconjuntos seja igual ao produto dos elementos do outro.
LivroOlimpiadaOcial
2006/5/27
page 57
i
i
i
i
i
i
i
i
[SEC. 2.3: N

UMEROS PRIMOS E COMPOSTOS 57


Demonstrac ao. Digamos que seja possvel essa decomposi c ao para algum n
e vamos denotar os conjuntos que obtemos com a decomposi c ao por A
1
e
A
2
. Observando a decomposi c ao dos elementos dos subconjuntos em fatores
primos, temos que todo fator primo de A
1
tambem devera pertencer a A
2
.
No conjunto dos seis n umeros so podemos ter um m ultiplo de 7, por isso
nao podemos tomar n como m ultiplo deste primo. Analogamente para
primos maiores que 7. Analisando o primo 5, conclumos que n e n + 5 sao
m ultiplos de 5, pois se nao, cairamos na analise anterior. Assim, os n umeros
n +1, n +2, n +3 e n +4 sao da forma 2

. Como entre eles existem dois


mpares, logo teremos duas potencias de 3 cuja diferen ca e 2, um absurdo.
Assim, nao existe n que satisfaz as condi c oes do enunciado.
Finalizamos esta se c ao com um exemplo que mostra como podemos com-
binar os fatos estudados para resolver problemas mais difceis
Exemplo 2.39. Encontre todos os n umeros que sao formados por 4 algar-
ismos da forma aabb e que sejam quadrados perfeitos.
Soluc ao. Como o n umero aabb e um quadrado perfeito, signica que:
N
2
=aabb ;
N
2
=10
3
a + 10
2
a + 10b + b =
_
10
3
+ 10
2
_
a + (10 + 1) b ;
N
2
=1100 a + 11 b ;
N
2
=11
_
100a + b
_
= 11
_
99a + a + b
_
;
Como 11 e primo e facil ver, usando a Proposi c ao 2.32, que 11
2
| N
2
.
Segue-se entao que 11 | (99a + a + b). Portanto, 11 | (a + b). Como
aabb tem 4 algarismos, segue-se que a = 0; portanto a {1, 2, 3, , 9} e
b {0, 1, 2, , 9}. De onde a + b 18. Logo, necessariamente devemos
ter a + b = 11. Podemos observar que a = 1, pois se a = 1 entao b = 10.
Analogamente, b = 0, 1. Portanto,
a {2, 3, 4, , 9} e b {2, 3, 4, , 9}.
Como em todo n umero quadrado perfeito o algarismo das unidades somente
pode acabar em 0, 1, 4, 5, 6 e 9. Segue-se que
b {4, 5, 6, 9}.
LivroOlimpiadaOcial
2006/5/27
page 58
i
i
i
i
i
i
i
i
58 [CAP. 2: DIVISIBILIDADE
Certamente b = 5, pois todo n umero que acaba em 5 quando e elevado ao
quadrado sempre acaba em 25. Assim,
b {4, 6, 9}.
Se b = 4, entao a = 7. Neste caso o n umero seria 7744 que e um
quadrado perfeito;
Se b = 6, entao a = 5. Neste caso o n umero seria 5566 que nao e um
quadrado perfeito;
Se b = 9, entao a = 2. Neste caso o n umero seria 2299 que nao e um
quadrado perfeito.
Finalmente, a unica solu c ao possvel e aabb = 7744 = 88
2
.
2.3.3 Identicando N umeros Compostos
Nao existe uma regra geral que nos permita determinar imediatamente
se um n umero inteiro e primo ou composto. Entretanto, existem alguns
criterios, baseados fundamentalmente em identidades algebricas, que nos
ajudam nessa difcil tarefa.
Criterio 1. Todo n umero que e diferenca de quadrados de dois inteiros
positivos, com diferenca distinta de um, e composto.
Demonstrac ao. Com efeito, se N = a
2
b
2
, entao N = a
2
b
2
= (ab)(a+
b). Logo, como a b = 1, temos que N e composto.
Exemplo 2.40. Prove que o n umero N = 2
20
25
4
e composto.
Soluc ao. Escrevemos N de outra forma, com o objetivo de facilitar nosso
trabalho. Com efeito, observemos que
N = (2
10
)
2
(25
2
)
2
= 1024
2
625
2
,
logo e composto por ser diferen ca de quadrados. Alem disso,
N = 1024
2
625
2
,
= (1024 625)(1024 + 625),
= 399 1649,
= 3 133 1649.
(2.26)
LivroOlimpiadaOcial
2006/5/27
page 59
i
i
i
i
i
i
i
i
[SEC. 2.3: N

UMEROS PRIMOS E COMPOSTOS 59


Portanto, podemos concluir que 3 | N.
Notemos que neste exemplo nao e muito pratico aplicar o criterio de
divisibilidade, estudado na Proposi c ao 2.5, para ver se o mesmo e divisvel
por 3.
O segundo criterio e uma generaliza c ao do primeiro.
Criterio 2. Sejam a, b inteiros positivos, com a b 2, e n N, com
n 2. Ent ao a b divide a
n
b
n
e, portanto, a
n
b
n
e composto.
Demonstrac ao. Os casos n = 2 e n = 3 seguem das classicas fatora c oes:
a
2
b
2
= (a b)(a + b), [diferen ca de quadrados]
a
3
b
3
= (a b)(a
2
+ ab + b
2
), [diferen ca de cubos].
Estas igualdades sugerem a seguinte fatora c ao para o caso geral
a
n
b
n
= (a b)
_
a
n1
+ a
n2
b + a
n3
b
2
+ + ab
n2
+ b
n1
_
,
de onde segue diretamente o resultado desejado. Para provar esta igualdade,
denotamos por o lado direito, ou seja,
= (a b)
_
a
n1
+ a
n2
b + a
n3
b
2
+ + ab
n2
+ b
n1
_
e desenvolvemos o mesmo para obtermos
= a
n
+
_
a
n1
b + a
n2
b
2
+ + a
2
b
n2
+ ab
n1
_
b
n

_
a
n1
b + a
n2
b
2
+ + a
2
b
n2
+ ab
n1
_
= a
n
b
n
,
terminando assim a prova.
Exemplo 2.41. Prove que N = (999994)
1234567890
1 e divisvel por
333331.
Soluc ao. Escolhendo a = 999994, b = 1 e n = 1234567890 pelo criterio 2
temos que
a 1 = 999993 | N. (2.27)
LivroOlimpiadaOcial
2006/5/27
page 60
i
i
i
i
i
i
i
i
60 [CAP. 2: DIVISIBILIDADE
Alem disso, 999993 = 3 333331; portanto
333331 | 999993. (2.28)
Entao, usando o item (a) da Proposi c ao 2.3 e as arma c oes feitas em (2.27)
e (2.28), temos que 333331 | N.
Outro criterio muito valioso e a Identidade de Sophie Germain, devido `a
matematica francesa Sophie Germain(1776-1831), que enunciamos a seguir.
Criterio 3. (Identidade de Sophie Germain). Dados a, b R, vale a
igualdade
a
4
+ 4b
4
= (a
2
+ 2b
2
+ 2ab)(a
2
+ 2b
2
2ab).
Demonstrac ao. A prova segue das seguintes igualdades:
a
4
+ 4b
4
= a
4
+ 4a
2
b
2
+ 4b
4
4a
2
b
2
= (a
2
+ 2b
2
)
2
4a
2
b
2
= (a
2
+ 2b
2
+ 2ab)(a
2
+ 2b
2
2ab).
Como aplica c ao desta identidade vejamos os seguintes exemplos.
Exemplo 2.42. Mostre que para qualquer inteiro positivo n > 1 o n umero
q
n
= n
4
+ 4
n
nunca e primo.
Soluc ao. O conjunto dos n umeros naturais e particionado em duas classes
disjuntas: o conjunto dos n umeros pares e o conjunto dos n umeros mpares.
Estudaremos cada classe por separado. Assim,
se n e um n umero par, entao n = 2k para algum inteiro positivo k 1.
Deste modo,
n
4
+ 4
n
= (2k)
4
+ 4
2k
= 16k
4
+ 2
4k
,
= 2
_
8k
4
+ 2
4k1
_
.
Portanto, neste caso, n
4
+4
n
e um n umero par maior do que 2. Logo,
se n > 1 e qualquer n umero inteiro positivo par temos que n
4
+ 4
n
nao e um n umero primo.
LivroOlimpiadaOcial
2006/5/27
page 61
i
i
i
i
i
i
i
i
[SEC. 2.3: N

UMEROS PRIMOS E COMPOSTOS 61


se n e um n umero mpar, entao n = 2k+1 para algum inteiro positivo
k 1. Assim,
n
4
+ 4
n
= (2k + 1)
4
+ 4
2k+1
= (2k + 1)
4
+ 4 4
2k
= (2k + 1)
4
+ 4 2
4k
= (2k + 1)
4
+ 4
_
2
k
_
4
.
Logo, tomando a = 2k + 1 e b = 2
k
, o resultado e uma conseq uencia
direta da identidade de Sophie Germain.
Exemplo 2.43. O n umero N = 4
2005
+ 2005
4
e primo?
Soluc ao. Notemos que
4
2005
+ 2005
4
= 4 4
2004
+ 2005
4
= 4
_
4
501
_
4
+ 2005
4
.
Na Identidade de Sophie Germain considerando a = 2005 e b = 4
501
, temos
que
N = (2005
2
+ 2 4
1002
+ 2 2005 4
501
) (2005
2
+ 2 4
1002
2 2005 4
501
).
Logo, o n umero 4
2005
+ 2005
4
nao e primo.
Tambem podemos resolver problemas deste tipo de varias formas.
Exemplo 2.44. Mostre que o n umero 5
20
+ 2
30
e composto.
Soluc ao 1. Escrevemos
5
20
+ 2
30
= 5
54
+ 2
2
2
28
=
_
5
5
_
4
+ 4
_
2
7
_
4
,
de onde podemos usar a Identidade de Sophie Germain com a = 5
5
e b = 2
7
para comprovar que o n umero 5
20
+ 2
30
e composto.
Soluc ao 2. As seguintes igualdades sao validas:
5
20
+ 2
30
=
_
5
10
_
2
+
_
2
15
_
2
=
_
5
10
_
2
+ 2 5
10
2
15
+
_
2
15
_
2
2 5
10
2
15
=
_
5
10
+ 2
15
_
2
2 5
10
2
15
=
_
5
10
+ 2
15
_
2
5
10
2
16
=
_
5
10
+ 2
15
+ 5
5
2
8
_ _
5
10
+ 2
15
5
5
2
8
_
.
LivroOlimpiadaOcial
2006/5/27
page 62
i
i
i
i
i
i
i
i
62 [CAP. 2: DIVISIBILIDADE
A ultima identidade mostra que o n umero 5
20
+ 2
30
e composto.
Finalizamos esta se c ao com mais alguns exemplos interessantes.
Exemplo 2.45. Mostre que o inteiro positivo
5
125
1
5
25
1
e composto.
Soluc ao. Sabemos que x
5
1 = (x 1)(x
4
+ x
3
+ x
2
+ x + 1), de onde
x
5
1
x 1
= x
4
+ x
3
+ x
2
+ x + 1 = (x
2
+ 3x + 1)
2
5x(x + 1)
2
.
Tomando x = 5
25
obtemos
5
125
1
5
25
1
=
_
5
50
+ 3 5
25
+ 1
_
2
5 5
25
_
5
25
+ 1
_
2
,
=
_
5
50
+ 3 5
25
+ 1
_
2
5
26
_
5
25
+ 1
_
2
,
=
_
5
50
+ 3 5
25
+ 1
_
2

_
5
13
_
5
25
+ 1
__
2
,
que e composto por ser uma diferen ca de quadrados.
Exemplo 2.46. Demonstre que o n umero 1 000 . . . 00
. .
2006 zeros
1 e composto.
Soluc ao. Observemos que
1 000 . . . 00
. .
2006 zeros
1 =10
2007
+ 1;
=
_
10
669
_
3
+ 1;
da identidade a
3
+ 1 = (a + 1)(a
2
a + 1) segue-se que:
1 000 . . . 00
. .
2006 zeros
1 =
_
10
669
+ 1
_ _
10
1338
10
669
+ 1
_
.
Esta ultima igualdade mostra que o n umero 1 000 . . . 00
. .
2006 zeros
1 e composto.
LivroOlimpiadaOcial
2006/5/27
page 63
i
i
i
i
i
i
i
i
[SEC. 2.4: UM POUCO SOBRE EQUAC

OES DIOFANTINAS 63
x y
Figura 2.3::
2.4 Um pouco sobre Equa coes Diofantinas
Come camos esta se c ao com uma brincadeira interessante.
Joao, ao sair da aula de matematica do professor Peitagoras, encontrou
Pedro e lhe prop os a seguinte brincadeira:
- Pense numa pe ca de domino, Pedro. Vou adivinhar que pe ca e essa
usando uma formula magica.
- Ok, Joao. Pode come car, ja pensei.
- Escolha um dos n umeros na pe ca e multiplique por 5. Depois disso
some tres a esse resultado. Multiplique agora o n umero que voce obteve por
dois. Some isto com o outro n umero da pe ca. Qual foi o resultado?
- Foi 40.
- Entao a pe ca que voce escolheu foi a 3 com 4!
- Como voce acertou? Me ensina!
Claro que de magico Joao nao tinha nada e decidiu contar seu segredo
a Pedro.
O jogo funciona assim: cada parte da pe ca de domino pode ser consider-
ada como um dos dgitos de um n umero de 2 algarismos, o qual denotamos
por N = xy = 10x + y (veja a Figura 2.3). Acompanhando os passos de
Joao, temos que:
(5x + 3)2 + y = 40 10x + y = 34, (2.29)
que claramente, tem por solu c oes: x = 3 e y = 4, usando a representa c ao
de 34 na base decimal.
A equa c ao (2.29) e um caso particular da seguinte forma mais geral:
ax + by = c, (2.30)
LivroOlimpiadaOcial
2006/5/27
page 64
i
i
i
i
i
i
i
i
64 [CAP. 2: DIVISIBILIDADE
onde a, b, c Z, com a = 0 e b = 0.
A equa c ao (2.30) e chamada de equac ao diofantina linear e uma solu c ao
desta e qualquer par de inteiros (x, y) que satisfa cam (2.30).

E conhecido
que todos os pontos do plano, com coordenadas (x, y), que satisfazem a
igualdade (2.30) representam, geometricamente, uma reta. Logo, as solu c oes
de uma equa c ao diofantina linear sao os pontos de coordenadas inteiras
(tambem conhecidos como pontos latices) que estao dispostos sobre a reta
que esta representa. Por exemplo, os pontos (1, 2) e (1, 1) sao solu c oes
da equa c ao diofantina 3x 2y = 1, veja a Figura 2.4.
-3 -2 -1 0 1 2 3
-3
-2
-1
0
1
2
3
x
y

L
Figura 2.4: A equacao da reta L e 3x 2y = 1.
Naturalmente nos perguntamos:

E sempre possvel achar solu c oes para
uma equa c ao diofantina linear ? A resposta e nao; o proximo resultado nos
diz quando isto e possvel. Alem disso, se uma equa c ao diofantina linear
tem uma solu c ao na verdade ela tem uma innidade de solu c oes.
Proposi cao 2.47. A equac ao diofantina linear
ax + by = c, a, b, c Z, com a = 0 e b = 0, (2.31)
tem soluc ao se, e somente se, d | c, onde d = (a, b). Alem disso, se (x
0
, y
0
)
e uma soluc ao, ent ao o conjunto de soluc oes de (2.31) e constitudo por
todos os pares de inteiros (x, y) da forma:
x = x
0
+ t
b
d
e y = y
0
t
a
d
, t Z. (2.32)
Demonstrac ao. Primeiramente suponhamos que (x
0
, y
0
) e uma solu c ao de
(2.31), logo ax
0
+ by
0
= c. Usando que d = (a, b) sabemos que existem
LivroOlimpiadaOcial
2006/5/27
page 65
i
i
i
i
i
i
i
i
[SEC. 2.4: UM POUCO SOBRE EQUAC

OES DIOFANTINAS 65
inteiros q
1
e q
2
, tais que dq
1
= a e dq
2
= b. Portanto, se verica a
igualdade
dq
1
x
0
+ dq
2
y
0
= d(q
1
x
0
+ q
2
y
0
) = c,
de onde segue obviamente que d | c.
Reciprocamente, suponhamos que d | c e portanto c = qd com q inteiro.
O Teorema de Bezout nos garante a existencia de dois inteiros, x
0
e y
0
, tais
que ax
0
+ by
0
= d. Multiplicando ambos os lados desta ultima igualdade
por q temos que
ax
0
q + by
0
q = c,
logo o par (x
1
, y
1
), com x
1
= x
0
q e y
1
= y
0
q, e solu c ao da equa c ao dio-
fantina.
Resta provar agora que temos innitas solu c oes da forma (2.32). Com
efeito, sendo (x, y) uma outra solu c ao qualquer alem de (x
0
, y
0
), vale que
ax
0
+ by
0
= c = ax + by, de onde ax
0
+ by
0
= ax + by. Desta igualdade
obtemos a(x x
0
) = b(y
0
y) e dividimos esta ultima por d para obtermos
a
d
(x x
0
) =
b
d
(y
0
y).
Como o (
a
d
,
b
d
) = 1, entao temos que
a
d
| (y
0
y) e
b
d
| (xx
0
). Logo, existe
inteiro t tal que
x = x
0
+ t
b
d
e y = y
0
t
a
d
.
Por outro lado, e facil vericar que para qualquer inteiro t as express oes
achadas acima para x e y resolvem a equa c ao diofantina.
A seguir damos um exemplo de como proceder para resolver equa c oes
diofantinas.
Exemplo 2.48. Achar todas as solu c oes inteiras da equa c ao
12x + 33y = 27.
Soluc ao. Observemos que (12, 33) = 3 e que 3 | 27, logo a equa c ao tem
innitas solu c oes. Como sabemos, basta achar uma delas e teremos as
restantes. Para achar esta solu c ao particular podemos trabalhar de duas
maneiras, que descrevemos a seguir:
Alternativa 1: Reduzimos a equa c ao `a forma equivalente
4x + 11y = 9,
LivroOlimpiadaOcial
2006/5/27
page 66
i
i
i
i
i
i
i
i
66 [CAP. 2: DIVISIBILIDADE
e por tentativa e erro vemos que x
0
= 5 e y
0
= 1 solucionam a mesma.
Entao pela Proposi c ao 2.47 temos que
x = 5 + 11t e y = 4t 1, t Z,
esgotam todas as solu c oes que procuramos.
Alternativa 2: Aplicamos o Algoritmo de Euclides para achar o m.d.c (12, 33),
obtendo os seguintes resultados:
33 = 12 2 + 9,
12 = 9 1 + 3,
9 = 3 3 + 0.
Da segunda e primeira igualdades temos, respectivamente, que
3 = 12 9 1 e 9 = 33 12 2.
Usando estas duas obtemos
3 = 12 (33 12 2) 1
= 12 33 + 12 2
= 3 12 1 33,
ou seja, achamos x
0
= 3 e y
0
= 1, garantidos pelo Teorema de Bezout,
que validam 3 = 12x
0
+ 33y
0
. Multiplicamos por 9 esta ultima igualdade
para obter
27 = 12(9x
0
) + 33(9y
0
).
Portanto, x
0
= 9x
0
= 27 e y
0
= 9y
0
= 9 resolvem, particularmente, a
equa c ao diofantina. Analogamente, como na alternativa, anterior podemos
escrever a solu c ao geral da forma:
x = 27 + 11s e y = 4s 9, s Z.
2.5 Exerccios
1. Encontre o resto que deixa
LivroOlimpiadaOcial
2006/5/27
page 67
i
i
i
i
i
i
i
i
[SEC. 2.5: EXERC

ICIOS 67
(a) 2001 2002 2003 2004 + 2005
2
quando e dividido por 7;
(b) 2
100
quando e dividido por 3;
(c)
_
12371
56
+ 34
_
28
quando e dividido por 111.
2. Provar que o n umero n
5
+ 4n e divisvel por 5 para qualquer n umero
natural n.
3. Provar que o n umero n
3
n e divisvel por 24 para qualquer n umero
natural n mpar.
4. Mostre que se n e mpar, entao n
2
1 e divisvel por 8.
5. Prove que se a e mpar, entao a
2
+ (a + 2)
2
+ (a + 4)
2
+ 1 e divisvel
por 12.
6. O n umero 2
1093
2 e divisvel por 1093
2
?
7. Utilizando o fato de que o resto de um quadrado quando dividido
por 4 so pode ser 0 ou 1, de uma outra solu c ao para o problema do
Exemplo 2.39.
8. Dados 3 inteiros tais que x
2
+y
2
= z
2
, mostre que x e y nao sao ambos
mpares e que xy e m ultiplo de 6.
9. Demonstre que o quadrado de um inteiro e da forma 8n ou 8n +1 ou
8n + 4.
10. Tres n umeros primos p, q e r, maiores que 3, formam uma progressao
aritmetica, ou seja, q = p +d e r = p +2d. Prove que d e divisvel por
6.
11. Demonstrar que existem innitos n umeros primos da forma 4m + 3 e
da forma 6m + 5, onde m Z.
12. Encontrar o ultimo dgito dos n umeros
(a) 1989
2005
;
(b) 777
777
+ 2
50
;
(c) 1 + 2
2
+ 3
2
+ + 2005
2
.
LivroOlimpiadaOcial
2006/5/27
page 68
i
i
i
i
i
i
i
i
68 [CAP. 2: DIVISIBILIDADE
13. Prove que a soma dos quadrados de cinco n umeros consecutivos nao
e um quadrado perfeito.
14. Prove que 1 00 00
. .
100zeros
5 00 00
. .
100zeros
1 nao e um cubo perfeito.
15. Use o Lema dos Restos para dar uma prova diferente para a Proposi c ao 2.5.
16. Dados n N e a
j
Z, j = 0, 1, , n; a
n
> 0. Prove que o conjunto
L
n
=
_
a
n
x
n
+ a
n1
x
n1
+ + a
0
, x Z
_
contem innitos n umeros compostos.
17. Prove que os n umeros
(a)
n
= 1 +
1
2
+
1
3
+ +
1
n
, com n > 1,
(b)
n
=
1
3
+
1
5
+ +
1
2n+1
, com n > 0,
nao sao inteiros.
18. Dizemos que um conjunto A
n
formado por n inteiros positivos escritos
no sistema binario (base 2) e regular se, para qualquer s inteiro nao
negativo a quantidade de n umeros de A
n
que contemplam 2
s
na rep-
resenta c ao binaria e par. Dizemos que A
n
e irregular se, pelo menos
para algum s este n umero e mpar. Demonstre que um sistema ir-
regular pode se converter em regular excluindo-se apenas um unico
elemento do mesmo, e, um sistema regular pode se converter em ir-
regular excluindo-se qualquer um dos seus elementos.
19. Seja n um inteiro positivo. Demonstrar que todos os coecientes do
desenvolvimento do binomio de Newton (a + b)
n
sao mpares se, e
somente se, n e da forma 2
s
1.
20. Prove que se (x
0
, y
0
) e uma solu c ao da equa c ao diofantina linear ax
by = 1, entao a area do tri angulo cujos vertices sao (0, 0), (b, a) e
(x
0
, y
0
) e 1/2.
21. Qual e a menor distancia possvel entre dois pontos (x
1
, y
1
) e (x
2
, y
2
),
com coordenadas inteiras, situados sobre a reta denida pela equa c ao
diofantina ax by = c ?
LivroOlimpiadaOcial
2006/5/27
page 69
i
i
i
i
i
i
i
i
Captulo 3
Contagem
Neste captulo discutiremos problemas envolvendo a contagem de elementos
de um conjunto dado. Por exemplo, responderemos perguntas do tipo: de
quantos modos podemos distribuir 32 sele c oes nacionais de futebol em seis
grupos de quatro times cada?
Para tanto, utilizaremos como ferramentas basicas os princpios aditivo
e multiplicativo da contagem. Veremos tambem que o uso simultaneo destes
princpios sera muito util para resolver problemas com certos nveis de com-
plexidade. Alem disto, serao abordados os conceitos de permuta c oes, arran-
jos e combina c oes, sendo estes de muita import ancia por serem os alicerces
de um ramo da matematica denominado Combinatoria.
Antes de prosseguirmos daremos algumas deni c oes e nota c oes que serao
uteis ao longo de todo o captulo. Dado um conjunto A denotamos por |A|
a quantidade de elementos que este possui. O produto cartesiano de n
conjuntos A
1
, A
2
, . . . , A
n1
e A
n
e o conjunto denido por
A
1
A
2
A
n
:=
_
(a
1
, a
2
, . . . , a
n
); a
i
A
i
, i = 1, 2, . . . , n
_
,
onde cada elemento (a
1
, a
2
, . . . , a
n
) e chamado de n-upla ordenada. Deno-
taremos o conjunto vazio com o smbolo .
3.1 Princpio Aditivo de Contagem
O princpio aditivo da contagem garante que dados dois conjuntos que nao
tem elemento em comum, o n umero de elementos da uniao e exatamente a
69
LivroOlimpiadaOcial
2006/5/27
page 70
i
i
i
i
i
i
i
i
70 [CAP. 3: CONTAGEM
soma do n umero de elementos de cada um, ou seja, se A
1
e A
2
sao disjuntos
(isto e, A
1
A
2
= ), entao
|A
1
A
2
| = |A
1
| +|A
2
|.
Apesar de sua simplicidade, muitos problemas podem ser resolvidos uti-
lizando esse simples princpio. A seguir enunciamos uma extensao deste
princpio para um n umero nito qualquer de conjuntos.
Princpio Aditivo da Contagem: Dados os conjuntos A
1
, A
2
, . . . ,
A
n
dois a dois disjuntos (isto e, A
i
A
j
= , i = j), temos que
|A
1
A
2
A
n
| = |A
1
| +|A
2
| + +|A
n
|.
Exemplo 3.1. Em Maceio entraram em cartaz 4 lmes distintos e 2 pe cas
de teatro. Se Pedro Vtor so tem dinheiro para assistir a um lme ou a uma
pe ca de teatro, diga quantos sao os possveis programas de Pedro Vtor.
Soluc ao. Denotemos por f
1
, f
2
, f
3
e f
4
os quatro lmes que estao em cartaz
e por t
1
e t
2
as duas pe cas de teatro. Agora, representemos pelo par (i, j),
com 0 i 4 e 0 j 2, o programa que consiste em assistir ao lme
f
i
e `a pe ca t
j
(caso i = 0 ou j = 0 isso signica que nao sera assistido a
nenhum lme ou a nenhuma pe ca, respectivamente).
Pelas limita c oes econ omicas do Pedro Vtor temos que ele so pode es-
colher um programa dentro dos seguintes conjuntos disjuntos:
A
1
=
_
(1, 0), (2, 0), (3, 0), (4, 0)
_
e A
2
=
_
(0, 1), (0, 2)
_
.
Logo, no total sao |A
1
A
2
| = |A
1
| + |A
2
| = 6 programas distintos, entre
os quais Pedro Vtor tera que escolher um.
Exemplo 3.2. Numa reuniao havia um certo n umero de pessoas e todos os
presentes apertaram as m aos entre si. Sabendo-se que ao todo foram feitos
66 cumprimentos, calcule o n umero de pessoas presentes `a reuniao.
Soluc ao. Vamos enumerar as pessoas com os n umeros do conjunto P =
{1, 2, . . . , n}. A cada aperto de m ao associaremos um par (i, j), signicando
que a pessoa i apertou a m ao da pessoa j. Assim, os apertos de m ao
envolvendo a pessoa 1 foram:
A
1
= {(1, 2), (1, 3), . . . , (1, n)}.
LivroOlimpiadaOcial
2006/5/27
page 71
i
i
i
i
i
i
i
i
[SEC. 3.1: PRINC

IPIO ADITIVO DE CONTAGEM 71


Do mesmo modo, denimos os apertos de m ao envolvendo a pessoa 2 que
nao envolvem a pessoa 1, como:
A
2
= {(2, 3), (2, 4), . . . , (2, n)}.
Note que o aperto (2, 1) e o mesmo que o aperto (1, 2), ja que se 1 aperta a
m ao de 2, entao 2 aperta a m ao de 1. Analogamente,
A
i
= {(i, i + 1), (i, i + 2), . . . , (i, n)}, para 1 i n.
Note que A
i
A
j
= para i = j. Observe tambem que todos os apertos
aparecem em um dos conjuntos A
i
. Assim, A
1
A
n
contem todos os
apertos de m ao. Logo, pelo Princpio Aditivo:
|A
1
A
2
A
n
| = |A
1
| +|A
2
| + . . . |A
n
|
= (n 1) + (n 2) + + 2 + 1
=
(n 1)n
2
= 66.
Resolvendo em n, temos que n = 12.
Vimos que o princpio aditivo nos fornece o n umero de elementos de
qualquer uniao de conjuntos dois a dois disjuntos. Discutiremos agora uma
extensao do princpio para qualquer uniao de conjuntos, nao necessaria-
mente dois a dois disjuntos.
Proposi cao 3.3. Considere A
1
e A
2
dois conjuntos arbitr arios, ent ao:
|A
1
A
2
| = |A
1
| +|A
2
| |A
1
A
2
|.
Demonstrac ao. Observe que
A
1
A
2
= (A
1
A
2
) A
2
onde a uniao e dois a dois disjunta. Pelo principio aditivo, temos que
|A
1
A
2
| = |A
1
A
2
| +|A
2
|. (3.1)
Analogamente, aplicando novamente este pricpio, temos que
|A
1
| = |A
1
A
2
| +|A
1
A
2
|; (3.2)
A Proposi c ao segue imediatamente combinando as igualdades (3.1) e (3.2).
LivroOlimpiadaOcial
2006/5/27
page 72
i
i
i
i
i
i
i
i
72 [CAP. 3: CONTAGEM
Para chegar a uma express ao analoga `a do princpio aditivo, vamos fazer
mais um caso, considerando agora tres conjuntos.
Corolario 3.4. Considere A
1
, A
2
e A
3
tres conjuntos quaisquer. Ent ao,
|A
1
A
2
A
3
| =|A
1
| +|A
2
| +|A
3
|

_
|A
1
A
2
| +|A
1
A
3
| +|A
2
A
3
|
_
+|A
1
A
2
A
3
|.
Demonstrac ao. Pela Proposi c ao 3.3 temos que,
|A
1
(A
2
A
3
)| = |A
1
| +|A
2
A
3
| |A
1
(A
2
A
3
)|,
de onde,
|A
1
A
2
A
3
| = |A
1
| +|A
2
A
3
| |(A
1
A
2
) (A
1
A
3
)|.
Novamente, pela Proposi c ao 3.3 temos que,
|A
1
A
2
A
3
| = |A
1
| +|A
2
| +|A
3
| |A
2
A
3
| |(A
1
A
2
) (A
1
A
3
)|.
Aplicando mais uma vez a Proposi c ao 3.3 temos que,
|(A
1
A
2
) (A
1
A
3
)| = |A
1
A
2
| +|A
1
A
3
| |(A
1
A
2
) (A
1
A
3
).
Combinando as duas ultimas igualdades obtemos
|A
1
A
2
A
3
| =|A
1
| +|A
2
| +|A
3
|

_
|A
1
A
2
| +|A
1
A
3
| +|A
2
A
3
|
_
+|A
1
A
2
A
3
| ,
como desejavamos.
Para facilitar nossa escrita, vamos denotar por A
1
A
2
. . . A
k
o conjunto
A
1
A
2
A
k
. Assim, outra forma de enunciar o Corolario 3.4 e a
seguinte:

3
_
i=1
A
i

=
3

i=1
|A
i
|

1i1<i23
|A
i1
A
i2
| +

1i1<i2<i33
|A
i1
A
i2
A
i3
|.
LivroOlimpiadaOcial
2006/5/27
page 73
i
i
i
i
i
i
i
i
[SEC. 3.1: PRINC

IPIO ADITIVO DE CONTAGEM 73


De forma geral, dados conjuntos A
1
, A
2
, . . . , A
n
, as express oes anteriores
nos levam a denir os n umeros:
S
1
=
n

i=1
|A
i
|
S
2
=

1i1<i2n
|A
i1
A
i2
|,
.
.
.
S
k
=

1i1<i2<<i
k
n
|A
i1
A
i2
. . . A
i
k
|,
.
.
.
S
n
= |A
1
A
2
. . . A
n
|.
Assim, a versao mais geral do princpio aditivo, tambem conhecida como
princpio de inclus ao e exclus ao, e:
Princpio Aditivo - Versao Geral: Dados quaisquer conjuntos
A
1
, A
2
. . . , A
n
, vale:

n
_
i=1
A
i

= S
1
S
2
+ S
3
S
4
+ + (1)
n1
S
n
.
Nao iremos provar essa versao, mas o leitor pode (e deve!) mostra-la
como exerccio, repetindo os argumentos anteriores. Vamos agora a uma
aplica c ao:
Exemplo 3.5. Num colegio foram entrevistados 78 estudantes. Destes, 32
estavam fazendo um curso de frances; 40 um curso de fsica; 30 um curso
de matematica; 23 um curso de historia; 19 frances e fsica; 13 frances e
matematica; 15 fsica e matematica; 2 frances e historia; 15 fsica e historia;
14 matematica e historia; 8 frances, fsica e matematica; 8 frances, fsica
e historia; 2 frances, matematica e historia; 6 fsica, matematica e historia
e 2 estavam fazendo todos os quatro cursos. Quantos estudantes estavam
fazendo pelo menos 1 curso nas 4 areas mencionadas?
LivroOlimpiadaOcial
2006/5/27
page 74
i
i
i
i
i
i
i
i
74 [CAP. 3: CONTAGEM
Soluc ao. Denotemos por A
1
, A
2
, A
3
, e A
4
os conjuntos dos estudantes
que fazem frances, fsica, matematica e historia, respectivamente. Observe-
mos que as igualdades
|A
1
| = 32,
|A
2
| = 40,
|A
3
| = 30,
|A
4
| = 23,
nos dao que S
1
=
4

i=1
|A
i
| = 125; as igualdades
|A
1
A
2
| = 19,
|A
1
A
3
| = 13,
|A
1
A
4
| = 2,
|A
2
A
3
| = 15,
|A
2
A
4
| = 15,
|A
3
A
4
| = 14,
nos dao que S
2
=

1i1<i24
|A
i1
A
i2
| = 78; as igualdades
|A
1
A
2
A
3
| = 8,
|A
1
A
2
A
4
| = 8,
|A
1
A
3
A
4
| = 2,
|A
2
A
3
A
4
| = 6,
nos dao que S
3
=

1i1<i2<i34
|A
i1
A
i2
A
i3
| = 24; assim como que S
4
=
|A
1
A
2
A
3
A
4
| = 2.
Segue-se entao, do princpio aditivo, que

4
_
i=1
A
i

= 125 78 + 24 2 =
69.
LivroOlimpiadaOcial
2006/5/27
page 75
i
i
i
i
i
i
i
i
[SEC. 3.1: PRINC

IPIO ADITIVO DE CONTAGEM 75


Deni cao 3.6. Denimos o complementar do conjunto A em rela c ao ao
conjunto U como sendo um subconjunto de U dado por
A
c
=
_
x U; x / A
_
.
U
A
Figura 3.1: A area pintada de verde corresponde a A
c
e o conjunto U e repre-
sentado por todo o retangulo.
Neste caso e facil vericar que os conjuntos A e A
c
sao disjuntos e que
U = A A
c
. Segue-se do princpio aditivo que |U| = |A| +|A
c
|; portanto,
|A
c
| = |U| |A|.
Analogamente, dados dois conjuntos A
1
U e A
2
U, temos que A
1
A
2
e (A
1
A
2
)
c
sao disjuntos e, alias, U = (A
1
A
2
)(A
1
A
2
)
c
. Novamente,
pelo do princpio aditivo, vale que
|U| = |A
1
A
2
| +|(A
1
A
2
)
c
|;
e conseq uentemente temos que
|(A
1
A
2
)
c
| = |U| (|A
1
| +|A
2
|) +|A
1
A
2
|.
Similarmente, dados tres conjuntos A
1
U, A
2
U e A
3
U podemos
demonstrar que
|(A
1
A
2
A
3
)
c
| = |U| (|A
1
| +|A
2
| +|A
3
|)
+ (|A
1
A
2
| +|A
1
A
3
| +|A
2
A
3
|)
|A
1
A
2
A
3
|.
Entao, usando a nota c ao S
0
= |U|, temos a seguinte proposi c ao:
LivroOlimpiadaOcial
2006/5/27
page 76
i
i
i
i
i
i
i
i
76 [CAP. 3: CONTAGEM
Proposi cao 3.7. Para toda famlia de subconjuntos A
i
U, i = 1, 2, . . . , n,
vale a relac ao:

_
n
_
i=1
A
i
_
c

= S
0

_
S
1
S
2
+ S
3
S
4
+ (1)
n1
S
n
_
= S
0
S
1
+ S
2
S
3
+ S
4
+ (1)
n
S
n
,
ou resumidamente,

_
n
_
i=1
A
i
_
c

= |A
c
1
A
c
2
A
c
n
| =
n

j=0
(1)
j
S
j
.
Observa cao 3.8. Observemos que na ultima rela c ao da proposi c ao usamos
a conhecida lei de Morgan: o complementar da uniao de uma familia nita
conjuntos, em rela c ao a um conjunto U, e a intersec c ao dos complementares
de cada um deles.
3.2 Princpio Multiplicativo de Contagem
Come camos esta se c ao discutindo um problema relacionado com o apaixo-
nante jogo de xadrez. O mesmo consiste no seguinte: queremos saber de
quantas maneiras diferentes podemos colocar duas torres num tabuleiro de
xadrez de forma tal que nenhuma ataque a outra. Uma situa c ao como a
que procuramos e mostrada na Figura 3.2, pois lembramos que torres so
se movimentam na dire c ao horizontal ou na dire c ao vertical do tabuleiro.
Antes de prosseguir deixamos claro o seguinte: se na Figura 3.2 trocamos
a posi c ao da torre a com a torre b consideraremos isto como uma situa c ao
diferente.
Notemos o seguinte: uma vez que coloquemos uma das torres numa
casa do tabuleiro nao podemos colocar a segunda torre na mesma linha ou
coluna em que esta se encontra, pois ela seria amea cada. Como cada linha e
cada coluna contem 8 casas do tabuleiro, sendo uma delas comum a ambas,
entao temos 15 posi c oes proibidas para colocar a segunda torre, ou seja, ela
so pode ser colocada em 64 15 = 49 posi c oes diferentes. Resumindo, por
cada uma das 64 possveis posi c oes para a torre a temos 49 possibilidades
diferentes para colocar a torre b, totalizando 64 49 = 3136 formas diferentes
de colocar ambas as torres no tabuleiro sem que elas se ataquem.
LivroOlimpiadaOcial
2006/5/27
page 77
i
i
i
i
i
i
i
i
[SEC. 3.2: PRINC

IPIO MULTIPLICATIVO DE CONTAGEM 77


a
b
Figura 3.2::
O exemplo acima traz a essencia do que e chamado princpio multiplica-
tivo da contagem: se um evento A
1
pode ocorrer de m maneiras distintas
e, se para cada uma dessas m maneiras possveis de A
1
ocorrer, um outro
evento A
2
pode ocorrer de n maneiras distintas, entao o n umero de maneiras
de ocorrerem sucessivamente os eventos A
1
e A
2
e m n. Na linguagem
matematica: dados dois conjuntos A
1
e A
2
, temos que
|A
1
A
2
| = |A
1
| |A
2
|.
Uma extensao deste princpio para um n umero nito qualquer de con-
juntos e a seguinte:
Princpio Multiplicativo da Contagem: Dados os conjuntos A
1
, A
2
,
. . . , A
n
temos que
|A
1
A
2
A
n
| = |A
1
| |A
2
| |A
n
|.
Note que neste princpio, nao e necessaria nenhuma hipotese adicional
sobre os conjuntos A
i
. Vamos agora dar alguns exemplos de como aplicar
esse princpio.
Exemplo 3.9. Em Maceio entraram em cartaz 4 lmes distintos e 2 pe cas
de teatro. Se agora o Pedro Vtor tem dinheiro para assistir exatamente a
um lme e a uma pe ca de teatro, diga quantos sao os possveis programas
que Pedro Vtor pode fazer.
LivroOlimpiadaOcial
2006/5/27
page 78
i
i
i
i
i
i
i
i
78 [CAP. 3: CONTAGEM
Soluc ao. Denotemos por f
1
, f
2
, f
3
e f
4
os quatro lmes que estao em cartaz
e por t
1
e t
2
as duas pe cas de teatro. Denamos os conjuntos
A
1
= {f
1
, f
2
, f
3
, f
4
} e A
2
= {t
1
, t
2
}.
Neste caso, as condi c oes econ omicas do Pedro Vtor permitem que ele
escolha um elemento do conjunto A
1
e outro elemento do conjunto A
2
. Este
tipo de escolha representa-se pelo conjunto
A
1
A
2
=
_
(f
i
, t
j
); 1 i 4 e 1 j 2
_
,
onde cada par (f
i
, t
j
) representa o programa que consiste em assistir ao lme
f
i
e `a pe ca t
j
. Logo, no total sao |A
1
A
2
| = |A
1
| |A
2
| = 8 programas
distintos.
Exemplo 3.10. Se numa loja de doces existem 9 tipos distintos de balas
e 5 tipos distintos de chiclete, diga quantas escolhas podemos fazer para
comprar somente uma bala e um chiclete.
Soluc ao. Denotemos por b
1
, b
2
, b
3
, b
4
, b
5
, b
6
, b
7
, b
8
e b
9
os nove tipos
distintos de balas e por c
1
, c
2
, c
3
, c
4
e c
5
os cinco tipos distintos de chicletes.
Denamos os conjuntos
B = {b
1
, b
2
, b
3
, b
4
, b
5
, b
6
, b
7
, b
8
, b
9
} e C = {c
1
, c
2
, c
3
, c
4
, c
5
}.
Como precisamos comprar simultaneamente um elemento do conjunto B
e um elemento do conjunto C, entao o conjunto B C me da o conjunto
de todas as escolhas possveis. Logo, o n umero de escolhas possveis para
comprar simultaneamente um tipo de bala e um tipo de chiclete e |BC| =
9 5 = 45.
Exemplo 3.11. O alfabeto Portugues e formado por 5 vogais e 18 con-
soantes. Suponha que uma slaba do alfabeto esteja formada por uma unica
consoante seguida de uma unica vogal. Diga quantas slabas tem nosso
alfabeto.
Soluc ao. Considere os conjuntos,
V ={ a, e, i, o, u },
C ={ b, c, d, f, g, h, j, l, m, n, p, q, r, s, t, v, x, z }.
LivroOlimpiadaOcial
2006/5/27
page 79
i
i
i
i
i
i
i
i
[SEC. 3.2: PRINC

IPIO MULTIPLICATIVO DE CONTAGEM 79


Uma slaba, pela deni c ao dada no nosso problema, e formada por uma
unica consoante seguida de uma unica vogal. Assim, o conjunto de todas
as slabas de nosso alfabeto e formado pelo conjunto C V. Como |C| = 18
e |V| = 5, segue-se que o n umero de slabas possveis e 18 5 = 90.
Exemplo 3.12. De quantas maneiras 2 pessoas podem estacionar seus
carros numa garagem com 10 vagas?
Soluc ao. Observando que a primeira pessoa pode estacionar seu carro de
10 formas distintas e que a segunda pessoa pode estacionar seu carro de 9
formas distintas, temos pelo princpio multiplicativo que existem 9 10 = 90
formas possveis nas quais duas pessoas podem estacionar seus carros numa
garagem com 10 vagas.
Exemplo 3.13. Dado o n umero 720, diga
(a) quantos divisores inteiros e positivos ele possui;
(b) dentre seus divisores inteiros e positivos, quantos sao pares;
(c) dentre seus divisores inteiros e positivos, quantos sao mpares;
(d) dos divisores acima aludidos quantos sao quadrados perfeitos.
Soluc ao. Pelo Teorema Fundamental da Aritmetica, todo n umero inteiro
positivo e primo ou produto de primos. Observe que a decomposi c ao de 720
em fatores primos vem dada por:
720 = 2
4
3
2
5
1
. (3.3)
Agora denamos os seguintes conjuntos:
A ={ todos os divisores de 720 que sao da forma 2
k
, onde k Z
+
},
B ={ todos os divisores de 720 que sao da forma 3
m
, onde m Z
+
},
C ={ todos os divisores de 720 que sao da forma 5
n
, onde n Z
+
}.
Observemos que 0 k 4, pois se k > 4 entao pelo menos a potencia
2
5
deveria estar presente em (3.3); como isto nao acontece segue-se que
0 k 4, de modo que
A =
_
2
0
, 2
1
, 2
2
, 2
3
, 2
4
_
,
LivroOlimpiadaOcial
2006/5/27
page 80
i
i
i
i
i
i
i
i
80 [CAP. 3: CONTAGEM
seguindo o mesmo raciocnio, podemos demonstrar que 0 m 2 e que
0 n 1. Assim,
B =
_
3
0
, 3
1
, 3
2
_
e C =
_
5
0
, 5
1
_
.
(a) O conjunto de todos os possveis divisores de 720 vem dado pelo con-
junto ABC. De onde o n umero de divisores inteiros e positivos de
720 e |AB C|. Porem, o Princpio Multiplicativo nos garante que
|AB C| = |A| |B| |C|. Portanto, o n umero de divisores inteiros e
positivos de 720 e 5 3 2 = 30, pois |A| = 5, |B| = 3 e |C| = 2.
(b) Para obter o conjunto de todos os divisores pares de 720 devemos
remover o elemento 2
0
do conjunto A. Assim, o conjunto de todos os
divisores pares e positivos de 720 vem dado pelo conjunto A{2
0
}
BC. O princpio multiplicativo nos garante que

_
A{2
0
}
_
BC

A{2
0
}

|B| |C|. Portanto, o n umero de divisores pares e positivos


de 720 e 4 3 2 = 24, pois

A{2
0
}

= 4, |B| = 3 e |C| = 2.
(c) Para obter o conjunto de todos os divisores mpares de 720 devemos
remover os elementos 2
1
, 2
2
, 2
3
e 2
4
do conjunto A. Assim, o con-
junto de todos os divisores mpares e positivos de 720 vem dado pelo
conjunto
_
A{2
1
, 2
2
, 2
3
, 2
4
}
_
B C.
O princpio multiplicativo nos garante que

_
A{2
1
, 2
2
, 2
3
, 2
4
}
_
B C

A{2
1
, 2
2
, 2
3
, 2
4
}

|B| |C|.
Portanto, o n umero de divisores mpares e positivos de 720 e 132 =
6; pois

A{2
1
, 2
2
, 2
3
, 2
4
}

= 1, |B| = 3 e |C| = 2.
(d) Para obter o conjunto de todos os divisores de 720 que sao quadrados
perfeitos devemos car com as potencias pares nos conjuntos A, B e
C, respectivamente. Portanto, devemos remover os elementos 2
1
, 2
3
do conjunto A. Tambem devemos remover o elemento 3
1
do conjunto
B. Finalmente do conjunto C devemos remover o elemento 5
1
. Logo, o
conjunto de todos os divisores quadrados perfeitos e positivos de 720
vem dado pelo conjunto
D :=
_
A{2
1
, 2
3
}
_

_
B {3
1
}
_

_
C {5
1
}
_
.
LivroOlimpiadaOcial
2006/5/27
page 81
i
i
i
i
i
i
i
i
[SEC. 3.2: PRINC

IPIO MULTIPLICATIVO DE CONTAGEM 81


O princpio multiplicativo nos garante que

A{2
1
, 2
3
}

B {3
1
}

C {5
1
}

.
Portanto, o n umero de divisores quadrados perfeitos e positivos de 720
e 3 2 1 = 6; pois

A{2
1
, 2
3
}

= 3,

B {3
1
}

= 2 e

C {3
1
}

= 1.
Observe que {1, 4, 9, 16, 36, 144} e o conjunto dos divisores de 720 que
sao quadrados perfeitos.
Exemplo 3.14. Se um n umero natural n se fatora como
n = p
k1
1
p
k2
2
p
kr
r
, (3.4)
onde os p
i
sao n umeros primos distintos e cada k
i
Z
+
, entao o n umero de
divisores positivos de n,denotado por d(n) e
d(n) = (k
1
+ 1)(k
2
+ 1) . . . (k
r
+ 1).
Soluc ao. Dena o conjunto
A
1
={ todos os divisores de n que sao da forma p
m1
1
, onde m Z
+
},
e em geral, dena
A
i
={ todos os divisores de n que sao da forma p
mi
i
, onde t Z
+
}.
Observemos que m
i
p
i
, pois se m
i
> p
i
, entao pelo menos a potencia
p
ki+1
i
deveria estar presente em (3.4); como isto nao acontece segue-se que
m
i
p
i
, de modo que
A
i
=
_
p
0
i
, p
1
i
, p
2
i
, . . . , p
ki
i
_
, para i = 1, 2, 3, . . . , k
i
.

E imediato ver que

A
i

= k
i
+ 1.
O conjunto de todos os possveis divisores de n vem dado pelo conjunto
A
1
A
2
A
r
. De onde se conclui que o n umero de divisores inteiros
e positivos de n e
d(n) = |A
1
A
2
A
r
| = |A
1
| |A
2
| |A
r
|,
LivroOlimpiadaOcial
2006/5/27
page 82
i
i
i
i
i
i
i
i
82 [CAP. 3: CONTAGEM
onde na ultima igualdade usamos o Princpio Multiplicativo. Portanto, o
n umero de divisores inteiros e positivos de n e
d(n) = (k
1
+ 1)(k
2
+ 1) . . . (k
r
+ 1).
Exemplo 3.15. De quantas maneiras podemos escolher dois inteiros de 1
a 20 de forma que a soma seja mpar?
Soluc ao. Observemos que
A soma de dois n umeros inteiros pares e um n umero par. Com efeito,
para quaisquer a, b Z temos que 2a + 2b = 2(a + b);
A soma de dois n umeros inteiros mpares e um n umero par. Com
efeito, para quaisquer a, b Z temos que (2a + 1) + (2b + 1) = 2(a +
b + 1);
A soma de um n umero inteiro par com qualquer outro inteiro mpar
sempre e um inteiro mpar. Com efeito, para quaisquer a, b Z temos
que 2a + (2b + 1) = 2(a + b) + 1.
Isto nos sugere denir os conjuntos
P = {2, 4, 6, 8, 10, 12, 14, 16, 18, 20},
I = {1, 3, 5, 7, 9, 11, 13, 15, 17, 19},
onde P I sao todas as formas possveis de somar um n umero inteiro par
com outro mpar. O princpio multiplicativo nos garante que nossa resposta
e |P I| = |P| |I| = 100, pois |P| = |I| = 10.
3.3 Uso simultaneo dos Princpios Aditivo e
Multiplicativo
Aproveitamos esta se c ao para apresentar problemas um pouco mais difceis
que os tratados nas se c oes anteriores. Nestes problemas, precisaremos em-
pregar simultaneamente o princpio aditivo e o princpio multiplicativo. Va-
mos ao primeiro deles:
LivroOlimpiadaOcial
2006/5/27
page 83
i
i
i
i
i
i
i
i
[SEC. 3.3: USO SIMULT

ANEO DOS PRINC

IPIOS ADITIVO E MULTIPLICATIVO 83


Exemplo 3.16. Sabemos que no incio da premia c ao da 1
a
fase da Olimpada
Alagoana de Matematica existem 10 livros diferentes de

Algebra, 7 livros
diferentes de Combinatoria e 5 livros diferentes de Geometria para home-
nagear os vencedores. Danielle e a primeira a pegar o premio que consiste
em 2 livros, com a condi c ao de que estes nao podem ser da mesma materia.
Diga quantas escolhas Danielle pode fazer para pegar seu premio.
Soluc ao. Denotemos por
A = {a
1
, . . . , a
10
}, C = {c
1
, . . . , c
7
} e G = {g
1
, . . . , g
5
}, os conjuntos de
livros de

Algebra, Combinatoria e Geometria, respectivamente. Observemos
que |A| = 10, |C| = 7 e |G| = 5 e Danielle tem as seguintes possibilidades
de escolha:
Escolher um livro de A e um livro de C. Neste caso, Danielle tem
|AC| = |A| |C| = 70 escolhas possveis (devido ao Princpio Multi-
plicativo).
Escolher um livro de A e um livro de G. Neste caso, Danielle tem
|AG| = |A| |G| = 50 escolhas possveis (devido ao Princpio Multi-
plicativo) ou
Escolher um livro de C e um livro de G. Neste caso, Danielle tem
|C G| = |C| |G| = 35 escolhas possveis (devido ao Princpio Multi-
plicativo).
Agora o Princpio aditivo nos garante que o n umero total de escolhas que
Danielle pode fazer e 70 + 50 + 35 =155.
Exemplo 3.17. Ha 18 mo cas e 12 rapazes, onde 5 deles sao irmaos (3
mo cas e 2 rapazes) e os restantes nao possuem parentesco. Diga quantos
casamentos sao possveis naquela turma (sabendo que irmaos nao se casam).
Soluc ao. Observemos que 15, dentre as 18 mo cas, nao tem parentesco
nenhum com os 12 rapazes, logo, pelo Princpio Multiplicativo temos que e
possvel efetuar 15 12 = 180 casamentos diferentes entre eles. Por outro
lado, as 3 mo cas restantes podem efetuar casamento com 10 dos 12 rapazes,
pois 2 deles sao seus irmaos. Novamente, pelo Princpio Multiplicativo e
possvel realizar 3 10 = 30 casamentos diferentes neste caso. Finalmente, o
Princpio Aditivo nos da que podem ser realizados um total de 180+30 = 210
casamentos.
LivroOlimpiadaOcial
2006/5/27
page 84
i
i
i
i
i
i
i
i
84 [CAP. 3: CONTAGEM
Exemplo 3.18. Quantas palavras de 5 caracteres podem ser formadas com
as letras , e de modo que em cada palavra nao falte nenhuma dessas
letras?
Soluc ao. Denamos os seguintes conjuntos,
U :={ palavras de 5 caracteres so com as letras , e };
A

:={ palavras que estao em U e onde nao aparece a letra };


A

:={ palavras que estao em U e onde nao aparece a letra };


A

:={ palavras que estao em U e onde nao aparece a letra }.


Por exemplo,
a palavra A

;
a palavra A

;
a palavra A

.
Primeiramente, notemos que cada caracter de U pode ser escolhido de
3 formas distintas. Segue-se entao do Princpio Multiplicativo que existem
3
5
formas de escrever uma palavra de 5 caracteres usando um alfabeto de 3
letras, isto e,
S
0
= |U| = 3
5
= 243.
Calculemos agora |A

|, isto e, o n umero de palavras onde nao aparece a


letra . Para isto, observemos que cada caracter em A

pode ser escolhido


de 2 formas. Logo, o Princpio Multiplicativo nos garante que existem 2
5
palavras em A

, ou seja, |A

| = 2
5
. Analogamente, podemos mostrar que
|A

| = |A

| = 2
5
. Portanto,
S
1
= |A

| +|A

| +|A

| = 2
5
+ 2
5
+ 2
5
= 96.
Prosseguimos com o calculo de |A

|, isto e, do n umero de palavras


onde nao aparecem as letras e ; portanto, cada caracter em A

pode
ser escolhido de 1 forma. Logo, o Princpio Multiplicativo nos garante que
existem 1
5
= 1 palavra em A

, ou seja, |A

| = 1. Similarmente,
podemos mostrar que |A

| = |A

| = 1. Portanto,
S
2
= |A

| +|A

| +|A

| = 3.
LivroOlimpiadaOcial
2006/5/27
page 85
i
i
i
i
i
i
i
i
[SEC. 3.4: PERMUTAC

OES SIMPLES 85
Por m, achamos |A

|, que nos da o n umero de palavras onde nao


aparecem as letras , e ; mas cada palavra em A

tem que usar


pelo menos um dos caracteres proibidos. Logo,
S
3
= |A

| = 0.
Finalmente, observamos que o conjunto das palavras de 5 caracteres que
podem ser formadas com as letras , e de modo que em cada palavra
nao falte nenhuma dessas letras e exatamente o conjunto A
c

A
c

A
c

. Usando
a Proposi c ao 3.7, temos:
|A
c

A
c

A
c

| =S
0
S
1
+ S
2
S
3
=243 96 + 3 0
=150.
3.4 Permutacoes Simples
Deni cao 3.19. Uma permuta c ao simples de n objetos distintos e qualquer
agrupamento ordenado desses n objetos. Denotaremos por P
n
o n umero de
todas as permuta c oes simples de n objetos dados.
Por exemplo, todas as permuta c oes dos 3 elementos do conjunto A =
{a
1
, a
2
, a
3
} sao:

1
= (a
1
, a
2
, a
3
),

2
= (a
1
, a
3
, a
2
),

3
= (a
2
, a
1
, a
3
),

4
= (a
2
, a
3
, a
1
),

5
= (a
3
, a
1
, a
2
),

6
= (a
3
, a
2
, a
1
).
Proposi cao 3.20. Seja n 1. O n umero total de permutac oes simples de
n objetos O = {o
1
, o
2
, . . . , o
n
} e dado por P
n
= n!
LivroOlimpiadaOcial
2006/5/27
page 86
i
i
i
i
i
i
i
i
86 [CAP. 3: CONTAGEM
Demonstrac ao.

E claro que a formula vale para n = 1. Vejamos agora que
existe a seguinte rela c ao entre P
n
e P
n1
para n 2:
P
n
= nP
n1
. (3.5)
Para comprovar isto, para cada i denamos A
i
como sendo as permuta c oes
dos n 1 objetos {o
1
, . . . , o
i1
, o
i+1
, . . . , o
n
}. Note que |A
i
| = P
n1
, para
cada i = 1, 2, . . . , n. Assim, para obtermos uma permuta c ao dos n objetos,
basta que xemos o objeto inicial o
i
e tomemos um elemento do conjunto A
i
,
que e uma permuta c ao dos n 1 objetos restantes. Pelo princpio aditivo,
temos que:
P
n
= |A
1
| +|A
2
| + +|A
n
| = nP
n1
.
Como nossa equa c ao (3.5) e valida para todo n 2, podemos aplica-la para
n 1, obtendo:
P
n1
= (n 1)P
n2
,
de onde vem que
P
n
= n(n 1)P
n2
.
Repetindo este argumento, obtemos que
P
n
= n(n 1)(n 2) 3 2 1 = n!,
como queramos demonstrar.
Exemplo 3.21. De quantas maneiras podemos formar uma la com 4 pes-
soas?
Demonstrac ao. Observe que se enumeramos os lugares da la e enumer-
amos as pessoas, p
a
, p
b
, p
c
, p
d
, cada distribui c ao vai corresponder a uma per-
muta c ao do conjunto {1, 2, 3, 4}. Por exemplo, a distribui c ao (p
c
, p
a
, p
b
, p
d
)
corresponde `a permuta c ao (3, 1, 2, 4). Assim, o n umero de distribui c oes na
la e 4! = 24.
Exemplo 3.22. De quantas maneiras k mo cas e k rapazes podem formar
pares para uma dan ca?
Soluc ao. Estando as mo cas em uma la e os rapazes em outra, podemos
enumera-los com n umeros de 1, 2, . . . , k. A uma permuta c ao desses n umeros,
digamos (a
1
, a
2
, . . . , a
k
) com a
i
{1, 2, . . . , k} faremos uma associa c ao da
mulher i com o rapaz a
i
. Por exemplo, a permuta c ao (2, 1, 3, . . . , k) signica
LivroOlimpiadaOcial
2006/5/27
page 87
i
i
i
i
i
i
i
i
[SEC. 3.5: ARRANJOS SIMPLES 87
que a mo ca 1 dan cara com o rapaz 2, a mo ca 2 com o rapaz 1, e a mo ca i
com o rapaz i, para i 3.
Observe que toda associa c ao de k mo cas e k rapazes produz uma per-
muta c ao, de modo que o n umero de associa c oes possveis das mo cas com os
rapazes e igual ao n umero de permuta c oes do conjunto {1, 2, 3, . . . , k}. Pela
Proposi c ao 3.20 existem k! modos diferentes de combinar as mo cas com os
rapazes.
3.5 Arranjos Simples
Deni cao 3.23. Consideremos n objetos e p um inteiro positivo tal que
0 < p n. Um arranjo simples de classe p dos n objetos dados e uma
sele c ao de p objetos distintos dentre estes que diferem entre si pela ordem
de coloca c ao ou pela natureza de cada um, isto e, o que importa e quem
participa ou o lugar que ocupa. Denotaremos por A
p
n
o n umero de arranjos
simples de classe p de n objetos.
Por exemplo, dados os objetos o
1
, o
2
e o
3
todos os arranjos possveis de
classe 2 sao: A
1
= (o
1
, o
2
), A
2
= (o
2
, o
1
), A
3
= (o
1
, o
3
), A
4
= (o
3
, o
1
),
A
5
= (o
2
, o
3
) e A
6
= (o
3
, o
2
).
Observa cao 3.24. Notemos que um arranjo simples de classe n de n
objetos dados nao e mais que uma permuta c ao desses n objetos. Logo,
P
n
= A
n
n
= n!.
Proposi cao 3.25. Seja n 1. O n umero total de arranjos simples de
classe p de n objetos O = {o
1
, o
2
, . . . , o
n
} e dado por A
p
n
=
n!
(np)!
.
Demonstrac ao. Para n = 1 a formula e obviamente valida. Similarmente
ao caso das permuta c oes, primeiramente provaremos que para n 2 vale a
seguinte igualdade:
A
p
n
= nA
p1
n1
. (3.6)
Agora denimos os conjuntos A
i
como sendo os arranjos simples de classe
p 1 dos n 1 objetos {o
1
, . . . , o
i1
, o
i+1
, . . . , o
n
}. Note que |A
i
| = A
p1
n1
,
para cada i = 1, 2, . . . , n. Assim, para obtermos um arranjo simples de
classe p dos n objetos, basta que xemos o objeto inicial o
i
e tomemos um
LivroOlimpiadaOcial
2006/5/27
page 88
i
i
i
i
i
i
i
i
88 [CAP. 3: CONTAGEM
elemento do conjunto A
i
, que e uma arranjo de classe p1 dos n1 objetos
restantes. Pelo princpio aditivo, temos que:
A
p
n
= |A
1
| +|A
2
| + +|A
n
| = nA
p1
n1
.
Como nossa equa c ao (3.6) e valida para todo n 2, podemos aplica-la para
n 1, obtendo:
A
p1
n1
= (n 1)A
p2
n2
,
de onde vem que
A
p
n
= n(n 1)A
p2
n2
.
Repetindo este argumento sucessivamente, obtemos que
A
p
n
= n(n 1)(n 2) (n (p 2))A
p(p1)
n(p1)
= n(n 1)(n 2) (n p + 2)A
1
np+1
.
Notemos agora que A
1
np+1
= n p + 1; logo, da igualdade acima segue-se
que
A
p
n
= n(n 1)(n 2) (n p + 2)(n p + 1)
=
n(n 1)(n 2) (n p + 2)(n p + 1) (n p) 1
(n p) 1
=
n!
(n p)!
,
como desejavamos.
Agora vamos dar alguns exemplos de como aparecem problemas praticos
que requerem fazer este tipo de calculo. O primeiro dele tem a ver com a
forma c ao de palavras diferentes com um conjunto dado de leras.
Um anagrama de uma palavra e uma transposi c ao de letras dessa palavra
para formar outra palavra. Por exemplo:
Um anagrama de amor e Roma;
Um anagrama de Celia e Alice;
Um anagrama de Caterina e Natercia;
LivroOlimpiadaOcial
2006/5/27
page 89
i
i
i
i
i
i
i
i
[SEC. 3.5: ARRANJOS SIMPLES 89
Um anagrama de Elvis e Lives.
Exemplo 3.26. Quantos anagramas de p letras distintas podemos formar
com um alfabeto de 23 letras, sendo p < 23 ?
Soluc ao. Claramente nosso problema consiste em achar todos os arranjos
de classe p de 23 objetos dados, que neste caso sao as 23 letras do alfabeto.
Logo, este n umero e
A
k
23
=
23!
(23 k)!
.
Exemplo 3.27. De quantos modos 2 pessoas podem se sentar em 5 cadeiras
que estao em la?
Soluc ao. Este problema e equivalente a achar o n umero total de arranjos de
classe 2 de 5 objetos, correspondendo as 5 cadeiras aos 5 objetos e as duas
pessoas indicando a ordem do arranjo. Logo, este n umero e dado por
A
2
5
=
5!
3!
= 20.
A Tabela 3.1 mostra como fazer todas as distribui c oes possveis. Na
mesma, p
1
e p
2
denotam as duas pessoas e c
1
, c
2
, , c
5
as 5 cadeiras em
la. Observe que a tabela e dividida em 5 grupos de 4 las cada um,
correspondendo o i-esimo deles (1 i 5) a todos os arranjos possveis
quando a pessoa p
1
se senta na cadeira c
i
.
Exemplo 3.28. Considere os dgitos 2, 3, 4, 5, 7 e 9. Supondo que a repeti c ao
de dgitos nao seja permitida, responda as seguintes perguntas:
(a) Quantos n umeros de tres dgitos podem ser formados ?
(b) Dentre os achados em (a) quantos sao pares ?
(c) Dentre os achados em (a) quantos sao mpares ?
(d) Dentre os achados em (a) quantos sao m ultiplos de 5 ?
(e) Dentre os achados em (a) quantos sao menores do que 400 ?
LivroOlimpiadaOcial
2006/5/27
page 90
i
i
i
i
i
i
i
i
90 [CAP. 3: CONTAGEM
c
1
c
2
c
3
c
4
c
5
p
1
p
2
p
1
p
2
p
1
p
2
p
1
p
2
p
2
p
1
p
1
p
2
p
1
p
2
p
1
p
2
p
2
p
1
p
2
p
1
p
1
p
2
p
1
p
2
p
2
p
1
p
2
p
1
p
2
p
1
p
1
p
2
p
2
p
1
p
2
p
1
p
2
p
1
p
2
p
1
Tabela 3.1: Cada linha da tabela indica uma forma diferente de distribuir as
duas pessoas nas 5 cadeiras.
Soluc ao. Seja O = {2, 3, 4, 5, 7, 9} nosso conjunto de objetos
(a) A quantidade de n umeros de tres dgitos que podemos formar sem
repeti c ao de algum deles e claramente o n umero de arranjos de classe
3 dos 6 dgitos de O, isto e,
A
3
6
=
6!
3!
= 120.
(b) Sabemos que em todo n umero par o ultimo dgito e um m ultiplo de
2, isto e, ele acaba em 0, 2, 4, 6 ou 8. Entao, em nosso caso as unicas
possibilidades sao que o n umero termine em 2 ou 4. Supondo que
LivroOlimpiadaOcial
2006/5/27
page 91
i
i
i
i
i
i
i
i
[SEC. 3.6: COMBINAC

OES SIMPLES 91
o ultimo dgito seja 2, temos que preencher as duas casas restantes
com os dgitos pertencentes ao conjunto O {2}. Assim, existem
A
2
|O{2}|
= A
2
5
=
5!
3!
= 20 n umeros dos achados em (a) que nalizam
em 2. De forma analoga, existem A
2
|O{4}|
= A
2
5
=
5!
3!
= 20 n umeros
dos achados em (a) que nalizam em 4. Logo, dentre os n umeros
achados em (a) existem 20 + 20 = 40 n umeros pares.
(c) Todo conjunto de n umeros pode ser dividido em duas classes disjun-
tas: a classe dos n umeros pares e a classe dos n umeros mpares que
pertencem ao mesmo. Segue-se que dentre os n umeros achados em (a)
existem 120 40 = 80 n umeros mpares.
(d) Todo n umero m ultiplo de 5 acaba em 0 ou 5; no nosso caso temos
que a unica possibilidade para o ultimo dgito e 5. Assim o problema
consiste em preencher as duas casas restantes com dgitos do conjunto
O{5}. De onde se segue que a quantidade de n umeros m ultiplos de
5 existentes em (a) vem dada por A
2
|O{5}|
= A
2
5
=
5!
3!
= 20.
(e) Para obter os n umeros menores do que 400 a casa das centenas so
podera ser ocupada pelos dgitos 1, 2 ou 3. Como 1 / O, temos que
as unicas possibilidades em nosso caso sao 2 ou 3. Entao, supondo
que o primeiro dgito do n umero seja 2, devemos preencher duas casas
restantes com os dgitos pertencentes a O {2}. De forma analoga,
existem A
2
|O{3}|
= A
2
5
=
5!
3!
= 20 n umeros dos achados em (a) e que
come cam com 3. Logo, dentre os n umeros achados em (a) existem
20 + 20 = 40 menores do que 400.
3.6 Combina coes Simples
O conceito de combina c ao simples surge naturalmente quando intentamos
responder `a seguinte pergunta: de quantas formas diferentes podemos sele-
cionar p objetos dentro de n objetos dados?
Por exemplo, suponha que queremos enfeitar uma festa de aniversario
com bolas de dois tipos de cores e na loja onde as compraremos existem bolas
nas cores azul, verde e vermelha. De quantas formas distintas podemos
enfeitar nossa festa ?

E claro que podemos enfeitar a festa de 3 formas
LivroOlimpiadaOcial
2006/5/27
page 92
i
i
i
i
i
i
i
i
92 [CAP. 3: CONTAGEM
diferentes: com bolas em azul e verde; com bolas em azul e vermelho ou
com bolas em verde e vermelho.
Notemos que, ao contrario do caso em que trabalhamos com arranjos,
quando fazemos uma sele c ao de duas cores nao estamos interessados na
ordem em que elas foram escolhidas.
Deni cao 3.29. Consideremos n objetos e p um inteiro positivo tal que
0 < p n. Uma combinac ao simples de classe p dos n objetos dados e uma
sele c ao de p objetos distintos dentre estes que diferem entre si apenas pela
natureza de cada um, isto e, o que importa e simplesmente quem partic-
ipa no grupo selecionado. Denotaremos por
_
n
p
_
o n umero de combina c oes
simples de classe p de n objetos.
Proposi cao 3.30. Seja n 1. O n umero total de combinac oes simples de
classe p de n objetos O = {o
1
, o
2
, . . . , o
n
} e dado por
_
n
p
_
=
n!
p!(np)!
.
Demonstrac ao. Veremos a seguir que arranjos simples e combina c oes sim-
ples de classe p estao estreitamente relacionados. Com efeito, para cada
combina c ao simples formada por p objetos distintos de O podemos gerar to-
dos os arranjos simples de classe p formados por estes p objetos. Basta para
isto fazer todas as suas permuta c oes possveis. Obtem-se assim p ! arranjos
simples diferentes com esses p objetos. Resumindo, para cada combina c ao
simples de classe p formada com p objetos diferentes de O podemos fazer
p ! arranjos simples diferentes de classe p com estes mesmos objetos; logo,
no total, teremos a seguinte rela c ao:
p !
_
n
p
_
= A
p
n
=
n!
(n p)!
,
de on segue-se que
_
n
p
_
=
n!
p!(n p)!
.
Exemplo 3.31. De quantas formas podemos dividir um grupo 5 pessoas
em um grupo de duas e outro de tres ?
Soluc ao. Temos
_
5
2
_
=
5 !
2 !3 !
= 10 formas diferentes de escolher duas pessoas
do grupo. Por cada uma dessas escolhas o outro grupo de tres pessoas e
automaticamente determinado; logo, temos 10 possibilidades diferentes de
fazer a divisao.
LivroOlimpiadaOcial
2006/5/27
page 93
i
i
i
i
i
i
i
i
[SEC. 3.6: COMBINAC

OES SIMPLES 93
Exemplo 3.32. De quantos modos podemos dividir 6 pessoas em:
(a) dois grupos de 3 pessoas cada?
(b) tres grupos de 2 pessoas cada?
Soluc ao. Come camos por (a).
`
A primeira vista, parece que a resposta deve
ser
_
n
3
_
=
6!
3! 3!
= 20, similarmente ao exemplo anterior. Porem, aqui ha
um problema pois, devido ao fato de estarmos dividindo em grupos que
tem a mesma quantidade de pessoas, entao as permuta c oes de cada dois
grupos formados sao consideradas divisoes iguais; logo, devemos dividir o
resultado por 2 !, obtendo assim 10 formas diferentes de obter dois grupos
com 3 pessoas cada.
Para resolver o item (b) seguimos os seguintes passos:
Primeiramente calcularemos o n umero de formas possveis para dividir
6 pessoas em um grupo de 2 e outro grupo de 4; esta quantidade vem
dada por
_
6
2
_
=
6!
4! 2!
.
Agora dividiremos as 4 pessoas restantes em um grupo de 2 e outro
grupo de 2; esta quantidade vem dada por
_
4
2
_
=
4!
2! 2!
.
Pelo princpio multiplicativo temos que existem
_
6
2
__
4
2
_
=
6!
(2!)
3
possibilidades
de dividir 6 pessoas em 3 grupos com duas pessoas cada. Igualmente ao
caso anterior, aqui as permuta c oes possveis de cada 3 grupos formados
sao consideradas iguais; logo, devemos dividir este ultimo resultado por 3 !.
Portanto, existem 15 formas diferentes de dividir 6 pessoas em tres grupos
de 2 pessoas cada.
Exemplo 3.33. Se voce possui 10 amigos, de quantas maneiras voce pode
jantar com 2 ou mais deles?
Soluc ao. Esquematizamos a solu c ao da seguinte maneira:
Primeiramente, vamos encontrar a quantidade de maneiras pelas quais
voce pode jantar com 2 amigos; isto e feito de
_
10
2
_
formas diferentes.
Depois, vamos encontrar a quantidade de maneiras pelas quais voce
pode jantar com 3 amigos; isto e feito de
_
10
3
_
formas diferentes.
Em seguida, encontramos a quantidade de maneiras pelas quais voce
pode jantar com 4 amigos; isto e feito de
_
10
4
_
formas diferentes.
LivroOlimpiadaOcial
2006/5/27
page 94
i
i
i
i
i
i
i
i
94 [CAP. 3: CONTAGEM
Em geral, o n umero de maneiras diferentes que voce tem de jantar
com p amigos e dado por
_
10
p
_
.
Pelo Princpio Aditivo, temos que a quantidade de formas diferentes que
voce tem de jantar com 2 ou mais de seus amigos, e dada por:
_
10
2
_
+
_
10
3
_
+ +
_
10
9
_
+
_
10
10
_
= 1013.
Exemplo 3.34. De um grupo de 10 pessoas das quais 4 sao mulheres,
quantas comissoes de 5 pessoas podem ser formadas de modo que pelo menos
uma mulher fa ca parte?
Soluc ao. Sendo que o grupo tem 10 pessoas e 4 destas sao mulheres, segue-
se que no grupo temos 6 homens. Para formar um grupo de 5 pessoas com
pelo menos uma mulher, temos as seguintes alternativas:
Nosso grupo e composto por uma mulher e 4 homens; neste caso poder-
emos formar
_
4
1
__
6
4
_
= 60 comissoes de 5 pessoas.
Nosso grupo e composto por 2 mulheres e 3 homens; neste caso poder-
emos formar
_
4
2
__
6
3
_
= 120 comissoes de 5 pessoas.
Nosso grupo e composto por 3 mulheres e 2 homens; neste caso poder-
emos formar
_
4
3
__
6
2
_
= 60 comissoes de 5 pessoas.
Nosso grupo e composto por 4 mulheres e um homem; neste caso
poderemos formar
_
4
4
__
6
1
_
= 6 comissoes de 5 pessoas.
Pelo princpio aditivo temos que e possvel formar 246 comissoes de 5 pessoas
de modo que pelo menos uma mulher fa ca parte.
3.7 Contagem e Probabilidades
Uma das aplica c oes interessantes da contagem de elementos de um conjunto
e quando desejamos estudar a probabilidade de eventos aleatorios. Por
exemplo, se lan carmos um dado de seis faces, temos os seguintes resultados
possveis:
= {1, 2, . . . , 6}.
LivroOlimpiadaOcial
2006/5/27
page 95
i
i
i
i
i
i
i
i
[SEC. 3.7: CONTAGEM E PROBABILIDADES 95
Se desejamos saber qual e a chance de que ocorra um n umero primo no
lan camento, devemos contar quantos primos aparecem em {1, 2, 3, 4, 5, 6}
e dividir por 6. Ou seja, a chance de ocorrer um n umero primo num
lan camento de um dado de seis faces e 3/6 = 0, 5.
Em geral, denimos entao a probabilidade de um subconjunto A
como
p(A) =
|A|
||
.
Tambem chamamos o subconjunto de todos os resultados possveis de
espaco amostral e um subconjunto A de de evento. Por exemplo, pode-
mos calcular a probabilidade de escolhermos um n umero par no conjunto
1, 2, 3, . . . , 15. Neste caso, o conjunto esta claro e e igual a = {1, 2, 3, . . . , 15}.
O conjunto A e A = {2, 4, 6, . . . , 14}. Logo,
p(A) =
|A|
||
=
7
15
.
Assim, ca claro que a maior diculdade para calcular a probabilidade
de um evento e contar quantos elementos pertencem a este evento e quantos
elementos pertencem ao espa co amostral. A seguir, veremos um exemplo
mais elaborado onde aplicamos a no c ao de combina c oes simples.
Exemplo 3.35. Vamos agora calcular a probabilidade de que escolhendo
um grupo de 44 pessoas, existam pelo menos duas que fazem aniversario no
mesmo dia do ano.
Podemos reescrever isso do seguinte modo: num saco existem bolas enu-
meradas com os n umeros 1, 2, . . . , 365 (correspondentes aos dias do ano).
Retiramos a bola b
1
e anotamos o n umero que apareceu. Devolvemos a
bola ao saco e efetuamos uma nova retirada, anotando novamente o n umero
que aparece. Repetindo este processo 44 vezes, obtemos uma lista com 44
n umeros. Assim, a pergunta se transforma em : de quantos modos diferentes
podemos escolher 44 bolas enumeradas com os n umeros 1, 2, 3, . . . , 365 com
reposi c ao, tal que existam pelo menos duas bolas com o mesmo n umero?
A primeira coisa que devemos fazer e calcular o espa co amostral, de
todas as possibilidades possveis de resultado. Como escolhemos 44 bolas
enumeradas num saco, cada resultado possvel e uma lista (n
1
, n
2
, . . . , n
44
)
com 44 n umeros. Observe que, pelo princpio multiplicativo, || = 365
44
,
pois temos 365 op c oes para escolher n
1
, 365 op c oes para escolher n
2
, etc.
LivroOlimpiadaOcial
2006/5/27
page 96
i
i
i
i
i
i
i
i
96 [CAP. 3: CONTAGEM
A segunda pergunta trata-se de saber quantos resultados sao favoraveis,
ou seja, quantas sao as escolhas tais que existam pelo menos duas bolas com
o mesmo n umero. Para isso e mais facil contar quantas escolhas existem tais
que os 44 n umeros sao diferentes. Neste caso, devemos escolher 44 n umeros
distintos entre 365. Como nao importa a ordem em que eles aparecem,
sao ao todo
_
365
44
_
grupos de 44 n umeros. Logo, o n umero de resultados
favoraveis, isto e, onde existem pelo menos duas pessoas que nasceram no
mesmo dia e
_
365
44
_
de onde conclumos que a probabilidade de que este
evento ocorra e
p =
365
44

_
365
44
_
365
44
= 1
_
365
44
_
365
44
.
Com ajuda de uma formula matematica chamada formula de Stirling e de uma
calculadora, obtemos que p e aproximadamente p

= 0.80, como havamos prometido
no Captulo 1.
3.8 Exerccios Propostos
1. De quantas maneiras podemos escolher tres n umeros distintos do con-
junto I
50
= {1, 2, 3, . . . , 49, 50} de modo que sua soma seja
a) um m ultiplo de 3?
b) um n umero par?
2. Considere o conjunto I
n
= {1, 2, 3, . . . , n 1, n}. Diga de quantos
modos e possvel formar subconjuntos de k elementos nos quais nao
haja n umeros consecutivos?
3. Considere as letras da palavra PERMUTA. Quantos anagramas de 4
letras podem ser formados, onde:
a) nao ha restri c oes quanto ao n umero de consoantes ou vogais?
b) o anagrama come ca e termina por vogal?
c) a letra R aparece?
d) a letra T aparece e o anagrama termina por vogal?
4. Calcular a soma de todos os n umeros de 5 algarismos distintos forma-
dos com os algarismos 1, 3, 5, 7 e 9.
5. Quantos n umeros podem ser formados pela multiplica c ao de alguns
ou de todos os n umeros 2, 2, 3, 3, 3, 5, 5, 6, 8, 9, 9?
LivroOlimpiadaOcial
2006/5/27
page 97
i
i
i
i
i
i
i
i
[SEC. 3.8: EXERC

ICIOS PROPOSTOS 97
6. Dentre todos os n umeros de sete dgitos, diga quantos possuem exata-
mente tres dgitos 9 e os quatro dgitos restantes todos diferentes?
7. De quantas maneiras podemos distribuir 22 livros diferentes entre 5
alunos se 2 deles recebem 5 livros cada e os outros 3 recebem 4 livros
cada?
8. Quantos sao os n umeros naturais de sete dgitos nos quais o dgito 4
gura exatamente 3 vezes e o dgito 8 gura exatamente 2 vezes?
9. De quantas maneiras uma comissao de 4 pessoas pode ser formada,
de um grupo de 6 homens e 6 mulheres, se a mesma e composta de
um n umero maior de homens do que de mulheres?
10. O comprimento de uma palavra e a quantidade de caracteres que ela
possui. Encontre a quantidade de palavras de comprimento 5 que
podemos formar fazendo uso de 10 caracteres distintos, de forma que
nao existam tres caracteres consecutivos identicos em cada palavra.
11. Quantos n umeros inteiros existem entre 1 e 10.000 que nao sao di-
visveis por 3, 5 e 7?
12. Quantas sao as permuta c oes da palavra PROPOR nas quais nao ex-
istem letras consecutivas iguais?
13. De quantos modos 6 casais podem sentar-se ao redor de uma mesa
circular de tal forma que marido e mulher nao quem juntos?
14. Quantas sao as permuta c oes das letras da palavra BRASIL em que o
B ocupa o primeiro lugar, ou o R ocupa o segundo lugar, ou o L o
sexto lugar.
15. De quantas formas podemos representar o n umero 15 como soma de
varios n umeros naturais?
16. Quantos quadrados perfeitos existem entre 40.000 e 640.000 que sao
m ultiplos simultaneamente de 3,4 e 5?
17. Oito amigos vao para o cinema assistir a um lme que custa um real.
Quatro deles possuem uma nota de um real e quatro possuem uma
nota de dois reais. Sabendo-se que o caixa do cinema nao possui
nenhum dinheiro, como eles podem organizar uma la para pagar o
lme permitindo o troco pelo caixa?
LivroOlimpiadaOcial
2006/5/27
page 98
i
i
i
i
i
i
i
i
98 [CAP. 3: CONTAGEM
18. Se considerarmos todas as congura c oes do tabuleiro com duas torres
que nao se atacam, como no Exemplo 3.2, sem distinguir as torres,
quantas congura c oes obteremos?
19. Continuando o problema anterior, generalize-o para 3, 4, 5, . . . torres
que nao se atacam, encontrando tambem o n umero maximo de torres
que podem ser colocadas no tabuleiro de modo que duas delas nao se
ataquem.
20. Tente fazer o problema anterior para cavalos de xadrez.
LivroOlimpiadaOcial
2006/5/27
page 99
i
i
i
i
i
i
i
i
Captulo 4
O Princpio da Casa dos
Pombos
Um importante instrumento para tratar problemas matematicos relaciona-
dos `a existencia de elementos de conjuntos validando certas exigencias e o
chamado princpio de Dirichlet, tambem conhecido como Princpio da Casa
dos Pombos (PCP). Este princpio foi usado por Dirichlet (1805-1859) para
resolver problemas na teoria dos n umeros, entretanto ele possui um grande
n umero de aplica c oes em diversos ramos da matematica como combinatoria
e geometria.
A versao mais simples do Princpio da Casa dos Pombos arma o seguinte:
Se distribumos N + 1 pombos em N casas, ent ao alguma das
casas contem dois ou mais pombos.
A prova deste princpio e muito facil e decorre de fazer uma simples
contagem dos pombos contidos em todas as casas depois de distribudos.
Com efeito, suponhamos pelo contrario que em cada casa nao existe mais do
que um pombo, entao contando todos os pombos contidos nas N casas nao
teremos mais do que N pombos, contradizendo isto a hipoteses de termos
N + 1 pombos distribudos nas N casas (ver Figura 4.1)
Nao e difcil detectar quando o princpio pode ser usado, mas a principal
diculdade para aplicar o mesmo reside em identicar, em cada problema,
quem faz papel de pombos e quem faz papel de casas.
99
LivroOlimpiadaOcial
2006/5/27
page 100
i
i
i
i
i
i
i
i
100 [CAP. 4: O PRINC

IPIO DA CASA DOS POMBOS


P
1
P
2
P
N
C
1
C
2
C
N
P
N+1
Figura 4.1: Em cada casa Cj, 1 j N, coloca-se um unico pombo, denotado
por Pj. O pombo restante, denotado por PN+1, deve ir para alguma das casas,
juntando-se ao que ja se encontrava contido nela.
Nas seguintes se c oes discutiremos varios exemplos de diferentes naturezas,
onde o Princpio da Casa dos Pombos e aplicado com sucesso.
4.1 Primeiros Exemplos
Exemplo 4.1. Numa oresta crescem 1000 jaqueiras.

E conhecido que uma
jaqueira nao contem mais do que 600 frutos. Prove que existem 2 jaqueiras
na oresta que tem a mesma quantidade de frutos.
Soluc ao. Temos 1000 jaqueiras, representando os pombos, e 601 casas iden-
ticadas pelos n umeros 0, 1, 2, 3, , 600. O n umero k associado a cada casa
signica que nela serao colocadas jaqueiras que tem exatamente k frutos.
Como 1000 > 602 = 601+1, o PCP nos garante que existem duas jaqueiras
com a mesma quantidade de frutos.
Exemplo 4.2. Em uma reuniao ha n pessoas. Mostre que existem duas
pessoas que conhecem exatamente o mesmo n umero de pessoas.
LivroOlimpiadaOcial
2006/5/27
page 101
i
i
i
i
i
i
i
i
[SEC. 4.1: PRIMEIROS EXEMPLOS 101
Soluc ao. Os pombos neste caso sao as n pessoas. As casas sao enumeradas
com os n umeros 0, 1, 2, , n1, indicando estes que na mesma serao colo-
cadas pessoas que tem essa quantidade de conhecidos. Notemos que uma
das casas enumeradas com 0 ou n 1 permanece desocupada, pois a possi-
bilidade de conhecer 0 e n1 pessoas nao acontece simultaneamente. Logo,
nas n 1 casas restantes havera uma ocupada por dois ou mais pombos,
depois de serem distribudos. Portanto, existem no mnimo duas pessoas
com o mesmo n umero de conhecidos.
Exemplo 4.3. Dados 8 n umeros inteiros mostre que existem dois deles cuja
diferen ca e divisvel por 7.
Soluc ao. Consideramos os 8 n umeros como sendo os pombos e as casas como
sendo os 7 possveis restos na divisao por 7. Como temos 8=7+1 n umeros
o PCP nos diz que existem dois n umeros dentro dos 8 dados que tem o
mesmo resto quando divididos por 7. Finalmente, observamos que se dois
n umeros deixam o mesmo resto na divisao por 7 entao a diferen ca entre eles
e divisvel por 7.
Uma forma alternativa e muito util na qual pode-se apresentar o Princpio
da Casa dos Pombos e a seguinte:
Se a soma de n n umeros naturais e igual S, ent ao existe pelo
menos um deles que n ao e maior que S/n, assim como existe
pelo menos um deles que n ao e menor que S/n.
Exemplo 4.4. Numa familia formada por 5 pessoas a soma das idades e
de 245 anos. Prove que podem ser selecionados 3 membros da familia cuja
soma das idades nao e menor que 147.
Soluc ao. Temos um total de
_
5
3
_
=
5!
3!2!
= 10 trios diferentes formados por
membros da familia. Alem disso, cada pessoa aparece exatamente em
_
4
2
_
=
4!
2!2!
= 6 trios. Entao, denotando por E
j
a soma das idades dos membros de
cada trio T
j
, j = 1, 2 10, temos que
E
1
+ E
2
+ + E
10
= 6 245 = 1470;
conseq uentemente existe algum trio T
j
tal que E
j

1470
10
= 147.
LivroOlimpiadaOcial
2006/5/27
page 102
i
i
i
i
i
i
i
i
102 [CAP. 4: O PRINC

IPIO DA CASA DOS POMBOS


4.2 Uma Versao mais Geral
A seguinte versao mais geral do PCP e bastante util na resolu c ao de alguns
problemas. Ela arma o seguinte:
Se distribumos Nk + 1 pombos em N casas, ent ao alguma das
casas contem pelo menos k + 1 pombos.
A prova deste enunciado mais geral e similar `a anterior. Com efeito,
suponhamos pelo contrario que em cada casa nao existe mais do que k
pombos, entao contando todos os pombos contidos nas N casas nao teremos
mais do que Nk pombos, contradizendo isto a hipoteses de termos Nk + 1
pombos distribudos nas N casas.
Notemos que se k = 1 esta versao mais geral coincide com a versao mais
simples.
Exemplo 4.5. Num colegio com 16 salas sao distribudas canetas nas cores
preta, azul e vermelha para realizar uma prova de concurso. Se cada sala
recebe canetas da mesma cor entao prove que existem pelo menos 6 salas
que receberam canetas da mesma cor.
Soluc ao. Fazendo a divisao com resto de 16 por 3 temos que 16 = 3 5 +1.
Consideramos as 16 salas como sendo os pombos e as tres cores, preto, azul
e vermelho como sendo as casas. Logo, podemos colocarcada sala em
uma das tres cores. Assim, o PCP com N = 3 e k = 5 nos da que existe
uma casa com pelo menos 6 pombos, ou seja, existem no mnimo 6 salas
que receberam canetas da mesma cor.
Exemplo 4.6. Uma equipe formada por seis alunos de matematica e se-
lecionada para representar o Brasil numa olimpada internacional. Mostre
que necessariamente existem tres deles que se conhecem mutuamente, ou
tres deles que nao se conhecem mutuamente.
Soluc ao. Resolveremos o problema com o auxlio da Figura 4.2. Cada aluno
A
j
, com j = 1, 2, , 6, e representado por um dos vertices de um hexagono
regular. Quando dois alunos se conhecem tra camos o segmento de reta que
liga os vertices correspondentes na cor azul; caso contrario tra camos este
segmento na cor vermelha. Logo, usando este esquema, o problema equivale
a provar que sempre existe um tri angulo azul ou um tri angulo vermelho com
vertices no conjunto A = {A
1
, A
2
, , A
6
}.
LivroOlimpiadaOcial
2006/5/27
page 103
i
i
i
i
i
i
i
i
[SEC. 4.3: APLICAC

OES NA TEORIA DOS N

UMEROS 103
Temos 5 segmentos (pombos) incidindo no vertice A
1
, cada um deles
pintado de azul ou de vermelho (estas duas cores sao consideradas como as
casas). Como 5 = 2 2 + 1, pelo PCP temos que 3 dos 5 segmentos sao
azuis ou vermelhos. Suponhamos que 3 sao azuis (caso contrario o argu-
mento e similar) e denotemos estes por A
1
A
3
, A
1
A
4
e A
1
A
6
(ver Figura
4.2). Se algum dos segmentos A
3
A
4
, A
3
A
6
ou A
4
A
6
for azul entao este seg-
mento junto aos que se ligam com A
1
formam um tri angulo azul. Por outro
lado, se nenhum deles for azul, entao eles formam um tri angulo vermelho,
completando isto a demonstra c ao.
A
1
A
2
A
3
A
4
A
5
A
6
Figura 4.2: O triangulo A1A2A5 indica que os alunos A1, A2 e A5 nao se con-
hecem mutuamente e o triangulo A1A4A6 indica que os alunos A1, A4 e A6 se
conhecem mutuamente.
4.3 Aplicacoes na Teoria dos N umeros
Nesta se c ao apresentamos alguns exemplos de aplica c oes do PCP na Teoria
dos N umeros. A primeira delas e:
Exemplo 4.7. Se n e m sao n umeros naturais, entao o conjunto A =
{m + 1, m + 2, . . . , m + n} possui algum divisor de n.
Soluc ao. Temos n n umeros diferentes no conjunto acima. Vamos utilizar
o metodo de redu c ao ao absurdo. Se nao existisse nenhum m ultiplo de n,
quando dividssemos os n umeros do conjunto A por n, os restos pertence-
riam ao conjunto B = {1, 2, . . . , n 1}, que possui n 1 elementos. Logo,
LivroOlimpiadaOcial
2006/5/27
page 104
i
i
i
i
i
i
i
i
104 [CAP. 4: O PRINC

IPIO DA CASA DOS POMBOS


devem existir dois n umeros m+i e m+j, com 1 i < j n tais que o resto
da divisao de m+i por n e o mesmo que o resto da divisao de m+j por n.
Logo, m+j (m+i) e um m ultiplo de n, o que implica que n > j i 1
e m ultiplo de n menor que n (absurdo!). Logo, deve existir algum m ultiplo
de n no conjunto A.
Como conseq uencia desse exemplo, podemos resolver o seguinte prob-
lema:
Exemplo 4.8. Demonstrar que todo inteiro tem um m ultiplo cuja repre-
senta c ao decimal come ca com o bloco de dgitos 1234567890.
Soluc ao. Se m e n sao inteiros positivos, pelo exemplo anterior um dos
n umero m+1, m+2, . . . , m+n e m ultiplo de n. Assim, Dado n um inteiro
qualquer, escolhe-se m = 123456789010
n+1
. Deste modo, todos os inteiros
m+1, m+2, . . . , m+n come cam com 1234567890 e algum deles e m ultiplo
de n.
Exemplo 4.9. Dado um n umero inteiro positivo n, mostre que existe um
m ultiplo de n que se escreve com os algarismos 0 e 1 apenas. (Por exemplo,
se n = 3, temos 111 ou 1101, etc...)
Soluc ao. Consideramos os n + 1 n umeros
1, 11, 111, 1111, , 111 1
. .
n+1vezes
, (4.1)
como sendo os pombos e n casas enumeradas com os n umeros
0, 1, 2, 3, , n 1,
ou seja, com os possveis restos na divisao por n. Similarmente ao exemplo
anterior existem dois n umeros na lista (4.1) que deixam o mesmo resto na
divisao por n e, portanto, a diferen ca entre o maior e o menor e m ultiplo
de n. Obviamente a diferen ca entre dois n umeros quaisquer da lista (4.1)
resulta em um n umero formado apenas pelos algarismos 0 e 1.
Exemplo 4.10. Prove que entre n + 1 elementos escolhidos no conjunto
{1, 2, 3, 2n} existem dois que sao primos relativos.
LivroOlimpiadaOcial
2006/5/27
page 105
i
i
i
i
i
i
i
i
[SEC. 4.4: APLICAC

OES GEOM

ETRICAS 105
Soluc ao. A escolha das casas e dos pombos neste exemplo nao e tao obvia.
Os pombos representam os n+1 n umeros escolhidos do conjunto {1, 2, 2n}
e as casas sao escolhidas como sendo os n conjuntos:
C
j
= {2j 1, 2j}, 1 j n.
Logo, pelo PCP, quando distribumos os n + 1 n umeros nos n conjuntos
C
j
, 1 j n, dois deles car ao juntos em algum conjunto C
j
, ou seja, estes
n umeros serao consecutivos e portanto primos entre si.
Finalizaremos esta se c ao com uma outra prova do Teorema de Bezout:
Exemplo 4.11. Seja d = (a, b) o m.d.c. entre os n umeros naturais a e b.
Entao, existem x e y n umeros inteiros tais que
ax + by = d.
Soluc ao. Denotando por m = a/d e n = b/d, podemos supor que a e b sao
primos entre si. Realmente, se podemos escrever
mx + ny = 1
entao, substituindo os valores de m e n na equa c ao acima, temos que ax +
by = d.
Se (a, b) = 1, considere a sequencia A = {a, 2a, . . . , ba}. Armamos que
existe algum n umero no conjunto A que deixa resto 1 quando dividido por
b. De fato, se isso nao ocorresse, teramos b n umeros em A deixando no
maximo b 1 restos diferentes quando divididos por b. Logo, pelo PCP,
dois deles, digamos ia e ja com b > j > i 1, devem deixar o mesmo resto
quando divididos por b. assim, (j i)a e divisvel por b. Como estamos
supondo que (a, b) = 1, temos que b deve dividir j i > 0. Como b > j i,
temos um absurdo.
Assim, algum dos n umeros em a deixa resto 1 quando divididos por
b. Digamos que esse n umero seja ax. Logo, ax 1 e m ultiplo de b, onde
ax 1 = by, o que encerra nossa prova.
4.4 Aplicacoes Geometricas
Na geometria tambem encontramos belas aplica c oes do PCP. Vejamos os
problemas a seguir para constatar isto.
LivroOlimpiadaOcial
2006/5/27
page 106
i
i
i
i
i
i
i
i
106 [CAP. 4: O PRINC

IPIO DA CASA DOS POMBOS


Exemplo 4.12. Mostre que se tomamos cinco pontos quaisquer sobre um
quadrado de lado 1, entao pelo menos dois deles distam menos que

2/2.
Soluc ao. Vamos dividir o quadrado em quatro quadradinhos de lado 1/2,
como mostra a Figura 4.3
1

Figura 4.3::
Logo, pelo PCP pelo menos dois deles devem estar no mesmo quadrad-
inho, uma vez que temos 4 quadradinhos e 5 pontos. Logo, como a maior
distancia num quadrado e a diagonal, o teorema de pitagoras nos garante
que a distancia desses dois pontos e no maximo

2/2, como queramos


mostrar.
Exemplo 4.13. Qual e o maior n umero de quadradinhos de um tabuleiro
de 88 que podem ser pintados de preto, de forma tal que qualquer arranjo
de tres quadradinhos como mostra a Figura 4.4 tenha pelo menos um dos
quadradinhos nao pintado de preto?
Figura 4.4::
Soluc ao. Primeiramente, pintamos o tabuleiro de 88 como um tabuleiro de
jogar xadrez, ou seja, 32 quadradinhos pintados de branco e 32 quadradinhos
pintados de preto (ver Figura 4.5).
Notemos que uma vez pintado o tabuleiro desta forma e satisfeita a
exigencia do problema, pois nunca temos 2 quadradinhos vizinhos (quadrad-
inhos com um lado comum) pintados de preto.
LivroOlimpiadaOcial
2006/5/27
page 107
i
i
i
i
i
i
i
i
[SEC. 4.4: APLICAC

OES GEOM

ETRICAS 107
Figura 4.5::
Mostraremos agora que se pintamos 33 quadradinhos de preto entao a
condi c ao exigida no problema falha. De fato, se dividimos o tabuleiro em 16
quadrados de 2 2 (casas) e pintamos 33 quadradinhos de preto (pombos);
entao, como 33 = 16 2 +1, pela versao geral do PCP um dos 16 quadrados
de 2 2 contem 3 quadradinhos pintados de preto. Portanto, este ultimo
contem um arranjo como na Figura 4.5 completamente pintado de preto.
Resumindo, o n umero maximo de quadradinhos que podemos pintar de
preto e 32.
Exemplo 4.14. Na regi ao delimitada por um tri angulo eq uilatero de lado
4 sao marcados 10 pontos. Prove que existe ao menos um par destes pontos
cuja distancia entre eles nao e maior que

3.
Soluc ao. Dividimos o tri angulo equilatero de lado 4 em 16 tri angulos equilateros
menores de lado 1, conforme a Figura 4.6.
Agora pintamos os tri angulos nas cores verde e cinza de maneira que dois
tri angulos vizinhos, isto e, com um lado comum, sao pintados de cores difer-
entes. Se tivessemos dois pontos no mesmo tri angulo a distancia maxima
possvel entre eles seria 1 e o problema estaria resolvido. Se tivessemos
pontos em tri angulos vizinhos, a maior distancia possvel entre eles seria

3 e tambem isto resolveria o problema. Se nao tivessemos nenhum dos


casos anteriores, nao seria difcil ver que os 10 pontos deveriam estar situa-
dos sobre os 10 tri angulos verdes, contendo cada tri angulo exatamente um
ponto. Dividendo o tri angulo DBE em 4 tri angulos iguais de lado 3/2 pelo
LivroOlimpiadaOcial
2006/5/27
page 108
i
i
i
i
i
i
i
i
108 [CAP. 4: O PRINC

IPIO DA CASA DOS POMBOS


A B
C
D
E

Figura 4.6::
PCP temos que pelo menos dois dos 6 pontos contidos em DBE estao num
destes 4 tri angulos, logo a distancia entre eles nao e maior que 3/2 <

3.
Com isto terminamos nossa prova.
4.5 Miscelanea
Exemplo 4.15. Em cada quadradinho de um tabuleiro 3 3 e colocado
um dos n umeros: 1, 0 ou 1. Prove que entre todas as somas das linhas,
colunas e diagonais do tabuleiro ha duas que sao iguais. Por exemplo, no
tabuleiro abaixo a soma da segunda linha e 2, que coincide com a soma da
terceira coluna.
-1 -1 1
1 0 1
0 -1 0
Soluc ao. Seja S = a
1
+ a
2
+ a
3
, onde cada a
1
, a
2
e a
3
podem tomar val-
ores: 1, 0 e 1. Entao, temos 7 valores possveis para S (casas), que sao:
3, 2, 1, 0, 1, 2, 3.
O tabuleiro 3 3 tem 3 linhas, 3 colunas e 2 diagonais, portanto, ao
somarmos os elementos de cada uma das linhas, colunas e diagonais, obter-
emos 8 n umeros (pombos). Como existem somente 7 valores possveis para
estes n umeros, pelo PCP pelo menos dois deles devem ser iguais.
LivroOlimpiadaOcial
2006/5/27
page 109
i
i
i
i
i
i
i
i
[SEC. 4.5: MISCEL

ANEA 109
Exemplo 4.16. Dado qualquer conjunto A formado por 10 n umeros nat-
urais escolhidos entre 1 e 99, inclusos, demonstre que existem dois sub-
conjuntos disjuntos e nao-vazios de A tal que a soma dos seus respectivos
elementos e igual.
Soluc ao:

E conhecido que A tem 2
10
1 = 1023 subconjuntos nao-vazios
diferentes. A soma dos elementos de cada um deles da uma quantidade
menor do que 1000, pois o subconjunto com no maximo 10 elementos de
maior soma possvel e o formado por 90, 91, , 99, e nesse caso 90 +
91 + + 99 = 945. Agora consideramos os pombos como sendo os 1023
subconjuntos distintos de A e as casas como sendo as somas possveis dos
elementos de cada um dos conjuntos. Logo, como o n umero de conjuntos
e maior que o n umero de somas possveis, devem existir dois conjuntos B
e C de A, de tal modo que a soma dos elementos de B e igual `a soma dos
elementos de C. Se B e C sao disjuntos, acabou a prova. Se nao, considere
D = B B C e E = C B C. Logo, os conjuntos D e E sao disjuntos e
a soma dos seus elementos e a mesma, pois retiramos de ambos a mesma
quantidade.
Exemplo 4.17. Dados sete n umeros reais arbitrarios, demonstre que exis-
tem dois deles, digamos x e y, tais que
0
x y
1 + xy

1

3
Soluc ao. Primeiramente observamos que a express ao
xy
1+xy
nos faz pensar
na formula
tan( ) =
tan tan
1 + tan tan
. (4.2)
Sejam x
1
, x
2
, , x
7
os sete n umeros selecionados arbitrariamente. Lem-
bramos que a fun c ao tangente e uma bije c ao entre o intervalo (

2
,

2
) e os
n umeros reais R, logo para cada x
i
, 1 i 7, existe um
i
(

2
,

2
) tal
que tan(
i
) = x
i
. Dividimos o intervalo (

2
,

2
) em seis subintervalos de
comprimento

6
, como mostra o desenho abaixo.

i1

i2

6
LivroOlimpiadaOcial
2006/5/27
page 110
i
i
i
i
i
i
i
i
110 [CAP. 4: O PRINC

IPIO DA CASA DOS POMBOS


Pelo PCP dois dos n umeros
i
pertencem ao mesmo subintervalo. De-
notemos os mesmos por
i1
e
i2
e suponhamos, sem perda de generalidade,
que
i1

i2
. Entao vale
0
i2

i1


6
.
Usando o fato de que a tangente e uma fun c ao crescente e a formula (4.2)
temos que
tan(0) tan(
i2

i1
) tan(

6
).
Equivalentemente,
0
x
i2
x
i1
1 + x
i2
x
i1

3
.
4.6 Exerccios
1. Seja x um n umero real arbitrario. Prove que entre os n umeros
x, 2x, 3x, , 101x
existe um tal que sua diferen ca com certo n umero inteiro e menor
0,011.
2. Mostre que entre nove n umeros que nao possuem divisores primos
maiores que cinco, existem dois cujo produto e um quadrado.
3. Um disco fechado de raio um contem sete pontos, cujas distancias
entre quaisquer dois deles e maior o igual que um. Prove que o centro
do disco e um destes pontos.
4. Na regi ao delimitada por um ret angulo de largura quatro e altura tres
sao marcados seis pontos. Prove que existe ao menos um par destes
pontos cuja distancia entre eles nao e maior que

5.
5. Seja a um n umero irracional. Prove que existem innitos n umeros
racionais r = p/q tais que |a r| < 1/q
2
.
LivroOlimpiadaOcial
2006/5/27
page 111
i
i
i
i
i
i
i
i
Captulo 5
Inducao Matematica
Imagine uma la com innitos dominos, um atras do outro. Suponha que
eles estejam de tal modo distribudos que, uma vez que um domino caia,
o seu sucessor na la tambem caia. O que acontece quando derrubamos o
primeiro domino?
Apesar da simplicidade da pergunta acima ela tras em sua essencia toda
a ideia usada no Metodo da Induc ao Matem atica. Esse metodo e uma das
grandes armas do matematico moderno e tem utilidade na solu c ao de varios
problemas, como iremos ver ao longo deste captulo.
5.1 Formula cao Matematica
No incio do seculo XX o matematico Giuseppe Peano (1858-1932) estabele-
ceu os axiomas necessarios que nos permitem hoje descrever com precis ao
o conjunto dos n umeros naturais denotado por N = {1, 2, 3, }. O ultimo
dos seus axiomas diz o seguinte: Seja A um subconjunto de N (A N). Se
1 A e se, alem disso, A contem todos os sucessores dos seus elementos,
entao A = N.
Este axioma e conhecido como axioma de induc ao e serve como base
do metodo de demonstra c ao por indu c ao matematica, o qual e de grande
utilidade para estabelecer provas rigorosas em matematica.
111
LivroOlimpiadaOcial
2006/5/27
page 112
i
i
i
i
i
i
i
i
112 [CAP. 5: INDUC

AO MATEM

ATICA
5.1.1 Metodo de Inducao Matematica
Neste captulo, nao assumiremos o axioma da indu c ao. Ao inves disso, ire-
mos utilizar o Princpio da Boa Ordena c ao, enunciado no incio do Captulo 2,
para mostrar que o Metodo da Indu c ao Finita e verdadeiro. O motivo de
fazer isto e mostrar ao leitor que os axiomas da boa ordem e de indu c ao nao
sao independentes e sem nenhuma conex ao. De fato, eles sao equivalentes,
ou seja, se consideramos o axioma da boa ordem como sendo um postulado
podemos deduzir o axioma de indu c ao e, reciprocamente, se consideramos o
axioma de indu c ao como sendo um postulado podemos deduzir o axioma da
boa ordem. Aqui, P(n) representa a propriedade P em rela c ao ao natural
n, podendo esta se vericar ou nao.
Teorema 5.1 (Metodo de Indu cao Finita). Considere n
0
um inteiro
n ao-negativo. Suponhamos que, para cada inteiro n n
0
seja dada uma
proposic ao P(n). Se podemos provar, simultaneamente, as duas seguintes
propriedades:
(a) A proposic ao P(n
0
) e verdadeira;
(b) Se P(n) e verdadeira ent ao P(n + 1) tambem e verdadeira, para todo
n n
0
;
ent ao P(n) e verdadeira qualquer que seja n n
0
.
A arma c ao (a) e chamada de base da indu c ao e a (b) de passo indutivo.
Demonstrac ao. Denamos o conjunto
V := {k inteiros n ao-negativos; k n
0
e P(k) e verdadeira}
Notemos que V e nao-vazio, pois a condi c ao (a) nos assegura que n
0
V.
A prova do teorema e equivalente a mostrarmos que
V = {n
0
, n
0
+ 1, n
0
+ 2, n
0
+ 3, },
ou equivalentemente, a provarmos que o conjunto
F := {k inteiros nao-negativos; k n
0
e P(k) e falsa}
e vazio.
Provaremos que F e vazio por contradi c ao; para isto suponhamos que
e nao-vazio. Pelo axioma da boa ordem segue-se que existe um menor
elemento k
0
F, onde P(k
0
) e falso. Observemos que,
LivroOlimpiadaOcial
2006/5/27
page 113
i
i
i
i
i
i
i
i
[SEC. 5.2: PROBLEMAS CL

ASSICOS 113
k
0
n
0
+1. De fato, k
0
n
0
, porem a possibilidade k
0
= n
0
contradiz
a condi c ao (a);
k
0
1 V. Com efeito, P(k
0
1) e verdadeira pois, caso contrario,
k
0
1 F e, alem disto, k
0
1 < k
0
, contradizendo isto a minimalidade
de k
0
.
Finalmente, como P(k
0
1) e verdadeira, segue da condi c ao (b) que P(k
0
)
tambem e verdadeira, o que e impossvel pela deni c ao de k
0
. Portanto, o
conjunto F e vazio, concluindo assim a prova.
O grande trunfo do metodo da indu c ao e poder provar que uma quan-
tidade innita de arma c oes e verdadeira, simplesmente vericando que
uma quantidade nita destas arma c oes e verdadeira. Vamos deixar clara
a utilidade deste metodo resolvendo alguns problemas.
5.2 Problemas Classicos
Muitas descobertas em matematica sao feitas baseadas na realiza c ao de
testes que nos fornecem evidencias empricas. Tais evidencias sao estu-
dadas para efetivamente vericarmos se os resultados que elas insinuam sao
verdadeiros. O metodo de indu c ao nita constitui uma ferramenta muito
util na hora de desvendar a veracidade de resultados provenientes deste tipo
de estudo.
Dentro da grande gama de problemas que podem ser abordados apli-
cando o metodo de indu c ao podemos distinguir tres importantes grupos:
Demonstra c ao de identidades;
Demonstra c ao de desigualdades;
Demonstra c ao de problemas de divisibilidade.
A seguir damos varios exemplos de como aplicar o metodo em problemas
referentes a cada um destes grupos.
5.2.1 Demonstrando Identidades
Come camos com os seguintes problemas classicos:
LivroOlimpiadaOcial
2006/5/27
page 114
i
i
i
i
i
i
i
i
114 [CAP. 5: INDUC

AO MATEM

ATICA
(P1) Determinar uma formula para a soma dos n primeiros n umeros pares,
isto e,
S
p
(n) := 2 + 4 + 6 + + 2n.
(P2) Determinar uma formula para a soma dos n primeiros n umerosmpares,
isto e,
S
i
(n) := 1 + 3 + 5 + + 2n 1.
Para induzir ambas as formulas, primeiro fazemos os calculos para varios
valores de n, os quais apresentamos na seguinte tabela.
n 1 2 3 4 5
S
p
(n) 2 = 1 2 6 = 2 3 12 = 3 4 20 = 4 5 30 = 5 6
S
i
(n) 1 = 1
2
4 = 2
2
9 = 3
2
16 = 4
2
25 = 5
2

Os resultados na tabela sugerem que S
p
(n) = n(n + 1) e que S
i
(n) =
n
2
. Entretanto, isto nao constitui por si so uma prova rigorosa destas
formulas, pois para poder garantir a veracidade das mesmas utilizando a
tabela teramos que vericar cada valor de n natural, sendo isto impossvel.
Provaremos agora que, de fato, as formulas induzidas sao validas usando o
metodo da indu c ao nita.
Exemplo 5.2. Demonstre que para qualquer n N, e valida a igualdade:
2 + + 2n = n(n + 1).
Soluc ao. Denamos a proposi c ao
P(n) : 2 + + 2n = n(n + 1)
e observemos que a mesma vale para n = 1 (base da indu c ao), de fato
P(1) : 2 = 1(1 + 1).
Agora partimos para a prova do passo indutivo:
Hipotese: Suponhamos que P(k) e verdadeira para um certo k >
1, k N.
Tese: Devemos mostrar que P(k + 1) tambem e verdadeira.
LivroOlimpiadaOcial
2006/5/27
page 115
i
i
i
i
i
i
i
i
[SEC. 5.2: PROBLEMAS CL

ASSICOS 115
Com efeito, como
2 + + 2k = k(k + 1),
somando 2(k + 1) a ambos lados desta igualdade, temos que
2 + + 2k + 2(k + 1) = k(k + 1) + 2(k + 1)
= (k + 2)(k + 1).
Esta ultima igualdade arma que P(k + 1) tambem e verdadeira.
O Metodo da Indu c ao nos garante que P(n) e verdadeira para qualquer
n N.
Exemplo 5.3. Demonstre que para qualquer n N, e valida a igualdade:
1 + 3 + 5 + + 2n 1 = n
2
.
Soluc ao. Aqui denimos a proposi c ao:
P(n) : 1 + 3 + 5 + + 2n 1 = n
2
e notamos que a mesma e valida se tomarmos, por exemplo, n = 1 e n = 2.
De fato,
P(1) : 1 = 2 1 1 e P(2) : 1 + 3 = 1 + 2 2 1 = 4 = 2
2
.
Agora so resta provar o passo indutivo:
Hipotese: Suponhamos que P(k) seja verdadeira para um certo k >
1, k N.
Tese: Devemos mostrar que P(k + 1) tambem e verdadeira.
Com efeito, como
1 + 3 + 5 + + 2k 1 = k
2
,
somando 2k + 1 a ambos lados desta igualdade, temos que
1 + 3 + 5 + + 2k 1 + 2k + 1 = k
2
+ 2k + 1
= (k + 1)
2
.
O Princpio de Indu c ao nos garante que P(n) e verdadeira para qualquer
n N.
LivroOlimpiadaOcial
2006/5/27
page 116
i
i
i
i
i
i
i
i
116 [CAP. 5: INDUC

AO MATEM

ATICA
Uma conseq uencia imediata do exemplo 5.2 e a formula para a soma dos
n primeiros n umeros naturais, dada por
S(n) = 1 + 2 + 3 + + n =
n(n + 1)
2
. (5.1)
Com efeito, como
2 + 4 + + 2n = n(n + 1),
entao dividindo por 2 ambos os membros da igualdade acima, obtemos a
equa c ao 5.1.
Continuando com o mesmo raciocnio, e natural nos perguntarmos se e
possvel obter uma formula para a soma dos n primeiros quadrados perfeitos,
ou seja, determinar Q(n) onde:
Q(n) := 1
2
+ 2
2
+ 3
2
+ + n
2
.
Para induzir a formula, consideramos os valores de S(n) e Q(n) numa tabela:
n 1 2 3 4 5 6
S(n) 1 3 6 10 15 21
Q(n) 1 5 14 30 55 91
Aparentemente sao existe nenhuma rela c ao entre S(n) e Q(n). Mas, se
considerarmos o quociente Q(n)/S(n) vejamos o que acontece:
n 1 2 3 4 5 6
Q(n)/S(n) 3/3 5/3 7/3 9/3 11/3 13/3
Isso nos sugere que vale a relacao
Q(n)
S(n)
=
2n + 1
3
,
logo nosso candidato para valor de Q(n) e
Q(n) =
S(n)(2n + 1)
3
=
n(n + 1)(2n + 1)
6
.
Convidamos o leitor a provar a veracidade da equacao acima utilizando o metodo
da inducao.
LivroOlimpiadaOcial
2006/5/27
page 117
i
i
i
i
i
i
i
i
[SEC. 5.2: PROBLEMAS CL

ASSICOS 117
5.2.2 Demonstrando Desigualdades
Apresentamos agora alguns exemplos de como usar inducao para provar desigual-
dades.
Exemplo 5.4. Prove que 3
n1
< 2
n
2
para todo n N.
Solucao. Denotamos por P(n) a propriedade: 3
n1
< 2
n
2
.

E claro que P(1) e
valida, pois 1 < 2. Agora supondo que P(n) e verdadeira temos que
3
n
= 3
n1
3 < 2
n
2
2
2n+1
= 2
(n+1)
2
,
logo P(n + 1) tambem vale. Observamos que na desigualdade acima usamos o
fato de que 3 < 2
2n+1
para qualquer n N.
No mesmo esprito do problema anterior damos o seguinte exemplo:
Exemplo 5.5. Mostre que para todo n umero n N maior que 3 vale 2
n
< n!.
Demonstracao. Para n = 4 a desigualdade e vericada, pois 2
4
= 16 < 4! = 24.
Vamos assumir como hipotese de inducao que a desigualdade e valida para n 4.
Entao, precisamos mostrar que a mesma vale tambem para n + 1. De fato, por
hipotese de inducao:
2
n
< n! (5.2)
Como 2 < n+1, podemos multiplicar o lado esquerdo da desigualdade em 5.2 por
2 e o lado direito por n+1, sem alterar o sinal de desigualdade. Logo, temos que:
2
n
.2 = 2
n+1
< n!(n + 1) = (n + 1)!,
concluindo nossa demonstracao.
5.2.3 Inducao em problemas de divisibilidade
Agora damos alguns exemplos de problemas de divisibilidade que podem ser
mostrados utilizando o Metodo da Inducao:
Exemplo 5.6. Mostre que
n

j=1
j ! e um n umero mpar.
Solucao. A armacao e verdadeira para n = 1, pois
1

j=1
j ! = 1, que obviamente
e mpar. Partimos agora para a prova do passo indutivo:
LivroOlimpiadaOcial
2006/5/27
page 118
i
i
i
i
i
i
i
i
118 [CAP. 5: INDUC

AO MATEM

ATICA
Hipotese:
k

j=1
j ! e um n umero mpar para k > 1.
Tese:
k+1

j=1
j ! e um n umero mpar.
Com efeito,
k+1

j=1
j ! =
k

j=1
j ! + (k + 1) !
Sendo k > 1 armamos que (k + 1) ! e um n umero par, pois, (k + 1) ! = 2
[3 k (k + 1)]. Portanto,
k+1

j=1
j ! = n umero mpar + n umero par
= n umero mpar.
Exemplo 5.7. Mostre que para qualquer n N, n
3
+ 2n e sempre divisvel por
3.
Solucao. Para n = 1 a armacao e valida, pois 1
3
+ 2 1 = 3, que obviamente e
divisvel por 3.
Assumamos como hipotese indutiva que a armacao vale para algum k N,
isto e,
Hipotese: k
3
+ 2k e divisvel por3.
Devemos mostrar que a armacao tambem e verdadeira para k + 1, ou seja,
temos que provar que
Tese: (k + 1)
3
+ 2(k + 1) e divisvel por3.
Para provar isto ultimo, usamos o fato de que
(k + 1)
3
+ 2(k + 1) = (k
3
+ 3k
2
+ 3k + 1) + (2k + 2);
agrupando adequadamente,
(k + 1)
3
+ 2(k + 1) = (k
3
+ 2k) + (3k
2
+ 3k + 3)
= (k
3
+ 2k)
. .
m ultiplo de 3
+3(k
2
+k + 1)
. .
m ultiplo de 3
= m ultiplo de 3,
LivroOlimpiadaOcial
2006/5/27
page 119
i
i
i
i
i
i
i
i
[SEC. 5.3: INDUC

AO NA GEOMETRIA 119
concluindo assim a prova.
Exemplo 5.8. Mostre que a soma dos cubos de tres n umeros naturais consecu-
tivos e divisvel por 9.
Solucao. Denamos a seguinte proposicao:
p(n) : n
3
+ (n + 1)
3
+ (n + 2)
3
e um m ultiplo de nove.
Notemos que P(1) e valida, pois
1
3
+ 2
3
+ 3
3
= 1 + 8 + 27 = 36 = 9 4.
Precisamos provar agora o passo indutivo, isto e,
Hipotese: P(k) e verdadeira para algum k N.
Tese: P(k + 1) tambem e verdadeira.
Para provar isto, observamos que
(k + 1)
3
+ (k + 2)
3
+ (k + 3)
3
= (k + 1)
3
+ (k + 2)
3
+ (k
3
+ 9k
2
+ 27k + 27)
. .

.
Ordenando adequadamente o lado direito da ultima igualdade, temos que
= k
3
+ (k + 1)
3
+ (k + 2)
3
+ (9k
2
+ 27k + 27)
= k
3
+ (k + 1)
3
+ (k + 2)
3
. .
m ultiplo de 9
+9(k
2
+ 3k + 3)
. .
m ultiplo de 9
= m ultiplo de 9,
completando assim nossa demonstracao.
5.3 Indu cao na Geometria
Tratamos aqui alguns exemplos que mostram a utilidade do Metodo de Inducao
na resolucao de problemas geometricos.
Vamos comecar estudando duas propriedades importantes dos polgonos. A
primeira delas trata da soma dos angulos internos de um polgono de n lados
(n-agono).
Exemplo 5.9. Mostre que a soma dos angulos internos de um polgono de n
lados (n 3) e igual a (n 2) radianos.
LivroOlimpiadaOcial
2006/5/27
page 120
i
i
i
i
i
i
i
i
120 [CAP. 5: INDUC

AO MATEM

ATICA
A
1
A
2
A
3
A
4
(a)
A
1
A
2
A
3
A
4
A
5
(b)
Figura 5.1::
Solucao. No caso de n = 3 a propriedade acima e muito bem conhecida. Desde
Tales de Mileto e Euclides se conhecia que a soma dos angulos internos de um
triangulo e radianos. Facamos mais um caso, tomando n = 4. Neste caso,
podemos dividir um quadrilatero em dois triangulos, como mostra a Figura 5.1-
(a)
Assim, a soma dos angulos internos de um quadrilatero e 2 radianos.
Para elucidar o processo de inducao e nao deixar d uvidas sobre o que iremos
fazer, vamos considerar mais um polgono, o pentagono (n = 5). Neste caso, para
mostrar que a soma dos seus angulos internos e (5 2) = 3 radianos, iremos
dividir o pentagono A1A2A3A4A5 em um quadrilatero A1A2A3A4 e um triangulo
A1A4A5, como mostra a Figura 5.1-(b). Assim, a soma dos angulos internos do
pentagono A1A2A3A4A5 e igual `a soma dos angulos internos do triangulo A1A4A5
(igual a ) mais a soma dos angulos internos do quadrilatero A1A2A3A4 (igual a
2), ou seja, e igual a 3.
Finalmente, vamos assumir como hipotese de inducao que para um certo n 3
mostramos que a soma dos angulos internos do n-agono e (n 2). Precisamos
mostrar que a soma dos angulos internos de um n + 1-agono e [(n + 1) 2] =
(n 1). De fato, podemos repetir o processo acima. Vamos denominar de
A1, A2, . . . , An, An+1 os vertices do (n + 1)-agono. Podemos dividi-lo no n-agono
A1A2 . . . An e no triangulo A1An+1An. Logo, a soma dos angulos internos do
(n + 1)-agono e (n 2) + = (n 1)
Exemplo 5.10. Mostre que o n umero de diagonais de um polgono convexo de
n-lados e igual a
n(n3)
2
.
Solucao. Observe que para n = 3 temos que existem 0 = 3.(3 3)/2 diagonais
num triangulo. Para n = 4, temos 2 = 4(4 3)/2 diagonais num quadrilatero
convexo (veja a gura 5.2).
LivroOlimpiadaOcial
2006/5/27
page 121
i
i
i
i
i
i
i
i
[SEC. 5.3: INDUC

AO NA GEOMETRIA 121
Vamos agora assumir como hipotese de inducao que se n e um n-agono con-
vexo entao o seu n umero de diagonais e n(n 3)/2 e vamos provar que a formula
vale para um (n + 1)-agono convexo. De fato, denote por A1, A2, . . . , An, An+1
os vertices do n + 1-agono. Podemos decompo-lo como a uniao do n-agono
A1, A2, . . . , An e do triangulo A1, An, An+1. Neste caso, para contarmos as di-
agonais do (n + 1)-agono devemos considerar os seguintes casos:
Diagonais do n-agono A1, A2, . . . , An; por hipotese de inducao, o n umero
dessas diagonais e n(n 3)/2.
n 2 diagonais que partem do vertice An+1 mais a diagonal A1An.
Assim, o n umero total de diagonais do (n + 1)-agono e
n(n 3)
2
+ (n 2) + 1 =
n
2
3n + 2n 2
2
=
n
2
n 2
2
=
(n + 1)(n 2)
2
,
como queramos demonstrar.
A
1
A
2
A
3
A
4
(a)
A
1
A
2
A
3
A
4
A
5
(b)
Figura 5.2::
Exemplo 5.11. Mostre que podemos cobrir os n
2
pontos no reticulado abaixo
tracando 2n 2 segmentos de reta sem tirar o lapis do papel.
Solucao. O caso n = 3 ja foi enunciado no exemplo 1.9 do Captulo 1. A gura
abaixo mostra a solucao, onde o caminho realizado com as 4 linhas e o seguinte:
A B C D B.
Daremos a prova do problema acima por inducao. Para isso, veja que podemos
resolver o caso n = 4 continuando a solucao do caso n = 3. Como paramos num
dos vertices do quadrado 3 3, acrescentamos mais uma linha e uma coluna para
obter um reticulado 4 4. Assim, conseguimos cobrir os 16 pontos utilizando
LivroOlimpiadaOcial
2006/5/27
page 122
i
i
i
i
i
i
i
i
122 [CAP. 5: INDUC

AO MATEM

ATICA

. .
nnpontos
Figura 5.3::



A
B
C
D
Figura 5.4::
4 + 2 = 6 linhas, sem tirar o lapis do papel e cobrindo dois lados do quadrado,
como mostram as linhas descontnuas na Figura 5.5.
Finalmente, vamos assumir como hipotese de inducao que podemos cobrir
n 2 um reticulado nn com 2n2 linhas, sendo que a ultima delas cobre um dos
lados do reticulado. Acrescentando 2n+1 pontos como mostra a gura, obtemos
um reticulado (n+1) (n+1) que pode ser coberto com 2n2+2 = 2(n+1) 2
pontos, como queramos demonstrar.
LivroOlimpiadaOcial
2006/5/27
page 123
i
i
i
i
i
i
i
i
[SEC. 5.4: MISCEL

ANEA 123



A
B
C
D
Figura 5.5::
5.4 Miscelanea
Nesta secao discutiremos alguns exemplos interessantes de como podemos aplicar
o metodo da inducao aos mais variados tipos de problemas. O primeiro deles e
uma generalizacao do problema 1.5:
Exemplo 5.12. Um rei muito rico possui 3
n
moedas de ouro. Porem, uma destas
moedas e falsa e seu peso e menor que o peso das demais. Com uma balanca de
2 pratos e sem nenhum peso, mostre que e possvel encontrar a moeda falsa com
apenas n pesagens.
Solucao. Para resolver este problema, vamos utilizar o metodo da inducao. De
fato, se n = 1, procederemos da seguinte forma: pegamos duas moedas quaisquer e
colocamos na balanca, deixando uma do lado de fora. Caso a balanca se equilibre,
a moeda que esta do lado de fora e necessariamente a que tem menor peso. Caso
a balanca se desequilibre, a que tem menor peso esta na balanca, no prato mais
alto. O caso n = 2 foi feito no problema 1.5.
Vamos agora assumir como hipotese de inducao que dadas 3 moedas, podemos
achar a moeda mais leve com n pesagens. Vamos mostrar que para n+1 moedas,
precisamos de n +1 pesagens. De fato, dividiremos as 3
n+1
moedas em 3 grupos,
A, B e C com 3
n
moedas cada. Colocamos na balanca os grupos A e B. Caso
os dois grupos se equilibrem, a moeda mais leve esta no grupo C. Caso o grupo
A esteja mais leve, a moeda mais leve se encontra no grupo A. De qualquer
modo, com uma pesagem conseguimos determinar em qual grupo de 3
n
elementos
a moeda mais leve se encontra. Por hipotese de inducao, precisamos de mais n
pesagens para encontrar a moeda mais leve, totalizando n + 1 pesagens.
Exemplo 5.13. Mostre que utilizando um balde com 5 litros de capacidade e
outro com 7 litros, e possvel separar qualquer quantidade superior a 24 litros.
LivroOlimpiadaOcial
2006/5/27
page 124
i
i
i
i
i
i
i
i
124 [CAP. 5: INDUC

AO MATEM

ATICA
Solucao. Novamente, faremos a prova utilizando o metodo da inducao. Neste caso,
comecaremos o processo de inducao a partir de 24. De fato, podemos separar 24
litros utilizando duas vezes o balde de 7 e duas vezes o balde de 5 litros. Note que
o problema acima equivale a mostrar que
Todo n umero maior ou igual a 24 pode ser escrito da forma 7x +5y,
onde x e y sao n umeros inteiros maiores ou iguais a zero.
Neste caso, escrevemos 24 como 24 = 2.7 + 2.5. Por hipotese de inducao,
vamos supor que conseguimos escrever um n umero n 24 como n = 7x+5y, com
x e y n umeros inteiros maiores ou iguais a zero. Devemos mostrar que n + 1 se
escreve deste modo tambem. Para isso, vamos dividir a analise em dois casos:
Caso 1: y 3
Logo, x 2 pois se isso nao ocorresse, teramos 7x + 5y 22 < 24, o que e
impossvel. Assim, podemos escrever:
n + 1 = 7x + 5y + 1 = 7(x 2) + 5(y + 3),
pois x 2 0.
Caso 2: y 4
Neste caso, y 4 e maior que 0. Logo, podemos escrever:
n + 1 = 7x + 5y + 1 = 7(x + 3) + 5(y 4),
nalizando a nossa prova por inducao
Finalizamos esta secao com um exemplo classico conhecido como Torre de
Hanoi.
Exemplo 5.14. A Torre de Hanoi e um jogo composto por uma base com 3
hastes e n aneis colocadas num deles. Os aneis sao colocados em ordem crescente
de acordo com seu tamanho (ver Figura 5.6). Os aneis podem ser deslocados de
uma haste para qualquer outra, sendo que nunca pode ser colocado um anel maior
em cima de um menor. Prove que:
(a)

E possvel mover todos os aneis para uma das hastes que estao livres;
(b) Isto pode ser realizado usando 2
n
1 movimentos.
Solucao. Resolveremos ambas questoes usando inducao sobre n. Primeiro veri-
camos que as armacoes sao validas para n = 1, 2. Com efeito, se temos somente
um anel basta mover este para qualquer outra haste com um unico movimento
(2
1
1 = 1). Se temos 2 aneis entao movemos o menor deles para a segunda
haste, o maior para a terceira haste e, nalmente, o menor para a terceira haste,
realizando um total de 3 movimentos (2
2
1 = 3).
LivroOlimpiadaOcial
2006/5/27
page 125
i
i
i
i
i
i
i
i
[SEC. 5.5: M

ETODO FORTE DA INDUC

AO 125
Figura 5.6: Torre de Hanoi
Uma vez constatada a veracidade das armacoes para n = 1, 2, supomos
que as mesmas valem para n = k. Agora resta provar que tambem valem para
n = k + 1. De fato, se temos k + 1 aneis na primeira haste, entao pela hipotese
indutiva podemos mover todos eles, exceto o maior, para a terceira haste usando
2
k
1 movimentos. Seguidamente movemos o anel restante para a segundo haste
e, nalmente, movemos os k aneis da terceira haste para a segunda haste usando
novamente 2
k
1 movimentos. Para nalizar, notamos que foram feitos (2
k
1) +
1 + (2
k
1) = 2
k+1
1 movimentos.
5.5 Metodo Forte da Indu cao
Muitas vezes, para conseguir mostrar que a hipotese P(n + 1) e verdadeira, pre-
cisamos supor que P(k) e verdadeira para todo n0 k n. Isto e a base do
Metodo Forte da Inducao Finita que enunciamos a seguir:
Teorema 5.15 (Metodo Forte da Inducao Finita). Considere n0 um in-
teiro nao negativo. Suponhamos que, para cada inteiro n n0 seja dada uma
proposicao P(n), a qual chamaremos de hipotese indutiva. Se podemos provar,
simultaneamente, as duas seguintes propriedades:
(a) a proposicao P(n0) e verdadeira;
(b) para cada inteiro nao negativo k, com n0 k n, temos que cada uma
das proposicoes P(k) sao verdadeiras, segue-se que P(n+1) e tambem ver-
dadeira,
entao, a proposicao P(n) e verdadeira para qualquer n n0.
Vejamos agora algumas aplicacoes interessantes deste metodo.
Utilizando o Metodo Forte da Inducao, vamos dar agora a prova alternativa
do Teorema Fundamental da Aritmetica estudado no Captulo 2.
LivroOlimpiadaOcial
2006/5/27
page 126
i
i
i
i
i
i
i
i
126 [CAP. 5: INDUC

AO MATEM

ATICA
Exemplo 5.16 (Teorema Fundamental da Aritmetica). Todo n umero nat-
ural N maior que 1 pode ser escrito como um produto
N = p1 p2 p3 pm, (5.3)
onde m 1 e um n umero natural e os pi, 1 i m sao n umeros primos. Alem
disso, a fatoracao em (5.3) e unica se exigirmos que p1 p2 pm.
Solucao. Para cada n N, n 2, denamos a proposicao
P(n) : n e um n umero primo ou um produto de n umeros primos.
Notemos que P(2) e verdadeira, pois 2 e um n umero primo
Agora plantemos o passo indutivo:
Hipotese: P(k) e verdade para cada inteiro k tal que 2 k n.
Tese: P(n+1) e verdade. Em outras palavras, temos que mostrar que n+1
e um n umero primo ou ele e um produto de n umeros primos.
Faremos a prova dividindo em dois casos:
(a) Se n + 1 e um numero primo, isto mostra que P(n + 1) e verdade, e isto
acaba nossa demonstracao.
(b) Se n+1 nao e um n umero primo, entao existem , N com 2 n e
2 n tais que n + 1 = .
Nossa hipotese indutiva e valida para P() e P(). Isto signica que e
um n umero primo ou um produto de n umeros primos e que e um n umero
primo ou um produto de n umeros primos. Portanto, n + 1 = e um
produto de n umeros primos.
Exemplo 5.17. Dada a seguinte relacao de recorrencia
a0 = 8;
a1 = 10;
an = 4an1 3an2, n 2.
Mostre que sua solucao e dada por
an = 7 + 3
n
, n Z
+
.
LivroOlimpiadaOcial
2006/5/27
page 127
i
i
i
i
i
i
i
i
[SEC. 5.5: M

ETODO FORTE DA INDUC

AO 127
Solucao. Denamos a proposicao P(n) : an = 7 + 3
n
. P(0) e verdadeira, pois
P(0) = 7 + 3
0
= 7 + 1 = 8. Suponhamos que P(k) e verdadeiro para cada inteiro
k tal que 1 k n. Vamos mostrar que P(k) e verdade para k = n + 1. Com
efeito,
an+1 = 4an 3an1
= 4(7 + 3
n
) 3(7 + 3
n1
)
= 7 + 4 3
n
3 3
n1
= 7 + 3
n1
_
4 3 3
_
= 7 + 3
n1
_
9
_
= 7 + 3
n1
3
2
= 7 + 3
n+1
.
Denimos a seq uencia de Fibonacci como sendo a seq uencia Fn tal que
F1 = 1;
F2 = 1;
Fn = Fn1 +Fn2, se n 3.
A seq uencia de Fibonacci adquiriu muita fama devido a suas conexoes com
areas das mais variadas na cultura humana. Ela aparece em problemas de biologia,
arquitetura, engenharia, fsica, qumica e muitos outros. Agora vamos utilizar
inducao para mostrar algumas de suas propriedades:
Exemplo 5.18. Considere Fn a seq uencia de Fibonacci. Mostre que
Fn <
_
7
4
_
n
.
Solucao. Denamos a proposicao P(n) := Fn <
_
7
4
_
n
. Para n = 1 temos que F1 =
1 <
7
4
, de modo que P(1) e verdadeira. Suponhamos que P(1), P(2), . . . , P(n), n
2, sejam todas verdadeiras. Mostraremos que Fn+1 <
_
7
4
_
n+1
. Com efeito,
Fn+1 = Fn +Fn1
<
_
7
4
_
n
+
_
7
4
_
n1
<
7
4
_
7
4
_
n1
+
_
7
4
_
n1
<
_
1 +
7
4
_ _
7
4
_
n1
.
Como
_
1 +
7
4
_
<
_
7
4
_
2
, segue-se que Fn+1 <
_
7
4
_
2
_
7
4
_
n1
. Portanto, Fn+1 <
_
7
4
_
n+1
.
LivroOlimpiadaOcial
2006/5/27
page 128
i
i
i
i
i
i
i
i
128 [CAP. 5: INDUC

AO MATEM

ATICA
Observacao 5.19. Quando aplicamos o Metodo da Inducao devemos tomar cer-
tos cuidados. A seguir damos um exemplo de como o metodo pode ser aplicado
de forma errada. Vamos mostrar a seguinte armacao:
Armac ao: Num conjunto qualquer de n bolas , todas as bolas pos
suem a mesma cor.
Observe que nossa proposicao e claramente falsa. Mas, mesmo assim, vamos dar
uma prova por inducao:
Para n = 1, nossa proposicao e verdadeira pois em qualquer conjunto com
uma bola, todas as bolas tem a mesma cor, pois so existe uma bola. Assuma
por hipotese de inducao que a proposicao e verdadeira para n e provemos que a
proposicao e verdadeira para n + 1. Ora, seja A = {b1, . . . , bn, bn+1} o conjunto
comn+1 bolas referido. Considere os subconjuntos de B e C de Acomn elementos,
construdos como:
B = {b1, b2, . . . , bn} e C = {b2, . . . , bn+1}
Observe que ambos os conjuntos temn elementos. Assim, as bolas b1, b2, . . . , bn
do conjunto B tem a mesma cor. Do mesmo modo, as bolas do conjunto C tem
a mesma cor. Em particular, a bola bn tem a mesma cor da bola bn+1. Assim,
todas as bolas tem a mesma cor.
Ache o erro no argumento!
5.6 Exerccios
1. Considere a, b R de modo que a +b > 0 e a = b. Mostre que
2
n1
(a
n
+b
n
) > (a +b)
n
, n N.
2. Mostre que para qualquer n N e valida a seguinte desigualdade
1

1
+
1

2
+
1

3
+ +
1

n
>

n.
3. Mostre que para qualquer n N e valida a seguinte desigualdade
1
n + 1
+
1
n + 2
+
1
n + 3
+ +
1
2n
>
13
24
.
4. Mostre a seguinte identidade trigonometrica
cos x + 2 cos 2x + +ncos nx =
(n + 1) cos nx ncos(n + 1)x 1
4 sin
2 x
2
.
LivroOlimpiadaOcial
2006/5/27
page 129
i
i
i
i
i
i
i
i
[SEC. 5.6: EXERC

ICIOS 129
5. Um torneio de xadrez tem n jogadores. Cada jogador joga uma unica par-
tida contra todos os outros jogadores. Calcule o n umero total de partidas
do torneio.
6. Demonstre que para qualquer n N, e valida a igualdade:
1
3
+ 2
3
+ 3
3
. . . +n
3
=
_
n(n + 1)
2
_
2
7. Demonstre que para qualquer n N, e valida a igualdade:
1
4
+ 2
4
+ 3
4
. . . +n
4
=
n(n + 1)(2n + 1)(3n
2
+ 3n 1)
30
8. Mostre que qualquer n umero natural n 0, 11
n+2
+ 12
2n+1
e sempre di-
visvel por 133.
9. Mostre que para todo n Z
+
temos que 3
2n+1
+ 2
n+2
e um m ultiplo de 7.
10. Mostre que para todo n Z
+
temos que 3
2n+2
+ 2
6n+1
e um m ultiplo de
11.
11. Lembrando que a seq uencia de Fibonacci dene-se recursivamente por
F1 = 1;
F2 = 1;
Fn = Fn1 +Fn2,
mostre as seguintes propriedades:
(a)
n

i=1
Fi = Fn+2 1; (b)
n

i=1
F2i1 = F2n;
(c)
n

i=1
F2i = F2n+1 1; (d) Fn1Fn+1 F
2
n
= (1)
n
;
(e) FnFn+1 Fn2Fn1 = F2n1; (f)
2n1

i=1
FiFi+1 = F
2
2n
;
(g)
2n

i=1
FiFi+1 = F
2
2n+1
1; (h)
n

i=1
F
2
i
= FnFn+1.
12. Considere Fn a seq uencia de Fibonacci. Mostre que
Fn =
1

5
_
1 +

5
2
_
n

5
_
1

5
2
_
n
.
LivroOlimpiadaOcial
2006/5/27
page 130
i
i
i
i
i
i
i
i
130 [CAP. 5: INDUC

AO MATEM

ATICA
13. Calcular o n umero de regioes em que o plano e dividido por n retas distintas
em cada uma das seguintes situacoes
(a) As n retas sao concorrentes;
(b) Nao existem duas retas paralelas nem tres retas concorrentes.
LivroOlimpiadaOcial
2006/5/27
page 131
i
i
i
i
i
i
i
i
Captulo 6
Problemas de Olimpadas
Este captulo esta inteiramente dedicado `a problemas de olimpadas regionais,
nacionais e internacionais. Nele voce vai encontrar um amplo material suciente
para algumas horas (ou dias!) de diversao e treinamento para olimpadas. Para
entender as siglas de cada competicao, veja a tabela no nal do captulo. Para
as solucoes, exerccios adicionais e continuacao do treinamento, recomendamos
a leitura dos livros [11], [12] e [13], alem da visita aos sites www.obm.org.br e
www.obmep.org.br. Agora que voce ja estudou bastante, mao na massa e boa
sorte!
Problema 6.1 (OIM-1988). Seja ABC um triangulo cujos lados sao a, b e
c. Divide-se cada lado do triangulo em n segmentos iguais. Sejam sA (respec-
tivamente, sB e sC) a soma dos quadrados das distancias do vertice A a cada
um dos pontos de divisao do lado oposto a A (respectivamente, B e C). Se
S = sA +sB +SC, demonstre que
S
a
2
+b
2
+c
2
e um n umero racional.
Problema 6.2 (IMO-1979). Seja P um ponto dado dentro de uma esfera. Tres
vetores mutuamente perpendiculares partem de P e intersectam a esfera nos pon-
tos U, V e W. Q denota o vertice diagonalmente oposto a P no paraleleppedo
determinado por PU, PV e PW. Encontre o lugar geometrico de Q, para as tri-
nas de raios que partem de P.
131
LivroOlimpiadaOcial
2006/5/27
page 132
i
i
i
i
i
i
i
i
132 [CAP. 6: PROBLEMAS DE OLIMP

IADAS
Problema 6.3 (OAM-2005). Decida se o seguinte n umero e primo, justi-
cando:
(2005)
4
+ 2
42
.
Problema 6.4 (OAM-2005). Sera que existe um m ultiplo de 2005 que se escreve
utilizando apenas zeros e uns?
Problema 6.5 (OBM-2002). Mostre que, entre dezoito inteiros consecutivos de
tres algarismos, sempre existe algum que e divisvel pela soma de seus algarismos.
Problema 6.6 (IMO-1983). Diga se e possvel escolher 1983 n umeros inteiros
positivos e distintos, todos menores ou iguais que 10
5
, onde nenhum de tres in-
teiros consecutivos estao em progressao aritmetica? Justique sua resposta.
Problema 6.7 (IMO-1978). Considere a seq uencia de inteiros positivos distin-
tos {a
k
}

k=1
. Mostre que
n

k=1
a
k
k
2

n

k=1
1
k
, n N.
Problema 6.8 (IMO-1979). Sejam p e q n umeros naturais tais que
p
q
= 1
1
2
+
1
3

1
4
+
1
1318
+
1
1319
.
Mostre que p e divisvel por 1979.
Problema 6.9 (IMO-1979). Considere como sendo um plano e tome P e
Q / . Encontre todos os pontos R tal que a razao
QP +PR
QR
seja maxima.
Problema 6.10 (IMO-1978). Sejam m e n n umeros naturais tais que 1
m < n. Se na representacao decimal de 1978
m
temos que seus tres ultimos dgitos
coincidem com os tres ultimos dgitos da representacao decimal de 1978
n
, encontre
m e n de modo que m+n seja o menor valor possvel.
Problema 6.11 (IMO-1979). Encontre todos os n umeros reais a para os quais
existem n umeros reais nao negativos x1, x2, x3, x4, x5 satisfazendo as relacoes
5

k=1
kx
k
= a,
5

k=1
k
3
x
k
= a
2
,
5

k=1
k
5
x
k
= a
3
.
Problema 6.12 (IMO-1981). Determine o valor maximo de m
2
+n
2
, onde m
e n sao inteiros satisfazendo m, n {1, 2, . . . , 1981} e (n
2
mn m
2
)
2
= 1.
LivroOlimpiadaOcial
2006/5/27
page 133
i
i
i
i
i
i
i
i
133
Problema 6.13 (IMO-1981). Resonda as seguintes questoes:
(a) Para quais valores de n > 2 existe um conjunto de n inteiros positivos
consecutivos de modo que o maior elemento neste conjunto seja um divisor
do mnimo m ultiplo comum dos restantes n 1 n umeros?
(b) Para quais valores de n > 2 existe exatamente um conjunto com a pro-
priedade enunciada em (a)?
Problema 6.14 (IMO-1981). A funcao f(x, y) satisfaz
f(0, y) =y + 1,
f(x + 1, 0) =f(x, 1),
f(x + 1, y + 1) =f(x, f(x + 1, y)),
para todos os inteiros nao negativos x, y. Determine f(4, 1981).
Problema 6.15 (IMO-1982). A funcao f(n) esta denida para todos os inteiros
positivos n e toma seus valores nos inteiros nao negativos. Tambem para quaisquer
m e n temos que
f(m+n) f(m) f(n) = 0 ou 1
f(2) = 0, f(3) > 0, e f(9999) = 3333.
Ache f(1982).
Problema 6.16 (IMO-1982). Prove que se n e um inteiro positivo tal que a
equacao
x
3
3xy
2
+y
3
= n
tem uma solucao inteira (x, y), entao existem pelo menos outras tres solucoes
inteiras. Demonstre que a equacao nao tem solucao inteira quando n = 2891.
Problema 6.17 (IMO-1982). Seja ABCDEF um hexagono regular. Sejam M
um ponto sobre a diagonal AC e N um ponto sobre a diagonal CE, de modo que
AM
AC
=
CN
CE
= r.
Determine r no caso em que B, M e N sejam colineares.
Problema 6.18 (IMO-1985). Dado um conjunto M de 1985 inteiros positivos
distintos onde nenhum deles tem um divisor primo maior que 26, prove que M
contem pelo menos um subconjunto de quatro elementos distintos cujo produto e
a quarta potencia de um inteiro.
Problema 6.19 (IMO-1977). Seja f(n) uma funcao denida no conjunto dos
inteiros positivos e tomando valores no mesmo conjunto. Prove que se f(n+1) >
f(f(n)), para cada inteiro positivo n, entao f(n) = n , n.
LivroOlimpiadaOcial
2006/5/27
page 134
i
i
i
i
i
i
i
i
134 [CAP. 6: PROBLEMAS DE OLIMP

IADAS
Problema 6.20 (IMO-1975). Quando 4444
4444
e escrito em notacao decimal,
temos que a soma de seus dgitos e A. Agora, denote por B a soma dos dgitos de
A. Encontre a soma dos dgitos de B. (A e B foram escritos em notacao decimal)
Problema 6.21 (IMO-1963). Encontre todas as razes da equacao
_
x
2
p + 2
_
x
2
1 = x,
onde p e um parametro real.
Problema 6.22 (OCM-2003). Um homem acha-se no centro de um crculo. A
periferia desse crculo e delimitada por uma cerca, que separa um homem de um
cachorro. Admitindo que o cachorro so pode correr ao longo da cerca, prove que o
homem pode escapar pulando a cerca sem ser mordido pelo cao se as velocidades
maximas possveis de serem desenvolvidas pelo cachorro e pelo homem estiverem
entre si na razao 4:1. Determine as relacoes entre as velocidades maximas do
cachorro e do homem para os quais o homem pode escapar.
Problema 6.23 (IMO-1997). Sejam x1, x2, ..., xn n umeros reais que vericam
as condicoes:
|x1 +x2 +... +xn| = 1 e |xi|
n + 1
2
, para i = 1, 2, . . . , n.
Demonstrar que existe uma reordenacao (ou permutacao) y1, y2, ..., yn de x1, x2, ..., xn
tal que
|y1 + 2y2 + +nyn|
n + 1
2
.
Problema 6.24 (IMO-1997). Determinar todos os pares (a, b) de inteiros a
1, b 1 que satisfazem a equacao
a
b
2
= b
a
.
Problema 6.25 (IMO-2002). Seja n um inteiro positivo e T o conjunto de
pontos (x, y) no plano onde x e y sao inteiros nao negativos com x + y < n.
Cada ponto de T e pintado de vermelho ou azul. Se um ponto (x, y) e vermelho,
entao todos os pontos (x

, y

) com x

x e y

y tambem sao. Um conjunto


X e um conjunto de n pontos azuis com abscissas distintas, e um conjunto Y e
um conjunto de n pontos azuis com ordenadas distintas. Prove que o n umero de
conjuntos X e igual ao n umero de conjuntos Y.
Problema 6.26 (IMO-2000). No incio existem n (n 2) pulgas numa reta
horizontal, nem todas no mesmo ponto. Para um n umero real positivo dene-se
um salto da seguinte maneira: Escolhem-se duas pulgas quaisquer nos pontos A e
LivroOlimpiadaOcial
2006/5/27
page 135
i
i
i
i
i
i
i
i
135
B com o ponto A `a esquerda do ponto B; A pulga que esta em A salta ate o ponto
C da reta, `a direita de B, tal que
BC
AB
= .
Determine todos os valores de para os quais qualquer ponto M na reta e quais-
quer que sejam as posicoes iniciais das n pulgas, existe uma sucessao nita de
saltos que levam todas as pulgas para pontos `a direita de M.
Problema 6.27 (IMO-2000). Um magico tem cem cartoes numerados de 1 a
100. Coloca-os em tres caixas, uma vermelha, uma branca e uma azul, de modo
que cada caixa contem pelo menos um cartao. Uma pessoa da plateia escolhe duas
das tres caixas, seleciona um cartao de cada caixa e anuncia a soma dos n umeros
dos dois cartoes que escolheu. Ao saber esta soma, o magico identica a caixa
da qual nao se retirou nenhum cartao. De quantas maneiras podem ser colocados
todos os cartoes nas caixas de modo de que este truque sempre funcione? (Duas
maneiras consideram-se diferentes se pelo menos um cartao e colocado numa caixa
diferente).
Problema 6.28 (IMO-2001). Sejam a, b, c, d inteiros com a > b > c > d > 0.
Considere que
ac +bd = (b +d +a c)(b +d a +c).
Prove que ab +cd nao e um n umero primo.
Problema 6.29 (OCS-1997). Seja n um n umero natural, n > 3. Demonstre
que entre os m ultiplos de 9 menores que 10n ha mais n umeros com a soma de seus
dgitos igual a 9(n 2) que n umeros com a soma de seus dgitos igual a 9(n 1).
Problema 6.30 (OCS-1997). Considere um triangulo acutangulo ABC, e seja
X um ponto do plano do triangulo. Sejam M, N e P as projecoes ortogonais de
X sobre as retas que contem as alturas do triangulo ABC. Determinar para que
posicoes de X o triangulo MNP e congruente a ABC. Nota: a projecao ortogonal
de um ponto X sobre uma reta l e a interseccao de l com a perpendicular a ela
que passa por X.
Problema 6.31 (OPM-2001). Um pacote com 100 bombons deve ser dividido
entre 15 garotos.
(a) Prove que existem dois garotos que receberam a mesma quantia de bombons.
(b) Qual o n umero mnimo de bombons no pacote, de modo que, depois de di-
vididos entre os 15 garotos, e possvel que nao existam dois garotos que
receberam a mesma quantia de bombons.
Obs: Considere que algum garoto pode receber zero bombons.
LivroOlimpiadaOcial
2006/5/27
page 136
i
i
i
i
i
i
i
i
136 [CAP. 6: PROBLEMAS DE OLIMP

IADAS
Problema 6.32 (OEMRJ-1998). Encontre todas as solucoes inteiras e positi-
vas de
1
x
+
1
y
=
1
p
,
onde p e um n umero primo (cada solucao e um par ordenado (x, y)). Encontre
pelo menos 5 solucoes inteiras e positivas de
1
x
+
1
y
=
1
1998
,
Problema 6.33 (OMRN-2000).

E possvel colocar os inteiros 1, 2, 3, 4, ...,
239, 240 numa tabela com 15 linhas e 16 colunas, de modo que a soma dos n umeros
em cada uma das colunas seja a mesma?
Problema 6.34 (ORMRS-1998). Mostrar que para cada n inteiro positivo:
_
1 +
1
n
_
n
< 3.
Problema 6.35 (OSM-1999). Os segmentos de reta AO, OB e OC sao mutu-
amente perpendiculares. Expresse a area do triangulo ABC em termos das areas
dos triangulos OBC, OCA e OAB.
Problema 6.36 (OCS-2004). Dada uma circunferencia C e um ponto P exte-
rior a ela, tracam-se por P as duas tangentes `a circunferencia, sendo A e B os
pontos de tangencia. Toma-se um ponto Q sobre o menor arco AB de C. Sejam
M a interseccao da reta AQ com a perpendicular a AQ tracada por P e N a in-
terseccao da reta BQ com a perpendicular a BQ tracada por P. Demonstre que,
ao variar Q no arco AB, todas as retas MN passam por um mesmo ponto.
Problema 6.37 (OM-2004). Ache todos os n umeros naturais x, y, z que veri-
cam simultaneamente:
xyz = 4104 e x +y +z = 77.
Problema 6.38 (OMEG-1999). Quantos retangulos diferentes constitudos de
um n umero inteiro de quadrados podem ser desenhados:
(a) Em um tabuleiro (xadrez) 8 8?
(b) Em um tabuleiro n n?
Observacao: consideramos dois retangulos diferentes se eles possurem di-
mensoes diferentes ou se ocuparem posicoes diferentes no tabuleiro.
Problema 6.39 (ORMSC-1999). Mostre que um n umero natural diferente de
1 formado somente por algarismos 1, nao pode ser um quadrado perfeito.
LivroOlimpiadaOcial
2006/5/27
page 137
i
i
i
i
i
i
i
i
137
Problema 6.40. Uma codorna e uma peca que pode se mover num quadriculado
innito no seguinte sentido: em qualquer quadrado seu primeiro movimento e
para um quadrado vizinho (horizontal ou vertical) e depois se move n quadrados
em direcao perpendicular ao primeiro movimento. Por exemplo, se n = 2, a
codorna e um cavalo do xadrez. Encontre todos os n tais que a codorna consegue
alcancar qualquer quadrado a partir de um quadrado qualquer.
Problema 6.41 (OBM-1982). De as solucoes inteiras da equacao x
2
+15
a
= 2
b
.
Problema 6.42 (OBM-1981). Encontre o n umero de solucoes inteiras, nao
negativas, da equacao x +y +z = n, onde n e um inteiro positivo.
Problema 6.43 (OBM-1982). Sejam x, y, z n umeros inteiros tais que x
3
+y
3

z
3
e m ultiplo de 7. Mostre que um desses n umeros e m ultiplo de 7.
Problema 6.44 (OBM-1984). Quantos quadrados perfeitos existem entre 40.000
e 640.000 que sao m ultiplos simultaneamente de 3, 4 e 5?
Problema 6.45 (OBM-1986). Dado um n umero natural n, determine de quan-
tas maneiras esse n umero pode ser representado como soma de naturais consecu-
tivos.
Problema 6.46 (OBM-1984). Seja n um n umero inteiro maior que 1. Mostre
que 4
n
+n
4
nao e primo.
Problema 6.47 (OBM-1983). Mostre que, para todo n umero natural n 2,
1 +
1
2
+
1
3
+ +
1
n
nao e inteiro.
Problema 6.48 (OBM-1981). Dado um n umero inteiro n, mostre que existe
um m ultiplo de n que se escreve apenas com os algarismos 0 e 1. (Por exemplo,
se n = 3, temos 111 ou 1011, etc).
Problema 6.49 (IMO-1985). Seja D um inteiro diferente de 2, 5, 13. Mostre
que podemos escolher A e B pertencentes a {2, 5, 13, D} tal que AB 1 nao seja
quadrado perfeito.
LivroOlimpiadaOcial
2006/5/27
page 138
i
i
i
i
i
i
i
i
138 [CAP. 6: PROBLEMAS DE OLIMP

IADAS
Problema 6.50 (OBM-1982). Um retangulo de lados inteiros m e n e dividido
em quadrados de lado 1. Um raio de luz entra no retangulo por um dos vertices,
na direcao da bissetriz do angulo reto, e e reetido sucessivamente nos lados do
retangulo. Quantos quadrados sao atravessados pelo raio de luz?
Problema 6.51 (OBM-1981). Um clube de Matematica contem 100 membros.
Suponha que em qualquer grupo de 4 membros, existe um membro que conheca
os outros 3. Prove que existe um membro do clube que conhece todos os outros
99 membros. Qual e o n umero mnimo de tais membros? (Obs.: se o membro A
conhece o membro B, entao o membro B tambem conhece o membro A).
Problema 6.52 (OBM-1985). Apenas 5 casais participam de uma reuniao.
Apos os cumprimentos, Joao pergunta a cada um dos outros 9 participantes:
Quantos apertos de mao voce deu?e obtem todas as nove respostas possveis:
0, 1, 2, 3, 4, 5, 6, 7, 8. Qual foi a resposta da esposa do Joao? (Obs.: obviamente
ninguem apertou a mao do proprio conjugue.)
Problema 6.53 (OBM-1983). Quantos n umeros, de 1 a 1983, podem ser es-
critos como soma de duas ou mais potencias distintas de 3?
Problema 6.54 (OBM-1983). Os n umeros a, b e c sao reais nao-negativos e p
e q sao inteiros positivos distintos. Prove que:
Se
_
a
p
+b
p
= c
p
a
q
+b
q
= c
q
, entao a = 0 ou b = 0.
Problema 6.55 (OBM-1985). Prove que se a, b e c sao n umeros reais tais que
a, b, c sao positivos e a +b +c = 1, entao a
2
+b
2
+c
2
1/3.
Problema 6.56. O ponto A e chamado pseudocentro de simetriade um con-
junto M (que contem mais de um ponto do plano) se e possvel remover um ponto
de M de modo que A seja o centro de simetria do conjunto resultante. Quantos
pseudocentros de simetria um conjunto nito pode ter?
Problema 6.57 (OBM-1983). Mostre que e possvel, usando apenas duas cores,
pintar os pontos de uma circunferencia de tal forma que nao exista triangulo
retangulo inscrito na circunferencia com vertices em pontos da mesma cor.
Problema 6.58 (OBM-1984). Colam-se 27 cubos de madeira, todos iguais,
para montar um cubo maior. Um cupim parte do centro de um cubo pequeno,
LivroOlimpiadaOcial
2006/5/27
page 139
i
i
i
i
i
i
i
i
139
colocado no centro de uma das faces do cubo grande. O cupim liga, sempre por
linhas retas, o centro de um dos cubos pequenos ao centro de outro cubo pequeno
com face adjacente ao primeiro. Desta forma, existe um caminho que passe ex-
atamente uma vez por cada um dos pequenos cubos e termine no centro do cubo
grande?
Problema 6.59 (OBM-1985). Dois triangulos semelhantes tem dois pares de
lados ordenadamente iguais. Qual a condicao para que os dois triangulos nao se-
jam congruentes?
Problema 6.60 (OBM-1979). Sejam ABCD um quadrado, M o ponto medio
de AB, N o ponto medio de BC e I a interseccao de DN e CM. Calcule a area
do triangulo NIC, tomando AB = 1.
Problema 6.61 (OAM-2004). Um n umero n e dito dobrado se ao escrevermos
seus dgitos na ordem inversa, obtemos exatamente o dobro de n. Por exemplo,
2004 nao e dobrado, pois 4002 = 2 2004. Mostre que nao existem n umeros
dobrados menores que 1000.
Problema 6.62. Tres amigos, Kleber, Ferdinando e Adao, que nunca mentem,
se reuniram e travou-se a seguinte conversa:
- Kleber fala: estou escolhendo dois n umeros inteiros positivos maiores que
um, cujos valores nao revelo e vou dar, em segredo, a soma deles para Ferdinando
e o produto deles para o Adao.
- Ferdinando fala: o valor que me foi dado nao excede 16. Adao fala: eu nao
consigo achar os valores dos dois n umeros escolhidos pelo Kleber.
- Ferdinando fala: eu ja sabia que voce nao encontraria os valores dos dois
n umeros.
- Adao fala: ah, entao eu sei quais sao os dois n umeros.
Responda: Qual o valor que foi dado a Ferdinando ? Baseado em que Adao
fez a ultima armativa ?
Problema 6.63 (OBM-2001). Mostre que nao existem dois n umeros inteiros
a e b tais que (a +b)(a
2
+b
2
) = 2001.
Problema 6.64. Considere um quadrado de lado x. Com centro em cada vertice
tracam-se quatro circunferencias de raio x. Determinar a area do quadrilatero
curvilneo interior ao quadrado dado.
LivroOlimpiadaOcial
2006/5/27
page 140
i
i
i
i
i
i
i
i
140 [CAP. 6: PROBLEMAS DE OLIMP

IADAS
Problema 6.65 (OBM-1981). Dados n pontos no plano, prove que existem 3
deles que determinam um angulo menor ou igual a

n
.
Problema 6.66 (OBM-1985). Qual o algarismo das unidades do n umero N =
1 3 5 7 1993?
Problema 6.67. Seja p o maior fator primo do n umero 3
14
+ 3
13
12. Qual o
valor de p?
Problema 6.68. Prove que entre 13 n umeros reais quaisquer y1, . . . , y13 ha dois
deles tais que:
0
yi yj
1 +yiyj

3
2 +

3
.
Problema 6.69 (OIM-1991). Encontre um n umero m de cinco algarismos dis-
tintos e nao nulos que seja igual `a soma de todos os n umeros de tres algarismos
distintos que se pode formar a partir dos algarismos de m.
Problema 6.70. Dado um conjunto M de 555 inteiros positivos, nenhum dos
quais tem divisor primo maior que 26, prove que M contem um subconjunto de
dois elementos distintos cujo produto e um quadrado perfeito.
Problema 6.71 (L.Putnan-1956). Demonstrar que todo inteiro tem um m ultiplo
cuja representacao decimal contem os 10 dgitos 1, 2, 3, 4, 5, 6, 7, 8, 9 e 0.
Problema 6.72 (OBM-2001). Dizemos que um n umero natural e legal quando
for soma de dois naturais consecutivos e tambem for soma de tres naturais con-
secutivos.
(a) Mostre que 2001 e legal, mas 1999 e 2002 nao sao legais.
(b) Mostre que 2001
2001
e legal.
Problema 6.73 (IMO-1983). Diga se e possvel escolher 1983 n umeros inteiros
positivos e distintos, todos menores ou iguais que 10
5
, onde nenhum de tres in-
teiros consecutivos estao em progressao aritmetica.
Problema 6.74. Considere a, b e c os comprimentos dos lados de um triangulo.
Mostre que
a
2
b(a b) +b
2
c(b c) +c
2
a(c a) 0.
Diga quando a igualdade ocorre.
LivroOlimpiadaOcial
2006/5/27
page 141
i
i
i
i
i
i
i
i
141
Problema 6.75 (IMO-1984). Sejam a, b, c e d n umeros inteiros mpares tais
que 0 < a < b < c < d e ad = bc. Suponha que existam inteiros k e m tais que
a +d = 2
k
e b +c = 2
m
. Mostre que a = 1.
Problema 6.76 (IMO-1984). Sejam x, y e z n umeros reais nao negativos tais
que x +y +z = 1. Mostre que
0 yz +zx +xy 2xyz
7
27
.
Problema 6.77. Resolva a equacao
3

x +
3

x 16 =
3

x 8.
Problema 6.78. Resolva o sistema de equacoes
_
_
x
y
+

xy =
80

xy
,
x +y = 20.
Problema 6.79. Encontre todos os polinomios P(x) de modo que
P(x
2
2x) = (P(x 2))
2
.
Problema 6.80. Determine os valores de p R tal que
2 <
x
2
+ 2px 2
x
2
2x + 2
< 2.
Problema 6.81. Mostre que para cada a > 0 temos:
a
4
+ 9
10a
>
4
5
.
Problema 6.82 (OCOM-2004). Numa lista de 10 n umeros inteiros positivos e
dada.

E permitido escolher tres termos consecutivos da lista e somar uma unidade
a cada um deles. Determine se sempre possvel, apos uma quantidade nita de
operacoes deste tipo, que todos os termos da lista sejam m ultiplos de quatro.
Problema 6.83 (IMO-1964). Prove que nao existe n tal que 2
n
+ 1 e divisvel
por 7 e ache todos os inteiros positivos n tais que 2
n
1 e divisvel por 7.
Problema 6.84 (OIM-1987). Encontre os f(x) tais que [f(x)]
2
.f(1x/1+x) =
64x para todo x distinto de 0, 1 e 1.
Problema 6.85. Prove que se m, n e r sao inteiros positivos, nao nulos, e 1 +
m+n

3 = (2 +

3)
2r1
entao e um quadrado perfeito.
LivroOlimpiadaOcial
2006/5/27
page 142
i
i
i
i
i
i
i
i
142 [CAP. 6: PROBLEMAS DE OLIMP

IADAS
Problema 6.86 (OIM-1987). Se dene a sucessao pn da seguinte maneira:
p1 = 2 e para todo maior ou igual a 2, pn e o maior divisor primo da expressao
p1p2...pn + 1. Prove que pn e diferente de 5.
Problema 6.87 (OIM-1988). Considere os conjuntos de n n umeros naturais
diferentes de zero nos quais nao ha tres elementos em progressao aritmetica.
Demonstre que em um desses conjuntos a soma dos inversos desses elementos
e maxima.
Problema 6.88 (OIM-1990). Seja f(x) = (x +b)
2
c um polinomio com b e
c n umeros inteiros.
(a) Se p e um n umero primo tal que p divide a c e p
2
nao divide a c, demonstrar
que qualquer que seja o n umero inteiro n, p
2
nao divide a f(n).
(b) Seja um n umero primo, distinto de 2 que nao divide a c.Se q divide a f(n)
para algum n umero inteiro n, demonstrar que para cada inteiro positivo r,
existe um n umero inteiro n

tal que q divide a f(n

).
Problema 6.89 (OIM-1991). Encontrar um n umero inteiro N de cinco dgitos
diferentes e nao nulas, que seja igual a soma de todos os n umeros de tres dgitos
distintos que se podem formar com os cinco dgitos de N.
Problema 6.90 (OIM-1992). Para cada inteiro positivo n, seja an o ultimo
dgito do n umero1 + 2 + 3 +... +n. Calcular a1 +a2 +... +a1992.
Problema 6.91 (OIM-1993). um n umero natural e capicua se ao escreve-lo
em notacao decimal, podemos le-lo da mesma forma da esquerda para a direita
como de direita para a esquerda, por exemplo 8, 23432, 6446. Sejam x1 < x2 <
... < xi < xi+1 < ... todos os n umeros capicuas. Para cada i seja yi = xi+1 xi.
Quantos n umeros primos distintos tem o conjunto {y1, y2, ...} ?
Problema 6.92 (OIM-1995). Determine os possveis valores das somas dos
dgitos de todos os quadrados perfeitos.
Problema 6.93 (OBMEP-2005). Capit u cortou uma folha de papel retangular
em 9 pedacos quadrados de lados 1, 4, 7, 8, 9, 10, 14, 15 e 18 centmetros cada
um.
(a) Qual e a area da folha antes de ser cortada?
(b) Quais eram as dimensoes da folha antes de ser cortada?
(c) Capit u precisa montar a folha de novo. Ajude-a, mostrando com um de-
senho, como fazer esta montagem.
LivroOlimpiadaOcial
2006/5/27
page 143
i
i
i
i
i
i
i
i
143
Problema 6.94. Fatore o n umero 5
1985
1 como o produto de tres inteiros, cada
um dos quais e maior que 5
100
.
Problema 6.95. m caixas sao dadas, cada uma contendo um certo n umero de
bolas. Se n < m uma operacao permitida e escolher n caixas e acrescentar uma
bola em cada uma delas. Prove que
(a) Se m e n sao primos entre si, entao e possvel apos um n umero nito de
operacoes deste tipo chegar a situacao onde cada caixa contem a mesma
quantidade de bolas;
(b) Se m e n nao sao primos entre si, existem conguracoes iniciais das bolas
de modo que nao e possvel chegar a situacao onde todas as caixas contem
a mesma quantidade de bolas.
Problema 6.96 (IMO-1981). Determine o valor maximo de m
2
+n
2
, onde m
onde n sao inteiros satisfazendo n, m {1, 2, . . . , 1981} e (n
2
mn m
2
)
2
= 1.
Problema 6.97 (IMO-1983).

E possvel escolher 1983 inteiros positivos distin-
tos, todos menores ou iguais a 10
5
, sem quaisquer tres deles nao sejam termos
consecutivos de uma progressao aritmetica? Justique sua resposta.
Problema 6.98 (IMO-1984). Ache um par de inteiros a, b tais que:
(a) ab(a +b) e divisvel por 7;
(b) (a +b)
7
a
7
b
7
e divisvel por 7
7
.
Justique sua resposta.
LivroOlimpiadaOcial
2006/5/27
page 144
i
i
i
i
i
i
i
i
144 [CAP. 6: PROBLEMAS DE OLIMP

IADAS
Siglas
Competicao Sigla
Olimpada Internacional de Matematica IMO
Olimpada Ibero-Americana de Matematica OIM
Olimpada do Cone Sul OCS
Olimpada de Maio OM
Olimpada Brasileira de Matematica OBM
Olimpada Brasileira de Matematica das Escolas P ublicas OBMEP
Olimpada Colombiana de Matematica OCOM
Olimpada Alagoana de Matematica OAM
Olimpada Cearense de Matematica OCM
Olimpada de Matematica do Estado de Goias OMEG
Olimpada de Matematica do Estado do Rio de Janeiro OMERJ
Olimpada Paraense de Matematica OPM
Olimpada de Matematica do Rio Grande do Norte OCM
Olimpada Regional de Matematica do Rio Grande do Sul ORMRS
Olimpada Regional de Matematica de Santa Catarina ORMSC
Olimpadas Sergipanas de Matematica OSM
LivroOlimpiadaOcial
2006/5/27
page 145
i
i
i
i
i
i
i
i
Referencias Bibliogracas
[1] FOMIN, Dimitri & GENKIN, Sergei & ITENBERG, Ilia. Mathematical
Circles(Russian Experience). American Mathematical Society. Mathemat-
ical World, Volume 7, 1996. 272 p.
[2] LIMA, Elon L. & PINTO, Paulo C. & WAGNER, Eduardo & MORGADO,
Augusto C. Temas e Problemas. Sociedade Brasileira de Matematica.
Colecao do Professor de Matematica, 2001. 193 p.
[3] RIBENBOIM, Paulo. N umeros Primos: Misterios e Recordes. Colecao
Matematica Universitaria, IMPA, 2001. 280 p.
[4] OLIVEIRA SANTOS, Jose Plnio de. Introducao `a Teoria dos N umeros.
Colecao Matematica Universitaria, IMPA, 1993. 199 p.
[5] VINOGRADOV, I. Fundamentos de la Teoria de los N umeros. Editorial
MIR, Moscu, 1987. 207 p.
[6] OBM: Olimpada Brasileira de Matematica. www.obm.org.br
[7] OBMEP: Olimpada Brasileira de Matematica das Escolas P ublicas.
www.obmep.org.br
[8] NIVEN, I., ZUCKERMAN, S., MONTGOMERY, L. An Introduction To
The Theory Of Numbers. John Wiley & Sons, Unided States of America,
1991, 529 p.
[9] ANDREWS, G. Number Theory. Dover Publications, Estados Unidos da
America, 1994, 259 p.
[10] MORGADO, A., CARVALHO, J., CARVALHO, P., FERNANDEZ, P.
Analise Combinatoria e Probabilidade. SBM, Brasil, 1991, 343 p.
[11] WAGNER, E., MOREIRA, C. 10 Olimpadas Iberoamericanas de
Matematica. OEI, Espanha, 1996, 277 p.
[12] MEGA, E., WATANABE, R. Olimpadas Brasileiras de Matematica,
1
a
a 8
a
. Editora N ucleo, Brasil, 1988, 403 p.
145
LivroOlimpiadaOcial
2006/5/27
page 146
i
i
i
i
i
i
i
i
146 REFER

ENCIAS BIBLIOGR

AFICAS
[13] GREITZER, S. International Mathematical Olympiads. The Mathe-
matical Association of America, Estados Unidos da America, 1978, 201 p.

Você também pode gostar